Science

NCERT | Science | Objective | Class-XI | सामान्य विज्ञान (जीव विज्ञान)

WhatsApp Group Join Now
Telegram Group Join Now

NCERT | Science | Objective | Class-XI | सामान्य विज्ञान (जीव विज्ञान)

जीव विज्ञान

1. जीव जगत

1. उपापचयी क्रियाओं (Metabolic Process) के संदर्भ में, निम्नलिखित कथनों पर विचार कीजिये :
1. जीवों के शरीर में होने वाली सभी रासायनिक क्रियाएँ उपापचयी क्रियाएँ हैं।
2. जीवों के शरीर में होने वाली सभी भौतिक क्रियाएँ उपापचयी क्रियाएँ हैं।
3. उपापचयी क्रियाएँ निर्जीवों में भी संपन्न होती हैं।
4. उपापचयी क्रियाएँ एककोशिकीय तथा बहुकोशिकीय दोनों प्रकार के जीवों में होती हैं।
उपर्युक्त कथनों में से कौन-से सही हैं?
(a) केवल 1 और 3
(b) केवल 1 और 4
(c) केवल 2 और 3
(d) 1, 2, 3 और 4
उत्तर : (b)
व्याख्या: उपापचयन जीवों का एक लक्षण है तथा सभी जीव रसायनों से बने होते हैं। ये रसायन छोटे, बड़े, विभिन्न वर्ग, माप और क्रिया वाले होते हैं जो अनवरत जैव अणुओं में बदलते रहते हैं। रसायनों का जैव अणुओं में होने वाला यह परिवर्तन रासायनिक अथवा उपापचयी क्रिया है। जीवों के शरीर में होने वाली सभी रासायनिक क्रियाएँ, उपापचयी क्रियाएँ हैं। अतः कथन ( 1 ) सही है ।
• चूँकि उपापचयी क्रियाएँ रासायनिक क्रियाएँ हैं। अतः कथन (2) गलत है।
• किसी भी निर्जीव में उपापचयी क्रियाएँ संपन्न नहीं हो सकतीं। अतः कथन ( 3 ) गलत है।
• कथन (4) सही है क्योंकि सभी जीवों में, चाहे वे बहुकोशिकीय हों अथवा एककोशिकीय, हज़ारों उपापचयी क्रियाएँ साथ-साथ चलती रहती हैं।
अतः उपर्युक्त प्रश्न का विकल्प (b) सही है ।
2. जीवों में जनन के संदर्भ में, निम्नलिखित कथनों पर विचार कीजिये :
1. जीव केवल लैंगिक जनन करते हैं।
2. जीव केवल अलैंगिक जनन करते हैं।
3. जीव लैंगिक तथा अलैंगिक दोनों प्रकार के जनन करते हैं।
4. जीवों के लिये लैंगिक जनन, अलैंगिक जनन से अधिक लाभकारी है।
उपर्युक्त कथनों में से कौन – सा / से सत्य है/हैं?
(a) केवल 1
(b) केवल 2
(c) केवल 3
(d) केवल 3 और 4
उत्तर : (d)
व्याख्या: कथन 1 और कथन 2 असत्य हैं क्योंकि जीवों में न तो केवल लैंगिक जनन और न ही केवल अलैंगिक जनन होता है बल्कि कुछ जीव लैंगिक तथा कुछ अलैंगिक जनन द्वारा संतति उत्पन्न करते हैं।
• कथन (3) सत्य है क्योंकि लैंगिक तथा अलैंगिक दोनों प्रकार से जनन कर संतति उत्पन्न करने वाले जीव पाए जाते हैं।
• कथन (4) सत्य है क्योंकि अलैंगिक जनन आनुवंशिक सूचनाओं को परिरक्षित रखता है, जबकि लैंगिक जनन द्वारा विभिन्नता व विशिष्ट आनुवंशिक व्यवस्था के संयोजन के प्रतिपादन का अवसर मिलता है, जो जीव या आबादी हेतु लाभकारी हो सकता है। लैंगिक जनन से विभिन्नता भी उत्पन्न होती है।
अतः उपर्युक्त प्रश्न का विकल्प (d) सही उत्तर है ।
3. निम्नलिखित में से कौन-सा वर्गिकी संवर्ग के संबंध में सही क्रम है?
(a) प्रजाति (Species) → वंश ( Genus) → संघ (Phylum) → वर्ग (Class) → जगत (Kingdom)
(b) वंश (Genus) → प्रजाति (Species) → वर्ग (Class) → गण ( Order) → जगत ( Kingdom)
(c) वर्ग (Class) → प्रजाति (Species) → वंश (Genus) → संघ (Phylum ) → जगत (Kingdom)
(d) प्रजाति (Species) → वंश (Genus) → वर्ग (Class) → संघ (Phylum ) → जगत (Kingdom)
उत्तर : (d)
व्याख्या:
• वर्गिकी संवर्ग में पदानुक्रम सोपान होते हैं, पौधों तथा प्राणियों दोनों में प्रजाति (स्पीशीज़) सबसे निचले संवर्ग में आती है। इसके बाद स्पीशीज़ का एक वर्ग आता है, जिसे वंश कहते हैं तथा अगला संवर्ग कुल है, जिसमें संबंधित वंश आते हैं।
• अगला संवर्ग गण (ऑर्डर) है, जो कुलों के समूह होते हैं तथा वर्ग (Class) में संबंधित गण आते हैं। वर्ग का अगला उच्चतर संवर्ग संघ (फाइलम) होता है। तथा विभिन्न संघों के सभी प्राणियों को उच्चतम संवर्ग जगत (किंगडम) में रखा गया है।
• इस प्रकार, वर्गिकी संवर्ग का पदानुक्रम निम्नलिखित है-
प्रजाति → वंश → कुल → गण → वर्ग → संघ → जगत
अतः उपर्युक्त विकल्पों में से विकल्प (d) सही क्रम प्रदर्शित करता है।
4. निम्नलिखित कथनों पर विचार कीजिये:
1. घरेलू मक्खी, इंसेक्टा कुल (फैमिली) के अंतर्गत आती है।
2. गेहूँ, आर्थ्रोपोडा संघ ( फाइलम) के अंतर्गत आने वाला पोएसी कुल (फैमिली) का पौधा है।
3. मानव, कार्डेटा संघ ( फाइलम) तथा डिप्टेरा गण (ऑर्डर) के अंतर्गत आता है।
उपर्युक्त कथनों में से कौन-सा/से वर्गिकी संवर्ग के आधार पर सत्य नहीं है/हैं?
(a) केवल 1 और 2
(b) केवल 2
(c) केवल 3
(d) 1, 2 और 3
उत्तर : (d)
व्याख्या: वर्गिकी संवर्ग पर आधारित कथनों में से-
• कथन (1) घरेलू मक्खी से संबंधित है जो कि इंसेक्टा वर्ग (क्लास) तथा म्यूसीडी कुल (फैमिली) के अंतर्गत आती है, इस प्रकार कथन (1) असत्य है।
• कथन (2) गेहूँ से संबंधित है जो कि एंजियोस्पर्मी संघ ( फाइलम ) तथा पोएसी कुल (फैमिली) के अंतर्गत आने वाला पौधा है। इस प्रकार, कथन (2) असत्य है।
• कथन (3) मानव से संबंधित है, जो कि कार्डेटा संघ (फाइलम) तथा प्राइमेट गण (ऑर्डर) के अंतर्गत आता है। इस प्रकार, कथन (3) असत्य है।
अतः कथन (1), (2) एवं (3) तीनों असत्य हैं और प्रश्न का विकल्प (d) सही उत्तर है।
5. निम्नलिखित में से कौन-सा कथन वनस्पति उद्यान तथा प्राणि उपवन (Zoological Garden) में अंतर स्पष्ट करता है?
(a) वनस्पति उद्यान एक्स- सीटू संरक्षण का उदाहरण है तथा प्राणि उपवन इन सीटू संरक्षण का उदाहरण प्रस्तुत करता है।
(b) वनस्पति उद्यान में पौधों के तथा चिड़ियाघर में प्राणियों के जीवित नमूने होते हैं या उन्हें परिरक्षित किया जाता है ।
(c) (a) और (b) दोनों
(d) उपर्युक्त में से कोई नहीं
उत्तर : (b)
व्याख्या: वनस्पति उद्यान तथा प्राणि उपवन के संदर्भ में-
• कथन (a) असत्य है क्योंकि वनस्पति उद्यान तथा प्राणि उपवन दोनों ही बाह्यस्थाने संरक्षण (एक्स-सीटू) के उदाहरण हैं। क्योंकि दोनों में ही मूल वातावरण से अलग स्थान पर संरक्षण किया जाता है।
• कथन (b) सत्य है क्योंकि वनस्पति उद्यान में पौधों को तथा चिड़ियाघर में प्राणियों को परिरक्षित किया जाता है।
अतः प्रश्न का विकल्प (b) सही उत्तर है।

2. जीव जगत का वर्गीकरण

1. मोनेरा तथा प्रोटिस्टा के संदर्भ में, निम्नलिखित कथनों पर विचार कीजिये :
1. मोनेरा में केंद्रक झिल्ली (Nuclear Membrane) अनुपस्थित होती है जबकि प्रोटिस्टा में उपस्थित होती है।
2. मोनेरा में यूकैरियोटिक कोशिकाएँ तथा प्रोटिस्टा में प्रोकैरियोटिक कोशिकाएँ पाई जाती हैं।
3. मोनेरा तथा प्रोटिस्टा दोनों किंगडम में कोशिका भित्ति (Cell Wall) पाई जाती है।
उपर्युक्त कथनों में से कौन-सा/से सही है/हैं?
(a) केवल 1
(b) केवल 1 और 3
(c) केवल 2
(d) 1, 2 और 3
उत्तर : (b)
व्याख्या: जीव जगत के वर्गीकरण के आधार पर मोनेरा तथा प्रोटिस्टा के लक्षणों को विभेदीकृत किया गया है जिसके अनुसार, मोनेरा में केंद्रक झिल्ली अनुपस्थित होती है तथा प्रोटिस्टा किंगडम में केंद्रक झिल्ली उपस्थित होती है, अतः कथन (1) सही है।
• मोनेरा में प्रोकैरियोटिक कोशिकाएँ तथा प्रोटिस्टा में यूकैरियोटिक कोशिकाएँ पाई जाती हैं, अतः कथन (2) गलत है।
• मोनेरा किंगडम में सेल्यूलोज रहित कोशिका भित्ति पाई जाती है तथा यह प्रोटिस्टा के कुछ वर्गों में भी उपस्थित होती है।
अतः प्रश्न का विकल्प (b) सही उत्तर है।
2. प्रोटिस्टा किंगडम के अंतर्गत किस प्रकार की पोषण विधि पाई जाती है ?
(a) केवल स्वपोषी
(b) केवल परपोषी
(c) स्वपोषी तथा परपोषी दोनों
(d) उपर्युक्त में से कोई नहीं
उत्तर : (c)
व्याख्या: सन् 1969 में आर. एच. व्हिटेकर द्वारा प्रस्तावित ‘पाँच जगत के वर्गीकरण’ के आधार पर प्रोटिस्टा किंगडम के अंतर्गत स्वपोषी (प्रकाश संश्लेषी) एवं परपोषी दोनों प्रकार की पोषण विधियाँ पाई जाती हैं।
अतः प्रश्न का विकल्प (c) सही उत्तर है।
3. निम्नलिखित में से बैक्टीरिया किस जगत के अंतर्गत आते हैं ?
1. कुछ बैक्टीरिया फंजाई तथा कुछ मोनेरा जगत के अंतर्गत आते हैं।
2. कुछ बैक्टीरिया मोनेरा तथा कुछ प्रोटिस्टा जगत के अंतर्गत आते हैं।
3. सभी बैक्टीरिया मोनेरा जगत के अंतर्गत आते हैं।
नीचे दिये गए कूट का प्रयोग कर सही उत्तर चुनिये :
(a) 1, 2 और 3
(b) केवल 1 और 2
(c) केवल 2
(d) केवल 3
उत्तर : (d)
व्याख्या : बैक्टीरिया सूक्ष्मजीवों में सर्वाधिक संख्या में होते हैं तथा लगभग सभी स्थानों पर पाए जाते हैं। जीव जगत के वर्गीकरण के आधार पर सभी बैक्टीरिया मोनेरा जगत ( Kingdom) के अंतर्गत आते हैं। अतः केवल कथन (3) सही है।
अतः प्रश्न का विकल्प (d) सही उत्तर है।
4. माइकोप्लाज़्मा के संदर्भ में, निम्नलिखित कथनों पर विचार कीजिये :
1. इसमें कोशिका भित्ति पूर्णतः अनुपस्थित होती है।
2. ये सबसे छोटी जीवित कोशिकाएँ हैं, जो ऑक्सीजन के बिना भी जीवित रह सकती हैं।
3. कई माइकोप्लाज़्मा प्राणियों और पादपों के लिये रोगजनक होता है।
4. इन्हें प्रोटिस्टा किंगडम के अंतर्गत रखा गया है।
उपर्युक्त कथनों में से कौन-से सही हैं?
(a) केवल 1, 2 और 4
(b) केवल 2, 3 और 4
(c) केवल 1 और 4
(d) केवल 1, 2 और 3
उत्तर : (d)
व्याख्या : माइकोप्लाज़्मा ऐसे जीवधारी हैं, जिनमें कोशिका भित्ति बिल्कुल नहीं पाई जाती है। अतः कथन (1) सही है।
• माइकोप्लाज़्मा सबसे छोटी जीवित कोशिकाएँ हैं, जो ऑक्सीजन के बिना भी जीवित रह सकती हैं तथा कई माइकोप्लाज़्मा प्राणियों और पादपों के लिये रोगजनक होता है। अतः कथन (2) और कथन (3) सही हैं।
• कथन (4) गलत है क्योंकि माइकोप्लाज़्मा प्रोटिस्टा जगत के अंतर्गत नहीं बल्कि मोनेरा जगत के अंतर्गत आता है।
अतः प्रश्न का विकल्प (d) सही उत्तर है।
5. निम्नलिखित में से कौन प्रोटिस्टा जगत ( किंगडम) के अंतर्गत नहीं आता है/आते हैं?
1. क्राइसोफाइट
2. यूग्लीनॉइड
3. यूबैक्टीरिया
4. प्रोटोजोआ
5. ऐस्कोमाइसिटीज
नीचे दिये गए कूट का प्रयोग कर सही उत्तर चुनिये :
(a) केवल 1, 2 और 3
(b) केवल 5
(c) केवल 3 और 5
(d) उपर्युक्त सभी
उत्तर : (c)
व्याख्या: क्राइसोफाइट, यूग्लीनॉइड तथा प्रोटोजोआ प्रोटिस्टा जगत के अंतर्गत आते हैं। अतः कथन (1), (2) और (4) गलत हैं।
• यूबैक्टीरिया मोनेरा संघ के अंतर्गत आता है। अतः कथन (3) सही है।
• ऐस्कोमाइसिटीज कवक (फंजाई) जगत के अंतर्गत आता है। अतः कथन (5) सही है।
अतः प्रश्न का विकल्प (c) सही उत्तर है।
6. निम्नलिखित समूहों में से कौन अन्य जीवों के साथ सहजीवी संबंध बना सकते हैं?
1. निडेरिया
2. कवक (फंजाई)
3. आदिजंतु (प्रोटोजोआ )
नीचे दिये गए कूट का प्रयोग कर सही उत्तर चुनिये :
(a) केवल 1 और 2
(b) केवल 2 और 3
(c) केवल 1 और 3
(d) 1, 2 और 3
उत्तर : (d)
व्याख्या: सहजीवी संबंध दो या दो से अधिक जीवों के मध्य घनिष्ठ पारिस्थितिक संबंध है।
• निडेरिया और डाईनोफ्लैगलेट शैवाल के बीच सहजीवी संबंध होता है। निडेरिया एनीमिलिया जगत के अंतर्गत आने वाला संघ है। अत; कथन 1 सही है।
• लाइकेन शैवाल तथा कवक का सहजीवी सहवास है। अतः कथन 2 सही है।
• आदिजंतु (प्रोटोजोआ ) में सहजीवन एक प्रोटोजोआ एवं एककोशिकीय जीव तथा एक प्रोटोजोआ एवं बहुकोशिकीय जीव के बीच पाया जाता है। अतः कथन 3 सही है।
अतः प्रश्न का विकल्प (d) सही उत्तर है।
7. ‘पक्ष्माभी (Ciliated) प्रोटोजोआ’ के संदर्भ में निम्नलिखित में से कौन-सा कथन सत्य है ?
(a) ये थलीय तथा धीरे-धीरे गति करने वाले जीवधारी हैं।
(b) इनमें एक गुहा (ग्रसिका ) होती है, जो कोशिका की सतह के बाहर की तरफ खुलती है।
(c) ट्रिपैनोसोमा इसका उदाहरण है।
(d) उपर्युक्त सभी कथन सत्य हैं।
उत्तर : (b)
व्याख्या: पक्ष्माभी प्रोटोजोआ (Ciliated Protozoans) जलीय तथा अत्यंत सक्रिय गति करने वाले जीवधारी हैं, अतः विकल्प (a) असत्य है ।
• ये प्रोटिस्टा जगत के अंतर्गत प्रोटोजोआ समूह में आने वाले जीव हैं, जिनमें एक गुहा होती है, जो कोशिका की सतह के बाहर की तरफ खुलती है। अतः विकल्प (b) सत्य है।
• ट्रिपैनोसोमा, कशाभी ( Flagellated) प्रोटोजोआ का उदाहरण है। अतः विकल्प (c) असत्य है।
अतः विकल्प (b) सही उत्तर है ।
8. पादप कोशिका में कोशिका भित्ति निम्नलिखित में से किस पदार्थ की बनी होती है?
(a) काइटिन
(b) म्यूकोपेप्टाइड
(c) सेल्यूलोज़
(d) म्यूरेमिक अम्ल
उत्तर : (c)
व्याख्या: पादप कोशिका में कोशिका भित्ति (Cell Wall) सेल्यूलोज़ की बनी होती है।
• विकल्प (a) गलत है क्योंकि काइटिन जो कि एक प्रकार का पॉलीसैकेराइड है, से कवकों की कोशिका भित्ति का निर्माण होता है।
• विकल्प (b) एवं (d) गलत हैं क्योंकि जीवाणुओं की कोशिका भित्ति म्यूकोपेप्टाइड तथा म्यूरेमिक अम्ल की बनी होती है ।
अतः विकल्प (c) सही उत्तर है ।
9. निम्नलिखित में से कौन-सा कथन लाइकेन के संदर्भ में सही नहीं है?
(a) लाइकेन, शैवाल तथा कवक के सहजीवी सहवास हैं।
(b) लाइकेन प्रदूषण के अच्छे संकेतक माने जाते हैं।
(c) शैवाल कवक के लिये आश्रय देता है तथा कवक शैवाल के लिये भोजन संश्लेषित करता है।
(d) उपर्युक्त में से कोई नहीं
उत्तर : (c)
व्याख्या: कथन (c) लाइकेन के संदर्भ में गलत है क्योंकि शैवाल (Algae) कवक (फंजाई) के लिये भोजन संश्लेषित करता है तथा कवक शैवाल को आश्रय देता है।
• कथन (a) एवं कथन (b) सही हैं क्योंकि लाइकेन एक ऐसा जीवन रूप है, जिसमें कवक तथा शैवाल का सहजीवी सहवास होता है एवं लाइकेन प्रदूषण के अच्छे संकेतक माने जाते हैं।
अतः विकल्प (c) सही उत्तर है।

3. वनस्पति जगत

1. शैवालों के संदर्भ में निम्नलिखित कथनों पर विचार कीजिये :
1. शैवाल केवल अलैंगिक जनन करते हैं।
2. क्लोरैला तथा स्प्रुिलाइना शैवाल में प्रोटीन प्रचुर मात्रा में पाया जाता है, जिसका उपयोग अंतरिक्ष यात्री भोजन के रूप में करते हैं।
उपर्युक्त कथनों में से कौन-सा/से सही है/हैं?
(a) केवल 1
(b) केवल 2
(c) 1 और 2 दोनों
(d) न तो 1 और न ही 2
उत्तर : (b)
व्याख्या: कथन (1) गलत है क्योंकि शैवाल कायिक (Vegetative). अलैंगिक (Asexual) तथा लैंगिक (Sexual) तीनों प्रकार से जनन करते हैं।
• कथन (2) सही है क्योंकि क्लोरैला तथा स्प्रुिलाइना एककोशिकीय शैवाल हैं, जिनमें प्रोटीन प्रचुर मात्रा में पाया जाता है तथा अंतरिक्ष यात्री कभी-कभी इसका उपयोग भोजन के रूप में भी करते हैं।
अतः विकल्प (b) सही है।
2. ‘लिवरवर्ट’ प्रायः किस प्रकार के स्थानों पर उगते हैं?
1. मरुस्थलीय क्षेत्रों में
2. चट्टानों में
3. नदियों के किनारे दलदले क्षेत्रों में
4. पेड़ों की छालों पर
नीचे दिये गए कूट का प्रयोग कर सही उत्तर चुनिये :
(a) केवल 1 और 3
(b) केवल 2
(c) केवल 2 और 3
(d) केवल 3 और 4
उत्तर : (d)
व्याख्या : लिवरवर्ट, ब्रायोफाइट के अंतर्गत आने वाले पादप हैं जो प्रायः नमी, छायादार स्थानों, जैसे- नदियों के किनारे, दलदले स्थानों, गीली मिट्टी, पेड़ों की छालों आदि पर उगते हैं। अतः कथन (1) और (2) | गलत हैं तथा कथन ( 3 ) और (4) सही हैं।
अतः विकल्प (d) सही उत्तर है।
3. निम्नलिखित कथनों पर विचार कीजिये :
कथन 1: पृथ्वी पर शैवाल कार्बन डाइऑक्साइड का स्थिरीकरण करते हैं।
कथन 2: शैवाल एक प्रकाश संश्लेषी जीव है।
उपर्युक्त कथनों के संदर्भ में निम्नलिखित में से कौन सा विकल्प सही है?
(a) कथन । और कथन 2 दोनों सही हैं और कथन 2 कथन । की सही व्याख्या करता है।
(b) कथन । और कथन 2 दोनों सही हैं, किंतु कथन 2, कथन 1 की सही व्याख्या नहीं करता है।
(c) कथन । सही है, किंतु कथन 2 गलत है।
(d) कथन | गलत है, किंतु कथन 2 सही है।
उत्तर : (a)
व्याख्या: पृथ्वी पर प्रकाश संश्लेषण के दौरान कुल स्थिरीकृत कार्बन डाइऑक्साइड का लगभग आधा भाग शैवाल स्थिर करते हैं। प्रकाश संश्लेषी जीव होने के कारण शैवाल अपने आस-पास के पर्यावरण में घुलित ऑक्सीजन का स्तर बढ़ा देते हैं।
अतः विकल्प (a) सही है।
4. प्रकृति में, निम्नलिखित में से किस जीव का/किन जीवों के मृदाविहीन सतह पर जीवित पाए जाने की सर्वाधिक संभावना है?
1. फर्न
2. लाइकेन
3. मॉस
4. छत्रक (मशरूम)
नीचे दिये गए कूट का प्रयोग कर सही उत्तर चुनिये :
(a) केवल 1 और 4
(b) केवल 2
(c) केवल 2 और 3
(d) केवल 1, 3 और 4
उत्तर : (c)
व्याख्या : लाइकेन एक जटिल जीवन रूप है, जिसमें कवक तथा शैवाल का सहजीवी संबंध होता है। लाइकेन कैसी भी सतह पर उग सकते हैं, उन्हें उगने के लिये मृदा की आवश्यकता नहीं होती। अतः कथन 2 सही है ।
• मॉस को पादप जगत के ब्रायोफाइट्स के रूप में वर्गीकृत किया गया है। ये तने एवं पत्ते वाले नॉन-फ्लॉवरिंग पौधे होते हैं, जो स्पोर पैदा करते हैं लेकिन इनमें वास्तविक जड़ें नहीं होती हैं। ये किसी भी स्थिर सतह, जैसे- नम मिट्टी, पेड़ की छाल, चट्टान आदि पर उग सकते हैं। अतः कथन 3 सही है।
अतः विकल्प (c) सही उत्तर है।
5. निम्नलिखित में से कौन-सा वर्ग टेरिडोफाइटा से संबंधित नहीं है?
(a) साइलोटम
(b) सिलैजिनेला
(c) इक्वीसीटम
(d) लिवरवर्ट
उत्तर : (d)
व्याख्या : टेरिडोफाइटा को चार वर्गों में बाँटा गया है –
(i) साइलोपसीडा (साइलोटम)
(ii) लाइकोपसीडा (सिलैजिनेला / लाइकोपोडियम)
(iii) स्फीनोपसीडा (इक्वीसीटम) तथा
(iv) टीरोपसीडा (ड्रायोप्टेरीस / एडिएंटम )
विकल्प (a), (b) तथा (c) टेरिडोफाइटा के वर्गों से संबंधित हैं, जबकि विकल्प (d) लिवरवर्ट, ब्रायोफाइटा के वर्ग से संबंधित है।
अतः विकल्प (d) सही उत्तर है ।
6. निम्नलिखित में से सर्वाधिक उपयुक्त विकल्प का चयन कीजिये :
(a) हरे शैवाल मैनोटोल लैमिनेरिन के रूप में भोजन संचित करते हैं।
(b) लाल शैवाल फ्लोरिडिऑन स्टार्च के रूप में भोजन का संचय करते हैं।
(c) भूरे शैवाल स्टार्च के रूप में भोजन का संचय करते हैं।
(d) भूरे शैवाल में कोशिका भित्ति केवल सेल्यूलोज़ की बनी होती है।
उत्तर : (b)

4. प्राणी जगत

1. प्राणी जगत के संदर्भ में निम्नलिखित कथनों पर विचार कीजिये :
1. प्लेटीहेल्मिंथीस, प्राणी जगत के अंतर्गत अगुहीय संघ है।
2. प्राणी जगत में आने वाले संघ पोरीफेरा में कोशिकीय स्तर का संगठन होता है।
3. प्राणी जगत के कॉर्डेटा संघ के प्राणियों में द्विपार्श्व सममिति (Bilateral Symmetry) पाई जाती है।
उपर्युक्त कथनों में से कौन-सा/से सत्य है/हैं?
(a) केवल 1
(b) केवल 1 और 3
(c) केवल 1 और 2
(d) 1, 2 और 3
उत्तर : (d)
व्याख्या: प्राणी जगत के वर्गीकरण में विभिन्न संघों को लक्षणों के आधार पर बाँटा गया है-
• कथन (1) सत्य है, क्योंकि प्लेटीहेल्मिंथीस संघ में गुहा (Cavity) अनुपस्थित होता है अतः यह अगुहीय (Acoelomates ) संघ है।
• कथन (2) सत्य है, क्योंकि वर्गीकरण में संगठन के स्तर पर संघों को बाँटा गया है जिसमें पोरीफेरा संघ कोशिकीय संगठन के स्तर के अंतर्गत आता है।
• कथन (3) सत्य है, क्योंकि प्राणी जगत में कुछ संघों में अरीय तथा कुछ में द्विपार्श्व सममिति पाई जाती है। इनमें से कॉर्डेटा संघ में द्विपार्श्व सममिति पाई जाती है।
अतः विकल्प (d) सही उत्तर है।
2. निम्नलिखित में से ऐसा कौन-सा संघ है, जिसमें परिसंचरण तंत्र तथा श्वसन तंत्र दोनों अनुपस्थित होते हैं?
(a) ऐनेलिडा
(b) टीनोफोरा
(c) मोलस्का
(d) आर्थ्रोपोडा
उत्तर : (b)
व्याख्या: प्राणी-जगत के विभिन्न संघों के प्रमुख लक्षणों के आधार पर ऐनेलिडा संघ में श्वसन तंत्र अनुपस्थित होता है लेकिन परिसंचरण तंत्र उपस्थित होता है। अतः विकल्प (a) गलत है।
• टीनोफोरा संघ में अपूर्ण पाचन तंत्र पाया जाता है एवं इसमें परिसंचरण तंत्र तथा श्वसन तंत्र दोनों अनुपस्थित होते हैं। अतः विकल्प (b) सही है।
• संघ मोलस्का में पूर्ण पाचन तंत्र पाया जाता है तथा परिसंचरण तंत्र एवं श्वसन तंत्र दोनों उपस्थित होते हैं। अतः विकल्प (c) गलत है।
• संघ आर्थ्रोपोडा में भी पूर्ण पाचन तंत्र पाया जाता है तथा परिसंचरण तंत्र एवं श्वसन तंत्र दोनों उपस्थित होते हैं। अत: विकल्प (d) गलत है।
अतः विकल्प (b) सही उत्तर है।
3. निम्नलिखित कथनों में से कौन-सा / से अरज्जुकी (Non-chordates ) एवं रज्जुकी (Chordates ) में अंतर स्पष्ट करता है/ करते हैं?
1. अरज्जुकी में पृष्ठरज्जु उपस्थित होता है, जबकि रज्जुकी में पृष्ठरज्जु (Notochord ) अनुपस्थित होता है।
2. अरज्जुकी तथा रज्जुकी दोनों में एकल केंद्रीय तंत्रिका तंत्र पाया जाता है।
3. रज्जुकी की ग्रसनी (Pharynx) में क्लोम छिद्र (Gill slits) पाए जाते हैं, जबकि अरज्जुकी में क्लोम छिद्र अनुपस्थित होते हैं।
4. अरज्जुकी में हृदय अधर भाग में होता है, जबकि रज्जुकी में पृष्ठ भाग में होता है।
नीचे दिये गए कूट का प्रयोग कर सही उत्तर चुनिये :
(a) केवल 2 और 3
(b) केवल 3
(c) केवल 2 और 4
(d) केवल 1, 2 और 4
उत्तर : (b)
4. निम्नलिखित कथनों पर विचार कीजिये :
1. संघ मोलस्का दूसरा सबसे बड़ा प्राणी संघ है।
2. इनका शरीर कोमल परंतु कठोर कैल्शियम की परतों से ढका रहता है।
3. इनके मुख में भोजन के लिये अंग होता है जिसे रेतीजिह्वा (रेडुला ) कहते हैं।
4. एस्टेरियस (तारामीन) इसका अच्छा उदाहरण है।
5. इसका परिवर्धन सामान्यत: लार्वा के द्वारा होता है।
संघ मोलस्का के संबंध में उपर्युक्त कथनों में से कौन-सा/से सही है/हैं?
(a) केवल 1, 2, 3 और 5
(b) केवल 1
(c) 1, 2, 3, 4 और 5
(d) केवल 1, 2, 3 और 4
उत्तर : (a)
व्याख्या: प्राणी जगत के अंतर्गत आने वाले संघ मोलस्का के प्राणियों में निम्नलिखित लक्षण पाए जाते हैं-
• आर्थ्रोपोडा संघ के बाद संघ मोलस्का दूसरा सबसे बड़ा प्राणी संघ है।
• ये द्विपार्श्व सममिति त्रिकोणकी (Tripoblastic ) तथा प्रगुही (Coelomate) प्राणी हैं।
• शरीर कोमल होता है, परंतु कठोर कैल्शियम के कवच से ढका रहता है।
• इसका शरीर अखंडित जिसमें सिर, पेशीय पाद तथा एक अंतरंग ककुद (Visceral Hump) होता है। त्वचा के ऊपर की नरम / स्पंजी परत ककुद के ऊपर प्रावार (Mantle) बनाती है।
• मुख में भोजन के लिये रेती के समान घिसने का अंग होता है, जिसे रेतीजिह्वा (रेडुला) कहते हैं।
• इसके उदाहरण – पाइला (सेबघोंघा ), सीपिया, डेन्टेलियम इत्यादि हैं।
• अतः कथन 1, 2, 3 एवं 5 सही हैं।
• इसका परिवर्धन सामान्यत: लार्वा के द्वारा होता है |
• कथन – 4 गलत है क्योंकि एस्टेरियस ( तारामीन) संघ एकाइनोडर्मेटा का उदाहरण है।
अतः विकल्प (a) सही उत्तर है।
5. निम्नलिखित में से किस जीव में जबड़े का अभाव होता है?
(a) मिक्सीन (हैग फिश)
(b) स्कॉलियोडोन ( कुत्ता मछली)
(c) समुद्री-एक्सोसिटस (उड़न मछली)
(d) राना टिग्रीना (मेंढक )
उत्तर : (a)
व्याख्या: ‘संघ कॉर्डेटा’ के अंतर्गत आने वाले उपसंघ ‘वर्टीब्रेटा’ को दो भागों (अनैथोस्टोमेटा एवं नैथोस्टोमेटा) में विभाजित किया गया है।
• अनैथोस्टोमेटा वर्ग के जीवों में जबड़ों का अभाव होता है, जबकि नैथोस्टोमेटा वर्ग के जीवों में जबड़े उपस्थित होते हैं।
• विकल्प (a) मिक्सीन ( हैग फिश) अनैथोस्टोमेटा के अंतर्गत आने वाले वर्ग साइक्लोस्टोमेटा का प्राणी है, जिसमें जबड़ा अनुपस्थित होता है।
• विकल्प (b) स्कॉलियोडोन (Dog Fish) जो कि नैथोस्टोमेटा के अंतर्गत आने वाले वर्ग कांड्रीक्थीज का प्राणी है। इसलिये, इसमें उपस्थित होता है।
• विकल्प (c) समुद्री एक्सोसिटस ( उड़न मछली), ओस्टिक्थीज वर्ग जो कि नैथोस्टोमेटा के अंतर्गत आता है, का प्राणी हैं इसमें जबड़ा पाया जाता है।
• विकल्प (d) मेंढक, एम्फीबिया वर्ग का प्राणी है तथा यह भी नैथोस्टोमेटा वर्ग के अंतर्गत आता है। अतः इसमें भी जबड़े उपस्थित होते हैं।
अतः विकल्प (a) सही उत्तर है।
6. निम्नलिखित कथनों पर विचार कीजिये :
1. शरीर सिर तथा धड़ में विभाजित होता है।
2. सभी के शरीर में पूँछ उपस्थित होती है।
3. हृदय तीन प्रकोष्ठों का बना होता है।
उपर्युक्त में से कौन-से कथन एम्फीबिया वर्ग के प्राणियों के लिये सत्य हैं?
(a) केवल 1 और 2
(b) केवल 2 और 3
(c) केवल 1 और 3
(d) केवल 1, 2 और 3
उत्तर : (c)
व्याख्या: कथन (1) और कथन (3) सत्य हैं क्योंकि एम्फीबिया (उभयचर ) वर्ग के प्राणियों में निम्नलिखित लक्षण पाए जाते हैं-
• एम्फीबिया (उभयचर) वर्ग के जीव जल तथा थल दोनों में रह सकते हैं।
• इनका शरीर सिर तथा धड़ में विभाजित होता है।
• कुछ उभयचर में पूँछ उपस्थित होती है।
• नेत्र पलक वाले होते हैं।
• ये असमतापी (Cold-blooded) प्राणी हैं।
• हृदय तीन प्रकोष्ठ का बना होता है।
कथन (2) असत्य है क्योंकि सभी उभयचरों के शरीर में नहीं बल्कि कुछ के शरीर में पूँछ उपस्थित होती है।
अतः विकल्प (c) सही उत्तर है।
7. सरीसृप वर्ग का ऐसा कौन-सा प्राणी है, जिसके हृदय में चार प्रकोष्ठ होते हैं?
(a) घरेलू छिपकली
(b) ज़हरीला सर्प
(c) मगरमच्छ
(d) एलीगेटर
उत्तर : (c)
व्याख्या : सरीसृप वर्ग के प्राणी रेंगने या सरकने वाले होते हैं। इसमें बाह्य कर्ण छिद्र नहीं पाए जाते हैं बल्कि कर्णपटल बाह्य कान का प्रतिनिधित्व करता है। इनका हृदय सामान्यतः तीन प्रकोष्ठ का होता है, अपवादस्वरूप मगरमच्छ का हृदय चार प्रकोष्ठ का होता है।
अतः विकल्प (c) सही उत्तर है।
8. निम्नलिखित कथनों पर विचार कीजिये :
1. एवीज (पक्षी) वर्ग के सभी प्राणियों में उड़ने की क्षमता है।
2. एवीज वर्ग के प्राणी विषमतापी होते हैं।
3. स्तनधारी वर्ग के प्राणियों का मुख्य लक्षण दूध उत्पन्न करने वाली ग्रोथ है, जिनसे बच्चे पोषण प्राप्त करते हैं।
4. सभी स्तनधारियों की त्वचा पर रोम पाए जाते हैं तथा सभी स्तनधारियों में आंतरिक निषेचन होता है।
उपर्युक्त कथनों में से कौन-से सही हैं ?
(a) केवल 1 और 3
(b) केवल 1, 2 और 4
(c) 1, 2, 3 और 4
(d) केवल 3 और 4
उत्तर : (d)
व्याख्या : एवीज वर्ग के प्राणियों में उड़ने की क्षमता होती है लेकिन कुछ प्राणी जैसे- शुतुरमुर्ग जैसे पक्षी उड़ नहीं सकते, अतः कथन (1) गलत है।
• एवीज वर्ग के प्राणियों में चोंच पाई जाती है। इनका हृदय पूर्णत: चार प्रकोष्ठ का बना होता है तथा ये समतापी (Warm-blooded) होते हैं। अत: कथन (2) गलत है।
• कथन (3) व (4) स्तनधारी वर्ग के प्राणियों के लक्षणों के संबंध में सही हैं।
अतः विकल्प (d) सही उत्तर है।

5. पुष्पी पादपों की आकारिकी

1. निम्नलिखित कथनों पर विचार कीजिये :
1. पादपों में मूल (Root) का रूपांतरण केवल पानी तथा खनिज लवण के अवशोषण तथा संवहन के लिये होता है।
2. कुछ पौधों के तनों की कक्षीय कलियाँ काष्ठीय, सीधे तथा नुकीले काँटों में रूपांतरित हो सकती हैं।
3. पत्तियाँ पौधों के लिये बहुत महत्त्वपूर्ण कायिक अंग (Vegetative Organs) हैं, क्योंकि ये भोजन का निर्माण करती हैं।
उपर्युक्त कथनों में से कौन-सा/से सत्य नहीं है/हैं?
(a) केवल 2
(b) केवल 1
(c) केवल 3
(d) 1, 2 और 3
उत्तर : (b)
व्याख्या: कथन (1) गलत है क्योंकि कुछ पादपों में मूल का रूपांतरण होता है। ये केवल पानी तथा खनिज लवणों के अवशोषण एवं संवहन के लिये नहीं बल्कि कुछ अतिरिक्त कार्यों को करने के लिये अपने आकार तथा संरचना में रूपांतरण कर लेते हैं।
• कथन (2) सत्य है क्योंकि कुछ पौधों में तने विभिन्न कार्यों को संपन्न करने के लिये अपने आपको रूपांतरित कर लेते हैं। जैसे- कुछ सब्ज़ियों तथा अंगूर लता के तनों की कक्षीय कलियाँ काष्ठीय, सीधे तथा नुकीले काँटों में रूपांतरित हो सकती हैं।
• कथन (3) पत्ती के संदर्भ में है। पत्ती पाश्र्वय, चपटी संरचना होती है जो तने पर लगी रहती है। यह गाँठ पर होती है और इसके कक्ष में कली होती है। पत्तियाँ प्ररोह (Shoot) के शीर्षस्थ मेरिस्टेम से निकलती हैं तथा ये पौधों के बहुत ही महत्त्वपूर्ण कायिक अंग हैं, क्योंकि ये भोजन का निर्माण करती हैं।
अतः विकल्प (b) सही उत्तर है।
2. निम्नलिखित युग्मों पर विचार कीजिये :
1. गेहूँ का पौधा : मूसला मूलतंत्र (Tap Root System)
2. सरसों का पौधा : झकड़ा मूलतंत्र (Fibrous Root System) ?
3. घास तथा बरगद : अपस्थानिक मूल (Adventitious Roots)
उपर्युक्त युग्मों में से कौन-सा/से सुमेलित है/हैं?
(a) केवल 1 और 3
(b) केवल 2 और 3
(c) केवल 3
(d) 1, 2 और 3
उत्तर : (c)
व्याख्या: केवल (3) सुमेलित है।
• अधिकांश द्विबीजपत्री पादपों में मूलांकुर के लंबे होने से प्राथमिक मूल बनती है, जो मिट्टी में उगती है। इसमें पाश्र्वय मूल होती है जिन्हें द्वितीयक तथा तृतीयक मूल कहते हैं। प्राथमिक मूल तथा इसकी शाखाएँ मिलकर मूसला मूलतंत्र बनाती हैं। इसका उदाहरण सरसों का पौधा है।
• एक बीजपत्री में प्राथमिक मूल अल्पायु होती है और इसके स्थान पर अनेक मूल निकल जाती हैं। ये मूल तने के आधार से निकलती हैं। इन्हें ‘झकड़ा मूलतंत्र’ कहते हैं। गेहूँ का पौधा इसका उदाहरण है।
• कुछ पौधों जैसे- घास तथा बरगद में मूल, मूलांकुर की बजाय पौधे के अन्य भाग से निकलती है। इन्हें अपस्थानिक मूल (Adventitious Roots) कहते हैं।
अतः विकल्प (c) सही उत्तर है।
3. पुष्पों के संदर्भ में निम्नलिखित कथनों पर विचार कीजिये :
1. प्रत्येक पुष्प में चार चक्र होते हैं।
2. जायांग (Gynoecium) पुष्पों का लैंगिक अंग है।
3. केलिक्स (Calyx) पुष्प का सबसे भीतरी चक्र है।
उपर्युक्त कथनों में से कौन-से सत्य हैं?
(a) केवल 1 और 3
(b) केवल 2 और 3
(c) 1, 2 और 3
(d) केवल 1 और 2
उत्तर : (d)
व्याख्या: कथन (1) सत्य है क्योंकि प्रत्येक पुष्प में चार चक्र होते हैं जैसे केलिक्स (Calyx), कोरोला (Corolla), पुमंग (Androecium) तथा | जायांग (Gynoecium) ।
• कथन (2) सत्य है क्योंकि पुष्पों में केलिक्स तथा कोरोला सहायक अंग हैं जबकि पुमंग तथा जायांग लैंगिक अंग हैं।
• कथन (3) असत्य है क्योंकि पुष्पों के चारों चक्र में से केलिक्स सबसे बाहरी चक्र होता है और इसकी इकाई को ‘बाह्य दल’ कहते हैं। अतः विकल्प (d) सही उत्तर है।
4. फल के संदर्भ में निम्नलिखित कथनों पर विचार कीजिये :
1. प्रायः फल में एकभित्ति अथवा फलभित्ति तथा बीज होते हैं।
2. फलभित्ति शुष्क अथवा गूदेदार हो सकती है।
3. आम तथा नारियल में फल के प्रकार को अष्ठिल (डूप) कहते हैं।
उपर्युक्त कथनों में से कौन सा/से सत्य है/हैं?
(a) केवल 1
(b) केवल 1 और 3
(c) केवल 2 और 3
(d) 1, 2 और 3
उत्तर : (d)
व्याख्या : फल (Fruit) पुष्पी पादपों का एक प्रमुख अभिलक्षण है। यह एक परिपक्व अंडाशय होता है जो निषेचन के बाद विकसित होता है।
• प्रायः फल में एकभित्ति अथवा फलभित्ति तथा बीज होते हैं। फलभित्ति शुष्क अथवा गूदेदार हो सकती है।
• आम तथा नारियल में फल के प्रकार को अष्ठिल (ड्रूप) कहते हैं। ये फल एकांडपी (Monocarpellary) ऊर्ध्ववर्ती अंडाशय से विकसित होते हैं और इनमें एक बीज होता है।
अतः विकल्प (d) सही उत्तर है ।
5. मरुस्थलीय क्षेत्रों में जल ह्रास को रोकने के लिये निम्नलिखित में से कौन-सा/से पूर्ण रूपांतरण होता है/होते हैं ?
1. कठोर एवं मोमी पर्ण
2. लघु पूर्ण
3. पर्ण की जगह काँटे
नीचे दिये गए कूट का प्रयोग कर सही उत्तर चुनिये-
(a) केवल 2 और 3
(b) केवल 2
(c) केवल 3
(d) 1, 2 और 3
उत्तर : (d)
व्याख्या : मरुस्थलीय भागों में वनस्पतियों का प्रायः अभाव पाया जाता है और जो वनस्पतियाँ होती भी हैं, वे झाड़ियों के रूप में अथवा काँटेदार होती हैं। उदाहरण के लिये नागफनी, कैक्टस, बबूल, खजूर इत्यादि ।
• मरुस्थलीय पौधे वाष्पोत्सर्जन को कम करने के लिये अपनी पत्तियों का आकार या तो बहुत छोटा कर लेते हैं या उन्हें काँटों में रूपांतरित कर लेते हैं।
• चूँकि मरुस्थलीय क्षेत्रों में पानी की नियमित उपलब्धता संभव नहीं हो पाती है और पानी का ज़मीनी स्तर भी काफी नीचे होता है। अतः इन प्रतिकूल परिस्थितियों में अपने आपको अनुकूलित करने के लिये पत्तियों और तनों में पानी भंडारण की क्षमता होती है।
• इसलिये पत्तियाँ और तने मोमी व फूले हुए होते हैं। इसके अलावा, जल ह्रास को रोकने के लिये तने थोड़े कठोर होते हैं।
अतः विकल्प (d) सही उत्तर है।
6. निम्नलिखित युग्मों पर विचार कीजिये :
1. शलजम : मूल का रूपांतरण
2. आलू : तने का रूपांतरण
3. गाजर : तने का रूपांतरण
4. प्याज : पत्ती का रूपांतरण
उपर्युक्त युग्मों में से कौन-सा/से सही है/हैं?
(a) केवल 2
(b) केवल 1, 3 एवं 4
(c) केवल 1, 2 एवं 4
(d) 1, 2, 3 एवं 4
उत्तर : (c)
व्याख्या: पौधों में रूपांतरण तीन तरह से होता है, कुछ पौधों में मूल (जड़) का रूपांतरण, कुछ में तने का रूपांतरण तथा रूपांतरण होता है। जैसे कुछ में पत्ती का –
• शलजम तथा गाजर में मूल का रूपांतरण होता है।
• आलू, अदरक, अरबी आदि में तने का रूपांतरण होता है।
• प्याज में पत्ती का रूपांतरण होता है।
अत: दिये गए विकल्पों में से विकल्प (c) सही उत्तर क्योंकि गाजर में मूल का रूपांतरण होता है, जबकि युग्म में उसे तने के रूपांतरण के साथ युग्मित किया गया है, जो गलत है।
7. निम्नलिखित में से किस पौधे में तना खाद्य सामग्री का भंडारण नहीं करता है?
(a) आलू
(b) अदरक
(c) प्याज
(d) अरबी
उत्तर : (c)
व्याख्या: पौधों में विभिन्न भाग जैसे कि जड़, प्ररोह ( Shoot), पत्ती, फूल, फल आदि खाद्य पदार्थ का भंडारण करते हैं।
• प्याज के मांसल पत्ते खाद्य पदार्थों का संग्रहण करते हैं।
• उपर्युक्त विकल्पों में से विकल्प (c) प्याज के अलावा विकल्प (a), (b) एवं (d) के तनों में खाद्य पदार्थों का संग्रहण होता है।
अतः विकल्प (c) सही उत्तर है।
8. एकबीजपत्री बीज की संरचना के संदर्भ में, निम्नलिखित में से कौन – सा बाह्यतम से आंतरिक भाग के क्रम में है?
(a) भ्रूणपोष → भ्रूण → एल्यूरोन सतह → बीजावरण
(b) एल्यूरोन सतह → भ्रूण → बीजावरण → भ्रूणपोष
(c) बीजावरण → भ्रूण → भ्रूणपोष → एल्यूरोन सतह
(d) बीजावरण → एल्यूरोन सतह → भ्रूणपोष → भ्रूण
उत्तर : (d)
व्याख्या: प्राय: एकबीजपत्री बीज भ्रूणपोषी होते हैं, किसी भी बीज में बाहरी आवरण बीजावरण (Fruit-wall) कहलाता है, फिर एल्यूरोन सतह तथा भ्रूणपोष (Endosperm ) तथा अंतरतम भाग भ्रूण (Embryo) होता है। भ्रूण, भ्रूणपोष के एक सिरे में खाँचे में स्थित होता है।
अतः विकल्प (d) सही उत्तर है।
9. आम व नारियल के संदर्भ में, निम्नलिखित को आंतरिक से बाह्यतम क्रमानुसार लिखने पर सर्वाधिक उपयुक्त विकल्प कौन-सा है?
(a) बीज → अंतः फलभित्ति → मध्य फलभित्ति → बाह्य फलभित्ति
(b) अतः फलभित्ति → बीज → मध्य फलभित्ति → बाह्य फलभित्ति
(c) बाह्य फलभित्ति → मध्य फलभित्ति → बीज → अंतः फलभित्ति
(d) मध्य फलभित्ति → अंतः फलभित्ति → बीज → बाह्य फलभित्ति
उत्तर : (a)
व्याख्या: आम तथा नारियल में फल के प्रकार को अष्ठिल (डूप) कहते हैं। ये फल एकांडपी ऊर्ध्ववर्ती अंडाशय से विकसित होते हैं और इनमें एक बीज (Seed) होता है।
• इनमें सबसे आंतरिक भाग में बीज होता है, उसके बाद अंतः फलभित्ति (Endocarp) तथा मध्य फलभित्ति (Mesocarp) होती है तथा इनमें सबसे बाह्तम बाह्य फलभित्ति (Epicarp) होती है। नारियल में मध्य फलभित्ति तंतुमयी होती है।
अतः विकल्प (a) सही उत्तर है।

6. पुष्पी पादपों का शरीर

1. पौधे में ऊतक के संदर्भ में निम्नलिखित कथनों पर विचार कीजिये :
1. पैरेंकाइमा एक जटिल ऊतक है |
2. पैरेंकाइमा की भित्ति सेल्यूलोज़ की बनी होती है।
3. स्क्लेरेंकाइमा में तंतु (Fibres ) मोटी भित्ति वाले, लंबे तथा नुकीली कोशिकाओं के होते हैं।
उपर्युक्त कथनों में से कौन-से सही हैं?
(a) केवल 1 और 3
(b) केवल 2 और 3
(c) केवल 1 और 2
(d) 1, 2 और 3
उत्तर : (b)
व्याख्या: पौधों में ऊतकों के वर्गीकरण का आधार कोशिकाओं का विभक्त होना अथवा न होना है। ऊतक को प्रमुख रूप से दो भागों मेरिस्टमी ऊतक एवं स्थायी ऊतक में बाँटा गया है।
• पैरेंकाइमा एक सरल ऊतक का उदाहरण है। पैरेंकाइमा की भित्ति पतली होती है तथा सेल्यूलोज़ की बनी होती है, पैरेंकाइमा बहुत से कार्य, जैसे- प्रकाश संश्लेषण, संचय, स्राव आदि संपन्न करते हैं।
• स्क्लेरेंकाइमा में लंबी, सँकरी कोशिकाएँ होती हैं। इनमें तंतु मोटी भित्ति वाले, लंबे तथा नुकीली कोशिकाओं के होते हैं। ये प्रायः पौधों के विभिन्न भागों में समूह के रूप में पाए जाते हैं।
अतः कथन (1) गलत है तथा कथन (2) एवं (3) सही हैं।
अतः विकल्प (b) सही उत्तर है।
2. निम्नलिखित कथनों पर विचार कीजिये :
1. जाइलम मूल से पानी तथा खनिज लवण को तने तथा पत्तियों तक पहुँचाने के लिये एक संवहन ऊतक की तरह कार्य करता है तथा यह एक सरल ऊतक है।
2. फ्लोएम प्राय: भोजन को पत्तियों से पौधे के अन्य भागों में पहुँचाते हैं। तथा फ्लोएम तंतु (Phloem Fibres) स्कलेरेंकाइमी कोशिकाओं के बने होते हैं।
उपर्युक्त कथनों में से कौन – सा /से सही है/हैं?
(a) केवल 1
(b) केवल 2
(c) 1 और 2 दोनों
(d) न तो 1, न ही 2
उत्तर : (b)
व्याख्या: कथन (1) गलत है क्योंकि जाइलम का कार्य मूल (जड़) से पानी तथा खनिज लवण को तने तथा पत्तियों तक पहुँचाना है लेकिन यह एक सरल ऊतक नहीं है, बल्कि यह एक जटिल ऊतक है।
• कथन (2) सही है क्योंकि फ्लोएम का कार्य प्रायः भोजन को पत्तियों से पौधे के अन्य भागों तक पहुँचाना है तथा फ्लोएम तंतु (Phloem Fibres) स्क्लेरेंकाइमी कोशिकाओं के बने होते हैं। .
अतः विकल्प (b) सही उत्तर है।
3. ‘क्यूटिकल’ क्या है?
(a) दो सेम के आकार की कोशिकाओं के बीच का रंध्र
(b) पत्तियों में पतली भित्ति वाला तथा क्लोरोप्लास्ट युक्त भरण ऊतक
(c) बाह्यत्वचा की बाहरी सतह जो मोम की मोटी परत से ढकी होती है
(d) पैरेंकाइमा कोशिकाएँ जो जाइलम तथा फ्लोएम बंडल के बीच में हैं
उत्तर : (c)
व्याख्या : विकल्प (c) सही है क्योंकि बाह्य त्वचीय (Epidermal) ऊतक तंत्र के अंतर्गत बाह्यत्वचा की बाहरी सतह मोम की मोटी परत से ढकी होती है, जिसे ‘क्यूटिकल’ कहते हैं। क्यूटिकल पानी की हानि को रोकती है एवं पौधे के मूल (जड़) में क्यूटिकल नहीं होती है।
• विकल्प (a) गलत है क्योंकि प्रत्येक रंध्र में दो सेम के आकार की कोशिकाओं को द्वार कोशिकाएँ (Guard Cells) कहते हैं।
• विकल्प (b) गलत है क्योंकि पत्तियों में पतली भित्ति वाला तथा क्लोरोप्लास्ट युक्त भरण ऊतक (Ground Tissue) को पर्णमध्योतक (जोफिल) कहते हैं।
• विकल्प (d) गलत है क्योंकि पैरेंकाइमा कोशिकाएँ जो जाइलम तथा फ्लोएम बंडल के बीच में हैं उन्हें कंजक्टिव ऊतक (Conjuctive Tissue) कहते हैं।
अतः विकल्प (c) सही उत्तर है।
4. निम्नलिखित में से ‘कैंबियम’ संबंधित होता है-
(a) संवहनी ऊतक तंत्र से (The Vascular Tissue System)
(b) एक बीजपत्री मूल से (Mono Cotyledonous Root)
(c) भरण ऊतक तंत्र से (The Ground Tissue System)
(d) बाह्य त्वचीय ऊतक तंत्र (Epidermal Tissue System)
उत्तर : (a)
व्याख्या : संवहनी ऊतक तंत्र में जटिल ऊतक, जाइलम तथा फ्लोएम होते हैं। जाइलम तथा फ्लोएम दोनों मिलकर संवहन बंडल (Vascular Bundles) बनाते हैं। द्विबीजपत्री में जाइलम तथा फ्लोएम के बीच ‘कैंबियम’ होता है।
अतः विकल्प (a) सही उत्तर है।
5. ‘स्प्रिंग वुड (Spring Wood)’ तथा ‘ऑटम वुड (Autumn Wood)’ के संबंध में निम्नलिखित कथनों पर विचार कीजिये :
1. बसंत (स्प्रिंग) के मौसम में कैंबियम शरद (ऑटम) की अपेक्षा अधिक सक्रिय रहता है। जिससे अधिक संख्या में वाहिकाएँ बनती हैं।
2. चौड़ी गुहिकाओं (Cavities) वाली काष्ठ को स्प्रिंग वुड तथा सँकरी गुहिकाओं वाली काष्ठ को ऑटम वुड कहा जाता है।
3. स्प्रिंग वुड का रंग गहरा तथा घनत्व कम होता है जबकि ऑटम वुड का रंग हल्का तथा घनत्व अधिक होता है।
उपर्युक्त कथनों में से कौन सा/से सत्य नहीं है/हैं?
(a) केवल 1 और 3
(b) केवल 2 और 3
(c) केवल 2
(d) केवल 3
उत्तर : (d)
व्याख्या: कैंबियम की क्रिया शारीरिक क्रिया तथा पर्यावरणीय कारकों से नियंत्रित होती है । उष्णकटिबंधीय क्षेत्रों में, जलवायु समान नहीं रहती तथा बसंत के मौसम में कैंबियम बहुत सक्रिय होता है और अधिक संख्या में वाहिकाएँ बनाता है। जिसकी गुहिकाएँ चौड़ी होती हैं। बसंत के मौसम में | बनने वाली इस काष्ठ को ‘स्प्रिंग वुड’ कहते हैं।
• सर्दियों में कैंबियम कुछ कम सक्रिय होता है और सँकरी वाहिकाएँ बनाता है, इस काष्ठ को ‘ऑटम वुड’ कहते हैं।
• इस प्रकार कथन (1) और (2) सत्य हैं।
• कथन (3) गलत है क्योंकि ‘स्प्रिंग वुड’ का रंग हल्का होता है तथा घनत्व भी कम होता है एवं ‘ऑटम वुड’ का रंग गहरा होता है तथा घनत्व भी अधिक होता है।
अतः विकल्प (d) सही उत्तर है ।
6. ‘हर्टवुड’ एवं ‘सैपवुड’ के संबंध में निम्नलिखित में से सर्वाधिक उपयुक्त कथन है-
(a) हर्टवुड पानी का संवहन करता है, यह तने को यांत्रिक सहारा देता है, जबकि सैपवुड केवल पानी तथा खनिज लवणों को मूल से पत्तियों तक पहुँचाने से संबद्ध होता है।
(b) सैपवुड पानी का संवहन नहीं करता, यह केवल तने को यांत्रिक सहारा देता है, जबकि हर्टवुड पानी तथा खनिज लवणों को मूल से पत्तियों तक पहुँचाने से संबद्ध होता है।
(c) हर्टवुट पानी का संवहन नहीं करता है, यह केवल तने को यांत्रिक सहारा देता है जबकि सैपवुड पानी तथा खनिज लवणों को मूल से पत्तियों तक पहुँचाने से संबद्ध होता है ।
(d) हर्टवुड तथा सैपवुड दोनों पानी तथा खनिज लवणों का संवहन करते हैं।
उत्तर : (c)
व्याख्या : ‘हर्टवुड’ में बहुत से कार्बनिक यौगिक, जैसे- टेनिन, रेजिन, तेल, गोंद, खुशबूदार पदार्थ तथा आवश्यक तेल होते हैं जो इसे सूक्ष्म जीवों तथा कीड़ों से बचाते हैं। यह पानी का संवहन नहीं करता बल्कि यह केवल तने को यांत्रिक सहारा देता है।
• द्वितीयक जाइलम की परिधि क्षेत्र को ‘सैपवुड’ कहते हैं, जो हल्के रंग का होता है, जिसमें सजीव पैरेंकाइमा कोशिकाएँ होती हैं। यह मूल से पानी तथा खनिज लवणों को पत्तियों तक पहुँचाने से संबद्ध है।
अतः विकल्प (c) सर्वाधिक उपयुक्त उत्तर है।
7. निम्नलिखित कथनों पर विचार कीजिये :
कथन 1 : कॉर्क कैंबियम के अंतर्गत कॉर्क कोशिकाओं में पानी प्रवेश नहीं कर सकता है।
कथन 2: कॉर्क कोशिकाओं की कोशिका भित्ति पर सुबेरिन जमा रहता है।
उपर्युक्त कथनों के संदर्भ में निम्नलिखित में से कौन – सा सही है ?
(a) कथन 1 और 2 दोनों सही हैं और कथन 2, कथन 1 की सही व्याख्या करता है।
(b) कथन 1 और कथन 2 दोनों सही हैं, किंतु कथन 2, कथन 1 की सही व्याख्या नहीं है।
(c) कथन 1 सही है, किंतु कथन 2 गलत है।
(d) कथन 1 गलत है, किंतु कथन 2 सही है।
उत्तर : (a)
व्याख्या: कॉर्क कैंबियम कुछ सतही मोटी और संकरी पतली भित्ति वाली आयताकार कोशिकाओं की बनी होती है। यह दोनों ओर कोशिकाओं को बाँटता है, बाहर की ओर कोशिकाएँ कॉर्क में बँट जाती हैं। तथा कॉर्क में पानी प्रवेश नहीं कर सकता क्योंकि इसकी कोशिका भित्ति पर सुबेरिन जमा रहता है।
अतः विकल्प (a) सही उत्तर है ।

7. प्राणियों में संरचनात्मक संगठन

1. उपकला ऊतक (Epithelial Tissue) के संदर्भ निम्नलिखित कथनों पर विचार कीजिये :
1. शल्की उपकला ऊतक चपटी कोशिकाओं के पतले स्तर से बनता है जिसके किनारे अनियमित होते हैं।
2. घनाकार उपकला ऊतक का प्रमुख कार्य स्रवण और अवशोषण है।
3. पक्ष्माभी (Ciliated) उपकला ऊतक मुख्यतः श्वसनिका तथा डिंबवाहिनी नलिकाओं (Fallopian tubes) जैसे खोखले अंगों की भीतरी सतह पर पाए जाते हैं।
उपर्युक्त कथनों में से कौन-से सही हैं?
(a) 1 और 2
(b) 1 और 3
(c) 2 और 3
(d) 1, 2 और 3
उत्तर : (d)
2. निम्नलिखित में से कौन-सा ऊतक किसी भी रासायनिक या यांत्रिक प्रतिवलों (Stresses) से शरीर की सुरक्षा का कार्य मुख्य रूप से करता है?
(a) संयोजी ऊतक
(b) संयुक्त उपकला ऊतक
(c) तंत्रिका ऊतक
(d) सरल उपकला ऊतक
उत्तर : (b)
व्याख्या : संयुक्त उपकला एक से ज़्यादा कोशिका स्तरों की बनी होती है। स्रवण एवं अवशोषण जैसे कार्यों में इसकी भूमिका सीमित है। इसका मुख्य कार्य रासायनिक व यांत्रिक प्रतिबलों (Stresses) से शरीर की रक्षा करना है।
• यह त्वचा की शुष्क सतह, मुखगुहा की नम सतह, ग्रसनी, लार ग्रंथियों और अग्न्याशयी वाहिनियों के भीतरी आस्तर को ढकता है।
अतः विकल्प (b) सही उत्तर है।
3. निम्नलिखित कथनों पर विचार कीजिये :
1. अस्थि खनिज युक्त ठोस संयोजी ऊतक है।
2. समस्त पेशी तंतु के समन्वित रूप से संकुचित होकर पुन: अपनी असंकुचित अवस्था में आना उद्दीपन के कारण है।
3. चिकनी पेशीय ऊतक का केवल एक किनारा पतला होता है तथा इनमें रेखा या धारियाँ नहीं होती हैं।
उपर्युक्त कथनों में से कौन-सा/से सही है/हैं?
(a) केवल 1
(b) केवल 1 और 3
(c) केवल 1 और 2
(d) केवल 3
उत्तर : (c)
व्याख्या : अस्थि खनिज युक्त ठोस संयोजी ऊतक है। इसका आनम्य आधात्री (Ground Substance) कॉलेजन तंतु एवं कैल्शियम लवण युक्त होता है, जो अस्थि को मज़बूती प्रदान करता है। अतः कथन 1 सही है।
• चिकनी पेशीय ऊतक की संकुचनशील कोशिका के दोनों किनारे पतले होते हैं तथा इनमें रेखा या धारियाँ नहीं होती हैं। कोशिका संधियाँ उन्हें एक साथ बांधे रखती हैं तथा ये संयोजी ऊतक के आवरण से ढके समूह रहते हैं। अतः कथन 3 गलत है।
• पेशी ऊतक अनेक लंबे, बेलनाकार तंतुओं (रेशों) से बना होता है, जो समानांतर पंक्ति में सजे रहते हैं। यह तंतु कई सूक्ष्म तंतुकों से बना है, जिसे पेशी तंतुक (Myofibril) कहते हैं। समस्त पेशी तंतु समन्वित रूप से उद्दीपन के कारण संकुचित हो जाते हैं तथा पुनः लंबे होकर अपनी असंकुचित अवस्था में आ जाते हैं। अत: कथन 2 सही है।
अतः विकल्प (c) सही उत्तर है।
4. ‘ केंचुआ’ के संबंध में निम्नलिखित कथनों पर विचार कीजिये :
1. इनका शरीर लंबा और लगभग 100-120 समान समखंडों (Metameres) में बँटा होता है।
2. फेरेटिमा व ल्यूम्ब्रिकस (Pheretima and Lumbricus) सामान्य भारतीय केंचुए हैं।
3. इनके उत्सर्जी अंग, खंडों में व्यवस्थित और वलयित नलिकाओं के बने होते हैं, जिन्हें नेफ्रीडिया (Nephridia) कहते हैं।
उपर्युक्त कथनों में से कौन-से सही हैं?
(a) केवल 1 और 2
(b) केवल 2 और 3
(c) 1, 2 और 3 तीनों
(d) केवल 1 और 3
उत्तर : (c)
व्याख्या: केंचुए की आकारिकी के अध्ययन में हम पाते हैं कि-
• इनका शरीर लंबा और लगभग 100-120 समान समखंडों में बँटा होता है।
• पृष्ठ तल पर एक गहरी मध्यरेखा दिखाई देती है।
• शरीर के अग्र भाग में मुख एवं पुरोमुख (Prossomium) होते हैं।
• उत्सर्जी अंग, खंडों में व्यवस्थित और नलिकाओं के बने होते हैं, जिन्हें वृक्कक (Nephridial) कहते हैं।
• वृक्कक शरीर तरल के आयतन एवं संगठन का नियमन करते हैं।
• फेरेटिमा व ल्यूम्ब्रिकस सामान्य भारतीय केंचुए हैं।
• अतः कथन 1, 2 और 3 तीनों सही हैं।
अतः विकल्प (c) सही उत्तर है।
5. ‘कृमि क्षिप्ति’ (Worm Casting) का संबंध किससे है ?
(a) केंचुए के उत्सर्जी मल पदार्थ से
(b) केंचुए के मुख को ढकने वाली संरचना से
(c) तिलचट्टा के बाह्य कंकाल की पट्टिकाओं से
(d) मेंढक के रक्षात्मक रंग परिवर्तन से
उत्तर : (a)
व्याख्या: केंचुए लाल-भूरे रंग के स्थलीय प्राणी होते हैं, जो कि नम मिट्टी की ऊपरी सतह में निवास करते हैं। दिन के समय ये जमीन के अंदर स्थित बिलों में रहते हैं, जो कि ये मिट्टी को छेदकर और निगलकर बनाते हैं। बगीचों में ये अपने द्वारा एकत्रित उत्सर्जी मल पदार्थों के बीच ढूँढ़े जा सकते हैं। इन उत्सर्जी मल पदार्थों को कृमि क्षिप्ती (Worm Casting) कहते हैं।
• केंचुए के मुख को ढकने वाली संरचना → पुरोमुख (Prossomium) कहलाती है।
• तिलचट्टा के बाह्य कंकाल की पट्टिकाएँ कहलाती हैं। → कठक (Sclerises)
• मेंढक के रक्षात्मक रंग परिवर्तन को → अनुहरण (Minimicry) कहते हैं।
अतः विकल्प (a) सही उत्तर है।
6. निम्नलिखित में से किसे किसान का मित्र कहा जाता है?
(a) चींटी
(b) मेंढक
(c) केंचुआ
(d) कॉकरोच
उत्तर : (c)
व्याख्या : केंचुआ किसानों का मित्र कहलाता है। यह मिट्टी में छोटे-छोटे बिल बनाता है, जिससे मिट्टी छिद्रित हो जाती है और बढ़ते पौधों की जड़ों के लिये वायु की उपलब्धता और उनका नीचे की ओर बढ़ना सुगम हो जाता है।
• इस प्रकार केंचुआ द्वारा मिट्टी को उपजाऊ बनाने की विधि या मिट्टी की उर्वर शक्ति बढ़ाने की विधि को कृमि कंपोस्ट खाद निर्माण कहते हैं। केंचुए मछली पकड़ने के लिये प्रलोभक के रूप में प्रयोग में भी लिये जाते हैं।
अतः विकल्प (c) सही उत्तर है।
7. मेंढक के संदर्भ में सही कथन चुनिये :
(a) मेंढक अकशेरूकी संघ के एफीबिया वर्ग से संबंधित है।
(b) इसके शरीर का ताप स्थिर होता है।
(c) मेंढक का शरीर सिर व धड़ में विभाजित रहता है। पूँछ व गर्दन का अभाव होता है।
(d) इसके मुख के ऊपर दो जोड़े नासिका द्वार खुलते हैं।
उत्तर : (c)
व्याख्या : मेंढक वह प्राणी है जो मीठे जल तथा धरती दोनों पर निवास करता है तथा कशेरूकी संघ के एफीबिया वर्ग से संबंधित होता है। भारत में पाई जाने वाली मेंढक की सामान्य जाति ‘राना टिग्रीना’ है।
• इसके शरीर का ताप स्थिर नहीं होता है, शरीर का ताप वातावरण ताप के अनुसार परिवर्तित होता रहता है।
• मेंढक का शरीर सिर व धड़ में विभाजित रहता है, पूँछ व गर्दन का अभाव होता है । मुख के ऊपर एक जोड़े नासिका द्वार खुलते हैं।
अतः विकल्प (c) सही उत्तर है ।

8. कोशिका : जीवन की इकाई

1. ‘कोशिका सिद्धांत’ के संदर्भ में निम्नलिखित कथनों पर कीजिये :
1. सभी जीव कोशिका व कोशिका उत्पाद से बने होते हैं।
2. सभी कोशिकाएँ पूर्व स्थित कोशिकाओं से निर्मित होती हैं।
3. कोशिका के इन वर्तमान सिद्धांतों को रुडोल्फ विचों ने स्पष्ट किया।
उपर्युक्त कथनों में से कौन-सा/से सही है/हैं?
(a) केवल 3
(b) केवल 2 और 3
(c) केवल 1 और 2
(d) 1, 2 और 3
उत्तर : (d)
व्याख्या : स्लाइडेन व श्वान ने संयुक्त रूप से कोशिका सिद्धांत को प्रतिपादित किया । यद्यपि इनका सिद्धांत यह बताने में असफल रहा कि नई कोशिकाओं का निर्माण कैसे होता है।
• पहली बार ‘रुडोल्फ विर्ची’ ( 1855) ने स्पष्ट किया कि कोशिका विभाजित होती है और नई कोशिकाओं का निर्माण पूर्व स्थित कोशिकाओं के विभाजन से होता है। इन्होंने ‘स्लाइडेन’ व ‘श्वान’ की कल्पना को रूपांतरिक कर नए कोशिका सिद्धांत को प्रतिपादित किया। वर्तमान समय के परिप्रेक्ष्य में कोशिका सिद्धांत निम्नवत हैं-
♦ सभी जीव कोशिका व कोशिका उत्पाद से बने होते हैं।
♦ सभी कोशिकाएँ पूर्व स्थित कोशिकाओं से निर्मित होती हैं।
अतः विकल्प (d) सही उत्तर है।
2. निम्नलिखित कथनों में से कौन-से पादप और प्राणी कोशिकाओं के बीच सामान्य अंतर के बारे में सही है ?
1. पादप कोशिकाओं में सेल्यूलोज की बनी कोशिका भित्तियाँ होती हैं। जबकि प्राणी कोशिकाओं में वे नहीं होती ।
2. पादप कोशिकाओं में प्लाज्मा झिल्ली नहीं होती, जबकि इसके विपरीत प्राणी कोशिकाओं में वे होती हैं।
3. परिपक्व पादप कोशिका में एक वृहद् रसधानी होती है, जबकि प्राणी कोशिका में अनेक छोटी रसधानियाँ होती हैं।
नीचे दिये गए कूट का प्रयोग कर सही उत्तर चुनिये-
(a) केवल 1 और 2
(b) केवल 2 और 3
(c) केवल 1 और 3
(d) 1, 2 और 3
उत्तर : (d)
व्याख्या: जंतु/प्राणी कोशिका एवं पादप कोशिका के बीच अंतर :
• जंतु कोशिकाएँ प्राय: छोटी होती हैं, जबकि पादप कोशिकाएँ अपेक्षाकृत बड़ी होती हैं।
• जंतु कोशिकाएँ विभिन्न आकृतियों की होती हैं। ये गोलाकार या अनियमित आकार की हो सकती हैं, जबकि पादप कोशिकाएँ ज़्यादातर आकार में समान होती हैं और चौकोर या घनाकार आकृति में होती हैं।
• जंतु कोशिका में कोशिका भित्ति (Cell Wall) नहीं होती, जबकि पादप कोशिका में कोशिका भित्ति पाई जाती है, जो सेल्यूलोज़ की बनी होती है। अतः कथन (1) सही है।
• प्लाज़्मा झिल्ली जंतु तथा पादप कोशिका दोनों में उपस्थित होती है। अतः कथन (2) असत्य है।
• जंतु कोशिका में कई छोटी-छोटी रसधानियाँ (Vacuole ) हो सकती हैं, जबकि पादप कोशिका में एक बड़ी केंद्रीय रसधानी होती है, जो पादप कोशिका में 90 प्रतिशत स्थान घेरती है। अत: कथन (3) सही है।
अतः विकल्प (c) सही उत्तर है।
3. निम्नलिखित कथनों पर विचार कीजिये :
1. शैवाल की कोशिका भित्ति केवल सेल्यूलोज़ की बनी होती है।
2. चिकनी अंतर्द्रव्यी जालिका (एन्डोप्लाज्मिक रेटीकुलम ) प्राणियों में केवल प्रोटीन संश्लेषण के मुख्य स्थल होते हैं।
3. गॉल्जीकाय का मुख्य कार्य द्रव्य को संवेष्टित कर अंतरकोशिकीय लक्ष्य तक पहुँचाना है।
उपर्युक्त कथनों में से कौन-सा/से सत्य नहीं है/हैं?
(a) केवल 1
(b) केवल 1 और 2
(c) केवल 2
(d) 1, 2 और 3
उत्तर : (b)
व्याख्या: कथन (1) असत्य है क्योंकि शैवाल की कोशिका भित्ति सेल्यूलोज़, गैलेक्टेन्स, मैनान्स व खनिज जैसे कैल्शियम कार्बोनेट की बनी होती है। असत्य है क्योंकि चिकनी अंतर्द्रव्यी जालिका प्राणियों में
• कथन (2) लिपिड संश्लेषण के मुख्य स्थल होते हैं। लिपिड की भाँति स्टीरॉयडल हार्मोन चिकने अंतर्द्रव्यी जालिका में होते हैं।
• कथन (3) सत्य है क्योंकि गॉल्जीकाय का मुख्य कार्य द्रव्य को संवेष्टित कर अंतर-कोशिकीय लक्ष्य तक पहुँचाना या कोशिका के बाहर स्रवण करना है। संवेष्टित द्रव्य अंतः प्रद्रव्यी जालिका से पुटिका के रूप में गॉल्जीकाय के सिस (Cis) सिरे से संगठित होकर परिपक्व सतह की ओर गति करते हैं।
अतः विकल्प (b) सही उत्तर है।
4. निम्नलिखित कथनों पर विचार कीजिये :
1. सभी यूकैरियोटिक कोशिकाएँ एक जैसी होती हैं ।
2. प्राणी कोशिकाओं में तारककाय (Controsome) अनुपस्थित होता है, जबकि सभी पादप कोशिकाओं में उपस्थित होता है।
उपर्युक्त में से कौन-सा/से कथन सत्य नहीं है/हैं?
(a) केवल 2
(b) केवल 1
(c) 1 और 2 दोनों
(d) न तो 1, न ही 2
उत्तर : (c)
व्याख्या: सभी आद्यजीव, पादप, प्राणी व कवक में यूकैरियोटिक कोशिकाएँ होती हैं। यूकैरियोटिक कोशिकाओं में आनुवंशिक पदार्थ गुणसूत्रों के रूप में व्यवस्थित रहते हैं। तथा सभी यूकैरियोटिक कोशिकाएँ एक जैसी नहीं होती हैं। पादप व जंतु कोशिकाएँ भिन्न-भिन्न होती हैं।
• पादप कोशिकाओं में कोशिका भित्ति, लवक एवं एक बड़ी केंद्रीय रसधानी मिलती है, जबकि प्राणी कोशिकाओं में ये अनुपस्थित होती हैं। दूसरी तरफ प्राणी कोशिकाओं में तारककाय (Centrosome) मिलता है, जो लगभग सभी पादप कोशिकाओं में अनुपस्थित होता है ।
अतः विकल्प (c) सही उत्तर है।
5. जंतु एवं पादपों में नई कोशिका का निर्माण किस प्रकार होता है ?
(a) जैविक पदार्थों से
(b) पूर्व स्थित कोशिकाओं के विभाजन से
(c) अजैविक पदार्थों से
(d) पुरानी कोशिकाओं के पुनरुत्पादन से
उत्तर : (b)
व्याख्या: विकल्प (b) सही उत्तर है क्योंकि पहली बार रुडोल्फ विच (1855) ने स्पष्ट किया कि कोशिका विभाजित होती है और नई कोशिकाओं का निर्माण पूर्व स्थित कोशिकाओं के विभाजन से होता है।
6. कवक व पौधों के जीवद्रव्य झिल्ली (Plasma Membrane) के बाहर पाया जाने वाला दृढ़ निर्जीव आवरण होता / होती है-
(a) कोशिका भित्ति
(b) एन्डोप्लाज्मिक रेटीकुलम
(c) साइटोस्केलेटन
(d) बाह्य झिल्ली
उत्तर : (a)
व्याख्या: कवक व पौधों की जीवद्रव्य झिल्ली के बाहर पाए जाने वाले दृढ़ निर्जीव आवरण को कोशिका भित्ति कहते हैं।
• एन्डोप्लाज्मिक रेटीकुलम ( अंतर्द्रव्यी जालिका) : यूकैरियोटिक कोशिकाओं के कोशिका द्रव्य में चपटे, आपस में जुड़े, थैली युक्त छोटी नलिकावत जालिका तंत्र बिखरे रहते हैं, जिसे अंतर्द्रव्यी जालिका कहते हैं।
• साइटोस्केलेटन : प्रोटीन युक्त विस्तृत जालिकावत तंतु जो कोशिका द्रव्य में मिलता है, उसे साइटोस्केलेटन या ‘साइटोपंजर’ कहते हैं।
अतः विकल्प (a) सही उत्तर है ।
7. निम्नलिखित में से किसकी अनुपस्थिति में अंतर्द्रव्यी जालिका चिकनी होती है?
(a) लाइसोसोम
(b) राइबोसोम
(c) रसधानी (Vacuoles)
(d) जीवद्रव्य झिल्ली
उत्तर : (b)
व्याख्या: प्रायः राइबोसोम अंतर्द्रव्यी जालिका के बाहरी सतह पर चिपके रहते हैं। जिस अंतर्द्रव्यी जालिका की सतह पर यह राइबोसोम मिलते हैं, उसे ‘खुरदरी अंतर्द्रव्यी जालिका’ कहते हैं। राइबोसोम की अनुपस्थिति पर अंतर्द्रव्यी जालिका चिकनी लगती है, अतः इसे ‘चिकनी अंतर्द्रव्यी जालिका’ कहते हैं।
• चिकनी अंतर्द्रव्यी जालिका प्राणियों में लिपिड संश्लेषण के मुख्य स्थल होते हैं। लिपिड की भाँति स्टीरॉयडल हार्मोन चिकनी अंतर्द्रव्यी जालिका में होते हैं।
अतः विकल्प (b) सही उत्तर है।
8. ‘टोनोप्लास्ट’ संबंधित है-
(a) संवेष्टन विधि द्वारा गॉल्जीकाय में बनी झिल्लीनुमा संरचना से
(b) विसरण द्वारा जल के प्रवाह से
(c) एकल झिल्ली से आवृत्त रसधानी से
(d) कोशिका में फैलाव से बनी जीवद्रव्य झिल्ली से
उत्तर : (c)
व्याख्या : कोशिकाद्रव्य में झिल्ली द्वारा घिरी जगह को ‘रसधानी’ कहते हैं।
• रसधानी एकल झिल्ली से आवृत्त होती है जिसे ‘टोनोप्लास्ट’ कहते हैं। पादप कोशिकाओं में यह कोशिका का 90% स्थान घेरता है।
अतः विकल्प (c) सही उत्तर है।
9. निम्नलिखित युग्मों पर विचार कीजिये :
1. क्रोमेटिन – केंद्रक में मिलने वाला पदार्थ
2. मनुष्यों में गुणसूत्र – 23 जोड़े
3. परिकेंद्रकीय अवकाश – केंद्रक आवरण के मध्य 5 से 10 नैनोमीटर का रिक्त स्थान
उपर्युक्त में से कौन-सा / से युग्म सुमेलित है / हैं?
(a) केवल 2
(b) केवल 2 और 3
(c) केवल 1 और 2
(d) 1, 2 और 3
उत्तर : (c)
व्याख्या : युग्म 1 सुमेलित है, कोशिका अंगक केंद्रक की खोज सर्वप्रथम रॉबर्ट ब्राउन ने 1831 से पूर्व की थी। बाद में फ्लेमिंग ने केंद्रक में मिलने वाला पदार्थ जो क्षारीय रंग से रंजित हो जाता है, उसे ‘क्रोमेटिन’ नाम दिया।
• युग्म 2 सुमेलित है, मनुष्यों में 23 जोड़े या 46 गुणसूत्र पाए जाते हैं ।
• युग्म 3 सुमेलित नहीं है क्योंकि केंद्रक आवरण दो समानांतर झिल्लियों से बना होता है, जिनके बीच 10 से 50 नैनोमीटर का रिक्त स्थान पाया जाता है, जिसे परिकेंद्रकीय अवकाश (Perinuclear Space) कहते हैं।
अत: विकल्प (c) सही उत्तर है।
10. लवक (प्लास्टिड) के संदर्भ में, निम्नलिखित कथनों पर विचार कीजिये :
1. वर्णी लवक (Chromoplast) विभिन्न आकृति एवं आकार के रंगहीन लवक होते हैं, जिनमें खाद्य पदार्थ संचित रहते हैं ।
2. हरित लवक (Chloroplast) के अंत: झिल्ली से घिरे हुए भीतर के स्थान को पीठिका (स्ट्रोमा) कहते हैं।
उपर्युक्त में से कौन-सा/से कथन सत्य है/हैं?
(a) केवल 1
(b) केवल 2
(c) 1 और 2 दोनों
(d) न तो 1, न ही 2
उत्तर : (b)
व्याख्या: कथन (1) असत्य है क्योंकि अवर्णी लवक (Leucoplast) विभिन्न आकृति एवं आकार के रंगहीन लवक होते हैं, जिनमें खाद्य पदार्थं संचित रहते हैं। जबकि वर्णी लवकों में वसा विलेय केरोटिनॉइड वर्णक जैसे- केरोटिन, जैंथोफिल व अन्य दूसरे मिलते हैं, इनके कारण पादपों में पीले, नारंगी व लाल रंग दिखाई पड़ते हैं।
• कथन (2) सत्य हैं, सूत्रकणिका की तरह हरित लवक द्विझिल्लिका युक्त होते हैं। उपर्युक्त दो में से इसकी भीतरी लवक झिल्ली अपेक्षाकृत कम पारगम्य होती है। हरितलवक के अंतः झिल्ली से घिरे हुए भीतर के स्थान को पीठिका (स्ट्रोमा) कहते हैं। तथा पीठिका में चपटे, झिल्लीयुक्त थैली जैसी संरचना संगठित होती है, जिसे थाइलोकोइड कहते ह
अतः विकल्प (b) सही उत्तर है।
11. निम्नलिखित में से सर्वाधिक उपयुक्त विकल्प का चयन कीजिये :
(a) जॉर्ज पैलेड ने प्रकाश सूक्ष्मदर्शी द्वारा राइबोसोम को सर्वप्रथम देखा।
(b) केमिलो ने इलेक्ट्रॉन सूक्ष्मदर्शी द्वारा सघन कणिकामय संरचना राइबोसोम को सर्वप्रथम देखा ।
(c) राइबोसोम राइबोन्यूक्लिक अम्ल व प्रोटीन की बनी झिल्ली से घिरे रहते हैं।
(d) जॉर्ज पैलेड ने इलेक्ट्रॉन सूक्ष्मदर्शी द्वारा सघन कणिकामय संरचना राइबोसोम को सर्वप्रथम देखा था ।
उत्तर : (d)
व्याख्या : जॉर्ज पैलेड (1953) ने इलेक्ट्रॉन सूक्ष्मदर्शी द्वारा सघन कणिकामय संरचना राइबोसोम को सर्वप्रथम देखा था। ये राइबोन्यूक्लिक अम्ल व प्रोटीन के बने होते हैं और किसी भी झिल्ली से घिरे नहीं रहते ।
अतः विकल्प (d) सर्वाधिक उपयुक्त है।

9. जैव अणु

1. जैव अणु के संदर्भ में निम्नलिखित कथनों पर विचार कीजिये :
1. जीव ऊतकों में मिलने वाले सभी कार्बनिक यौगिक, ‘जैव अणु’ कहलाते हैं।
2. ऐसे यौगिक जिनका अणुभार एक हज़ार डाल्टन से कम होता है, उन्हें सूक्ष्म अणु या जैव अणु कहते हैं; जबकि जो अम्ल अविलेय अंश पाए जाते हैं, उन्हें वृहद् अणु या ‘वृहद् जैव अणु’ कहते हैं।
उपर्युक्त कथनों में से कौन-सा/से सही है/हैं?
(a) केवल 1
(b) केवल 2
(c) 1 और 2 दोनों
(d) न तो 1, न ही 2
उत्तर : (c)
व्याख्या: कथन 1 सही है क्योंकि विश्लेषणात्मक तकनीकी का प्रयोग कर किसी भी यौगिक के आण्विक सूत्र व संभावित संरचना के बारे में जानकारी प्राप्त की जा सकती है। जिसके अनुसार जीव ऊतकों में मिलने वाले सभी कार्बनिक यौगिकों को ‘जैव अणु’ कहते हैं।
• जैव अणु अर्थात् जीवों में मिलने वाले रासायनिक यौगिक 2 प्रकार के होते हैं। एक वे हैं जिनका अणुभार एक हजार डाल्टन से कम होता है, उन्हें सामान्यतः सूक्ष्म अणु या जैव अणु कहते हैं; जबकि जो अम्ल अविलेय अंश में पाए जाते हैं, उन्हें वृहद् अणु या वृहद् जैव अणु कहते हैं। अतः कथन 2 सही है।
अतः विकल्प (c) सही उत्तर है।
2. निम्नलिखित कथनों पर विचार कीजिये :
1. प्रोटीन पॉलीपेप्टाइड होते हैं।
2. प्रत्येक प्रोटीन अमीनो अम्ल का बहुलक है।
3. प्रोटीन एक समबहुलक है।
प्रोटीन के संबंध में उपर्युक्त में से कौन-से कथन सत्य हैं?
(a) केवल 1 और 2
(b) केवल 1 और 3
(c) केवल 2 और 3
(d) 1, 2 और 3
उत्तर : (a)
व्याख्या : प्रोटीन पॉलीपेप्टाइड होते हैं। ये अमीनो अम्ल की रेखीय श्रृंखलाएँ । होती हैं, जो पेप्टाइड बंधों से जुड़ी होती हैं। अतः कथन (1) सत्य है ।
• प्रत्येक प्रोटीन अमीनो अम्ल का बहुलक है। अमीनो अम्ल 20 प्रकार के होने से प्रोटीन समबहुलक नहीं, बल्कि विषम बहुलक होते हैं।
अतः कथन (2) सत्य है एवं कथन (3) असत्य है।
अतः विकल्प (a) सही उत्तर है।
3. निम्नलिखित में से कौन से न्यूक्लिक अम्ल में विषम चक्रीय यौगिक नाइट्रोजन क्षार प्यूरीन के अंतर्गत आते हैं?
1. एडेनीन
2. ग्वानीन
3. यूरेसिल
4. थायमिन
5. साइटोसिन
नीचे दिये गए कूट के आधार पर सही विकल्प चुनिये :
(a) केवल 1 और 2
(b) केवल 1, 2 और 5
(c) केवल 3 और 4
(d) 1, 2, 3, 4 और 5
उत्तर : (a)
व्याख्या : न्यूक्लिक अम्ल में विषमचक्रीय यौगिक नाइट्रोजन क्षार, जैसेएडेनीन, ग्वानीन, यूरेसिल, साइटोसिन व थायमिन होते हैं। एडेनीन व ग्वानीन प्रतिस्थापित प्यूरीन हैं तथा अन्य तीन प्रतिस्थापित पिरीमिडीन हैं। विषम चक्रीय वलय को क्रमशः प्यूरीन व पिरीमिडीन कहते हैं।
अतः विकल्प (a) सही उत्तर है।
4. एंजाइम के संबंध में निम्नलिखित कथनों पर विचार कीजिये :
1. लगभग सभी एंजाइम प्रोटीन होते हैं।
2. किसी भी एंजाइम को रेखीय चित्र द्वारा चित्रित नहीं कर सकते।
3. एंजाइम में प्रोटीन की तरह द्वितीयक एवं तृतीयक संरचना नहीं मिलती।
उपर्युक्त में से कौन-सा/से कथन सही है/हैं?
(a) केवल 1
(b) 1 और 3
(c) केवल 3
(d) 1 और 2
उत्तर : (a)
व्याख्या: लगभग सभी एंजाइम प्रोटीन होते हैं। कुछ न्यूक्लिक अम्ल एंजाइम की तरह व्यवहार करते हैं, इन्हें राइबोजाइम्स कहते हैं। किसी भी एंजाइम को रेखीय चित्र द्वारा चित्रित कर सकते हैं। अतः कथन (1) सही है, एवं कथन (2) गलत है।
• एक एंजाइम में भी प्रोटीन की तरह प्राथमिक संरचना मिलती है, जो एमीनो अम्ल की श्रृंखला से बना होता है तथा प्रोटीन की तरह एंजाइम में भी द्वितीयक व तृतीयक संरचना मिलती है। अतः कथन (3) गलत है।
अतः विकल्प (a) सही उत्तर है।
5. निम्नलिखित कथनों पर विचार कीजिये :
कथन 1: निम्न तापक्रम एंजाइम को अस्थायी रूप से निष्क्रिय अवस्था में सुरक्षित रखता है, जबकि उच्च तापक्रम एंजाइम की क्रियाशीलता को समाप्त कर देता है।
कथन 2: उच्च तापक्रम एंजाइम के प्रोटीन को विकृत कर देता है।
उपर्युक्त कथनों के संदर्भ में, निम्नलिखित में से कौन-सा सही है?
(a) कथन 1 और कथन 2 दोनों सही हैं और कथन 2 कथन 1 की सही व्याख्या करता है।
(b) कथन 1 और कथन 2 दोनों सही हैं, किंतु कथन 2, कथन 1 की सही व्याख्या नहीं करता है।
(c) कथन 1 सही है, किंतु कथन 2 गलत है।
(d) कथन 1 गलत है, किंतु कथन 2 सही है।
उत्तर : (a)
व्याख्या : एंजाइम क्रियाविधि को प्रभावित करने वाले कारकों में से तापक्रम प्रमुख है।
• निम्न तापक्रम एंजाइम को अस्थायी रूप से निष्क्रिय अवस्था में सुरक्षित रखता है, जबकि उच्च तापक्रम एंजाइम की क्रियाशीलता को समाप्त कर देता है क्योंकि उच्च तापक्रम एंजाइम के प्रोटीन को विकृत कर देता है।
अतः विकल्प (a) सही उत्तर है।
6. निम्नलिखित कथनों पर विचार कीजिये :
1. प्राणी जगत में सर्वाधिक प्रचुरता में मिलने वाला प्रोटीन ‘रुबिस्को’ (RUBISCO) है।
2. संपूर्ण जैवमंडल में सर्वाधिक प्रचुरता में मिलने वाला प्रोटीन ‘कोलेजन’ है।
3. एंजाइम अभिक्रिया की दर को धीमी कर देते हैं।
उपर्युक्त कथनों में से कौन-सा/से सत्य नहीं है/हैं?
(a) केवल 1 और 3
(b) केवल 3
(c) केवल 1 और 2
(d) 1, 2 और 3
उत्तर : (d)
व्याख्या: प्राणी जगत में सर्वाधिक प्रचुरता में मिलने वाला प्रोटीन ‘कोलेजन’ है व संपूर्ण जैवमंडल में सर्वाधिक प्रचुरता में मिलने वाला प्रोटीन ‘रुबिस्को’ (Rubisco) है। अतः कथन 1 व 2 गलत हैं।
• एंजाइम्स प्रोटीन होते हैं, जो कोशिकाओं में जैव रासायनिक क्रियाओं की उत्प्रेरक शक्ति होते हैं। प्रोटीनमय एंजाइम्स को क्रियाशीलता हेतु ईष्टतम तापक्रम तथा पी. एच. (Ph) की आवश्यकता होती है। एंजाइम्स अभिक्रिया की दर को काफी तीव्र कर देते हैं। अतः कथन 3 गलत है।
अतः विकल्प (d) सही उत्तर है।

10. कोशिका चक्र और कोशिका विभाजन

1. निम्नलिखित कथनों पर विचार कीजिये :
1. कोशिका का निर्माण पूर्ववर्ती कोशिका के विभाजन से होता है।
2. लैंगिक जनन करने वाले किसी भी जीवधारी का जीवन चक्र एक कोशिकीय युग्मनज ( जाइगोट) से प्रारंभ होता है।
3. कोशिका विभाजन जीवधारी के वयस्क होने के बाद रुक जाता है।
उपर्युक्त कथनों में से कौन-से सही हैं?
(a) 1 और 3
(b) 2 और 3
(c) 1 और 2
(d) 1, 2 और 3
उत्तर : (c)
व्याख्या: कोशिका सिद्धांत के अनुसार एक कोशिका का निर्माण पूर्ववर्ती कोशिका से होता है। इस प्रक्रिया को ‘कोशिका विभाजन’ कहते हैं। अत: कथन 1 सही है।
• लैंगिक जनन करने वाले किसी भी जीवधारी का जीवन चक्र एक कोशिकीय युग्मनज (जाइगोट) से प्रारंभ होता है। अतः कथन 2 सही है।
• कोशिका विभाजन जीवधारी के वयस्क होने के बाद भी नहीं रुकता, बल्कि यह जीवन भर चलता रहता है। अत: कथन (3) गलत है।
अतः विकल्प (c) सही उत्तर है।
2. कोशिका चक्र की प्रावस्थाओं के संदर्भ में निम्नलिखित कथनों पर विचार कीजिये :
1. कोशिका चक्र की दो मूल प्रावस्थाओं में से कोशिका चक्र की कुल अवधि की 95 प्रतिशत से अधिक की अवधि अंतरावस्था (Interphase) में ही व्यतीत होती है ।
2. अंतरावस्था को ‘विश्राम प्रावस्था’ भी कहते हैं।
उपर्युक्त कथनों में से कौन-सा/से सही है/हैं?
(a) केवल 1
(b) केवल 2
(c) 1 और 2 दोनों
(d) न तो 1, न ही 2
उत्तर : (c)
3. निम्नलिखित में से कौन-से समसूत्री विभाजन (M-Phase) की अंत्यावस्था (Telophase) की विशेषताओं के संबंध में सत्य हैं?
1. गुणसूत्र विपरीत ध्रुवों की ओर एकत्रित हो जाते हैं तथा इनकी पृथक पहचान बनी रहती है।
2. गुणसूत्र समूह के चारों तरफ केंद्रक झिल्ली का निर्माण हो जाता है।
3. केंद्रिका, गॉल्जीकाय व अंतर्द्रव्यी जालिका का पुनर्निर्माण हो जाता है।
उपर्युक्त कथनों में से कौन-सा/से सही है/हैं?
(a) केवल 1
(b) केवल 1 और 3
(c) 1, 2 और 3
(d) केवल 2 और 3
उत्तर : (d)
व्याख्या: समसूत्री विभाजन की अंतिम अवस्था के प्रारंभ में अंत्यावस्था गुणसूत्र जो क्रमानुसार अपने ध्रुवों पर चले गए हैं; असंघनित होकर अपनी संपूर्णता को खो देते हैं। इस अवस्था की मुख्य विशेषताएँ निम्नवत हैं-
• गुणसूत्र विपरीत ध्रुवों की ओर एकत्र हो जाते हैं और इसकी पृथक पहचान समाप्त हो जाती है। इस प्रकार कथन ( 1 ) असत्य है।
• गुणसूत्र समूह के चारों तरफ केंद्रक झिल्ली का निर्माण हो जाता है।
• केंद्रिका, गॉल्जीकाय व अंतर्द्रव्यी जालिका का पुनर्निर्माण हो जाता है।
अतः कथन 2 और 3 सत्य हैं।
अतः विकल्प (d) सही उत्तर है।
4. ‘नारियल का तरल भ्रूणपोष’ निम्नलिखित में से किसका उदाहरण है?
(a) संकोशिका (Syncytium)
(b) कोशिका पट्टिका (Cell-Plate)
(c) तनुपट्ट (Leptotene)
(d) युग्मपट्ट (Zygotene)
उत्तर : (a)
व्याख्या: कुछ जीवों में केंद्रक विभाजन के साथ कोशिकाद्रव्य का विभाजन नहीं हो पाता है; इसके परिणामस्वरूप एक ही कोशिका में कई केंद्रक बन जाते हैं, ऐसी बहुकेंद्रकी कोशिका को ‘संकोशिका’ कहते हैं। उदाहरण के लिये – नारियल का तरल भ्रूणपोष ।
• कोशिका पट्टिका : पादप कोशिकाओं में नई कोशिका भित्ति निर्माण कोशिका के केंद्र से शुरू होकर बाहर की ओर पूर्व स्थित पार्श्व कोशिका भित्ति से जुड़ जाता है। नई कोशिका भित्ति का निर्माण एक साधारण पूर्वगामी रचना से प्रारंभ होता है, जिसे कोशिका पट्टिका कहते हैं।
• तनुपट्ट (लिप्टोटीन) : यह अर्द्धसूत्री विभाजन I के अंतर्गत आने वाली एक अवस्था है, जिसके दौरान साधारण सूक्ष्मदर्शी द्वारा देखने पर गुणसूत्र धीरे-धीरे स्पष्ट दिखाई देने लगते हैं।
• युग्मपट्ट ( जाइगोटीन ) : यह भी अर्द्धसूत्री विभाजन I के अंतर्गत आने वाली एक अवस्था है, जिसके दौरान गुणसूत्रों का आपस में युग्मन प्रारंभ हो जाता है और इस प्रकार की संबद्धता को सूत्रयुग्मन कहते हैं।
अतः विकल्प (a) सही उत्तर है।
5. अर्द्धसूत्री विभाजन (Meiosis) के संबंध में, निम्नलिखित कथनों पर विचार कीजिये :
1. इस कोशिका विभाजन द्वारा बनने वाली अगुणित संतति कोशिकाओं में गुणसूत्रों की संख्या दोगुनी हो जाती है।
2. अर्द्धसूत्री विभाजन के दौरान केंद्रक व कोशिका विभाजन के दो अनुक्रमिक चक्र संपन्न होते हैं, जबकि डीएनए प्रतिकृति (DNA Replication) का केवल एक चक्र पूर्ण होता है।
उपर्युक्त कथनों में से कौन-सा/से सत्य है/हैं?
(a) केवल 1
(b) केवल 2
(c) 1 और 2 दोनों
(d) न तो 1, न ही 2
उत्तर : (b)
6. निम्नलिखित में से ऐसी कौन-सी प्रक्रिया है, जिसके द्वारा लैंगिक जनन करने वाले जीवों की प्रत्येक जाति में गुणसूत्रों की संख्या पीढ़ीगत संरक्षित रहती है ?
(a) समसूत्री विभाजन द्वारा
(b) अर्द्धसूत्री विभाजन द्वारा
(c) असूत्री विभाजन द्वारा
(d) पूर्व-सूत्री विभाजन द्वारा
उत्तर : (b)
व्याख्या : अर्द्धसूत्री विभाजन एक ऐसी प्रक्रिया है जिसके द्वारा लैंगिक जनन करने वाले जीवों की प्रत्येक जाति में विशिष्ट गुणसूत्रों की संख्या पीढ़ी-दर-पीढ़ी संरक्षित रहती है। यद्यपि विरोधाभासी प्रक्रिया के परिणामस्वरूप गुणसूत्रों की संख्या आधी हो जाती है। इसके द्वारा जीवधारियों की संख्या में एक पीढ़ी से दूसरी पीढ़ी तक आनुवंशिक विभिन्नताएँ बढ़ती जाती हैं।
अतः विकल्प (b) सही उत्तर है ।

11. पौधों में परिवहन

1. निम्नलिखित कथनों पर विचार कीजिये :
1. मूलीय पादपों में जाइलम परिवहन आवश्यक रूप से एकदिशीय होता है।
2. विसरण पादप शरीर में गैसीय गति का अकेला माध्यम है।
3. विसरण की दर ताप तथा दाब से प्रभावित नहीं होती है।
उपर्युक्त कथनों में से कौन-सा/से सही है/हैं?
(a) केवल 2
(b) 1 और 2
(c) 1, 2 और 3
(d) केवल 3
उत्तर : (b)
व्याख्या: मूलीय पादपों में जाइलम परिवहन आवश्यक रूप से एकदिशात्मक अर्थात् मूल से तने तक होता है। कार्बनिक और खनिज पोषकों का परिवहन बहुदिशात्मक होता है। अतः कथन 1 सही है।
• पौधों के लिये विसरण अत्यंत ही महत्त्वपूर्ण है क्योंकि पादप शरीर में गैसीय गति का यह अकेला माध्यम है। अतः कथन 2 सही है। सांद्रता की प्रवणता, उन्हें अलग करने वाली झिल्ली की
• विसरण की दर पारगम्यता, ताप तथा दाब से प्रभावित होती है। अतः कथन 3 असत्य है।
अतः विकल्प (b) सही उत्तर है।
2. ‘पोरिन’ संबंधित है-
(a) प्रोटीन से
(b) लिपिड से
(c) विटामिन से
(d) झिल्ली से
उत्तर : (a)
व्याख्या: ‘पोरिन’ एक प्रकार का प्रोटीन है जो प्लास्टिड, माइटोकॉण्ड्रिया तथा बैक्टीरिया की बाह्य झिल्ली में बड़े आकार के छिद्रों का निर्माण करती है ताकि झिल्ली से होकर प्रोटीन के छोटे आकार के अणु भी उसमें से गुजर सकें।
अतः विकल्प (a) सही उत्तर है।
3. निम्नलिखित कथनों पर विचार कीजिये :
कथन 1 : सभी विलयनों में शुद्ध जल की अपेक्षा जल विभव निम्न होता है।
कथन 2 : विलेय के द्रवीकरण के कारण जल की सांद्रता घट जाती है।
उपर्युक्त कथनों के संदर्भ में निम्नलिखित में से कौन – सा सही है?
(a) कथन 1 और कथन 2 दोनों सही हैं और कथन 2, कथन 1 की सही व्याख्या करता है।
(b) कथन 1 और कथन 2 दोनों सही हैं, किंतु कथन 2, कथन 1 की सही व्याख्या नहीं करता है।
(c) कथन 1 सही है, किंतु कथन 2 गलत है।
(d) कथन 1 गलत है, किंतु कथन 2 सही है।
उत्तर : (a)
व्याख्या : यदि कुछ विलेय शुद्ध जल में घोले जाते हैं, तो घोल में मुक्त पानी के अणु कम हो जाते हैं जिससे जल की सांद्रता (स्वतंत्र ऊर्जा ) घट जाती है और जल विभव भी कम हो जाता है। इसलिये सभी विलयनों में शुद्ध जल की अपेक्षा जल विभव निम्न होता है। इस निम्नता का परिमाण एक विलेय के द्रवीकरण के कारण है जिसे ‘विलेय विभव’ कहा जाता है।
अतः विकल्प (a) सही उत्तर है।
4. जब कोशिका से पानी बाहर निकल जाए तथा पादप कोशिका की कोशिका झिल्ली (Cell Membrane) सिकुड़कर कोशिका भित्ति से अलग हो जाती है तब, क्या होता है?
(a) परासरण (Osmosis)
(b) अंत: शोषण (Imbibition)
(c) जीवद्रव्यकुंचन (Plasmolysis)
(d) वाष्पोत्सर्जन (Transpiration)
उत्तर : (c)
व्याख्या: जीवद्रव्यकुंचन तब होता है जब कोशिका से पानी बाहर निकल जाए तथा पादप कोशिका की कोशिका झिल्ली सिकुड़कर कोशिका भित्ति से अलग हो जाती है। यह तब होता है, जब एक कोशिका ( या ऊतक) को अतिपरासारी घोल डाला जाता है, सबसे पहले जीवद्रव्य से पानी बाहर आता है फिर रसधानी से ।
• जब कोशिका से विसरण द्वारा पानी निकलकर बाह्यकोशिका द्रव्य में जाता है, तब जीवद्रव्य कोशिका भित्ति से अलग हो जाती है और इसे कोशिका का जीवद्रव्यकुंचन कहा जाता है।
अतः विकल्प (c) सही उत्तर है।
5. विसरण के संदर्भ में निम्नलिखित कथनों पर विचार कीजिये :
1. पौधों में लंबी दूरी तक पदार्थों का परिवहन केवल विसरण के द्वारा हो सकता है।
2. विसरण एक धीमी प्रक्रिया है।
उपर्युक्त में से कौन-सा/से कथन सही है/हैं?
(a) केवल 1
(b) केवल 2
(c) 1 और 2 दोनों
(d) न तो 1, न ही 2
उत्तर : (b)
व्याख्या : लंबी दूरी तक पदार्थों का परिवहन केवल विसरण द्वारा नहीं हो सकता है क्योंकि विसरण एक धीमी प्रक्रिया है, यह छोटी दूरी तक अणुओं को पहुँचाने में कारगर है। उदाहरण के लिये एक प्रारूपिक पादप कोशिका ( लगभग 50pm ) के आर-पार अणुओं को गतिमान करने में लगभग 2.5 sec का समय लगता है। अतः कथन 1 गलत है एवं कथन 2 सही है।
अतः विकल्प (b) सही उत्तर है ।
6. निम्नलिखित कथनों पर विचार कीजिये :
1. निष्क्रिय परिवहन में विसरण के द्वारा झिल्ली के आर-पार बिना ऊर्जा व्यय किये पोषकों की गति सांद्रता प्रवणता के अनुसार होती है।
2. सक्रिय परिवहन में एटीपी की ऊर्जा, अणुओं को सांद्रण प्रवणता के विरुद्ध झिल्ली के पार पंप करती है।
3. जल विभव पानी की स्थितिज ऊर्जा है, जो जल के अणुओं की गति में सहायता करती है।
उपर्युक्त में से कौन-से कथन सत्य हैं ?
(a) केवल 2 और 3
(b) केवल 1 और 3
(c) केवल 1 और 2
(d) 1, 2 और 3
उत्तर : (d)
व्याख्या : निष्क्रिय परिवहन ( विसरण, परासरण) तथा सक्रिय परिवहन, जीवों में पोषकों को झिल्लिकाओं के आर-पार संचरित करने के दो तरीके हैं। निष्क्रिय परिवहन में विसरण के द्वारा झिल्ली के आर-पार बिना ऊर्जाव्यय किये पोषकों की गति सांद्रता प्रवणता के अनुसार होती है। पदार्थों का विसरण आकार तथा उसके जल में या कार्बनिक विलयन में घुलनशीलता पर निर्भर करता है। अतः कथन 1 सत्य है।
• सक्रिय परिवहन में एटीपी ऊर्जा, सांद्रण प्रवणता के विरुद्ध झिल्ली के पार अणुओं को पंप करती है। तथा जल विभव पानी की स्थितिज ऊर्जा है, जो जल के अणुओं की गति में सहायता करती है। अतः कथन 2 और कथन 3 सत्य हैं।
अतः विकल्प (d) सही उत्तर है।

12. खनिज पोषण

1. जल संवर्धन (Hydroponics) के बारे में निम्नलिखित कथनों पर विचार कीजिये :
1. इसको जर्मन वैज्ञानिक जूलियस वॉन सच्स (Julius Von Sachs) ने सर्वप्रथम दर्शाया था।
2. यह पादपों को पोषक विलयन घोल में उगाने की एक तकनीक है।
उपर्युक्त में से कौन-सा/से कथन सत्य है/हैं?
(a) केवल 1
(b) केवल 2
(c) 1 और 2 दोनों
(d) न तो 1 और न ही 2
उत्तर : (c)
व्याख्या : जर्मनी के प्रसिद्ध पादप वैज्ञानिक जूलियस वॉन सच्स ने सर्वप्रथम यह दिखाया कि पादपों को मृदा की अनुपस्थिति में पोषक विलयन के घोल में उगाया जा सकता है। पादपों को पोषक विलयन के घोल में उगाने की इस प्रक्रिया को वृहद् जल संवर्धन के नाम से जाना जाता है। अतः उपर्युक्त दोनों कथन सही हैं।
अतः विकल्प (c) सही उत्तर है।
2. पौधों में क्षीण विकास के लक्षण हैं-
(a) क्लोरोसिस (Chlorosis)
(b) पुष्पन में देरी (Delay in Flowering)
(c) नेक्रोसिस (Necrosis)
(d) उपर्युक्त सभी
उत्तर : (d)
3. वृहद् पोषक के संदर्भ में निम्नलिखित कथनों पर विचार कीजिये :
1. वृहद् पोषकों को सामान्यतः पादप शुष्क पदार्थ का 1 से 10 मिग्रा./ली. की सांद्रता से विद्यमान होना चाहिये ।
2. इस श्रेणी में कार्बन, हाइड्रोजन, ऑक्सीजन, कैल्शियम आदि तत्त्व अनुपस्थित रहते हैं।
3. वृहद् पोषक प्रक्रिया में कार्बन, हाइड्रोजन तथा ऑक्सीजन मुख्यतः CO2 एवं H2 से प्राप्त होते हैं।
उपर्युक्त में से कौन-सा /से कथन सत्य है/हैं?
(a) केवल 1
(b) केवल 2
(c) 1, 2 एवं 3
(d) केवल 1 और 3
उत्तर : (d)
4. पौधों द्वारा खनिजों का अवशोषण निम्नलिखित में से किसके द्वारा होता है ?
(a) एपोप्लास्ट पथ (Apoplast Pathway) के द्वारा
(b) सिमप्लास्ट पथ (Symplast Pathway) के द्वारा
(c) (a) और (b) दोनों द्वारा
(d) उपर्युक्त में से कोई नहीं
उत्तर : (c)
व्याख्या: पौधे खनिज तत्त्वों का अवशोषण एपोप्लास्ट (बाह्य स्थान ) तथा सिमप्लास्ट (आंतरिक स्थान) पथ द्वारा करते हैं। एपोप्लास्ट पौधे का निकटवर्ती कोशिका भित्ति का तंत्र है। यह जड़ों की अंतस्त्वचा की कैस्पेरियन पट्टियों को छोड़कर पूरे पौधे में पाया जाता है।
• सिमप्लास्ट तंत्र पौधे के अंत: संबंधित जीवद्रव्य का एक तंत्र है। यह समीपवर्ती कोशिकाएँ कोशिकाद्रव्यी तंतुओं से जुड़ी रहती हैं।
अतः विकल्प (c) सही उत्तर है।
5. पौधों में नाइट्रोजन किसका मुख्य संघटक है?
(a) प्रोटीन
(b) हार्मोन
(c) (a) तथा (b) दोनों
(d) इनमें से कोई नहीं
उत्तर : (c)
6. अपचयित एमोनीकरण प्रक्रिया के संदर्भ में निम्नलिखित कथनों पर विचार कीजिये :
1. इस प्रक्रिया में अमोनिया कीटोग्लूटेरिक अम्ल के साथ अभिक्रिया करती है।
2. अमोनिया कीटोग्लूरेटिक अम्ल के साथ क्रिया करके ग्लूटेमिक अम्ल बनाती है।
उपर्युक्त में से कौन-सा/से कथन सत्य है/हैं?
(a) केवल 1
(b) केवल 2
(c) 1 और 2 दोनों
(d) न तो 1 और न ही 2
उत्तर : (c)
7. नाइट्रोजन स्थिरीकरण (Nitrogen Fixation) के संदर्भ में निम्नलिखित कथनों पर विचार कीजिये :
1. नाइट्रोजन स्थिरीकरण के लिये शक्तिशाली अपचायक और एटीपी (ATP) के रूप में ऊर्जा की अनिवार्यता नहीं होती है।
2. नाइट्रोजन स्थिरीकरण सूक्ष्मजीवों मुख्यतः राइजोबियम (Rhizobium) से होता है।
उपर्युक्त में से कौन-सा/से कथन सत्य है/हैं?
(a) केवल 1
(b) केवल 2
(c) 1 और 2 दोनों
(d) न तो 1 और न ही 2
उत्तर : (b)
व्याख्या : नाइट्रोजन जीवन के अस्तित्व के लिये अति अनिवार्य है। पौधें वातावरणीय नाइट्रोजन का उपयोग प्रत्यक्ष नहीं कर पाते हैं। लेकिन कुछ पादप मुख्यतः लेग्यूम की जड़ें वातावरणीय N2 को जैविक उपयोगी रूप में बदल देती हैं।
• नाइट्रोजन स्थिरीकरण के लिये शक्तिशाली अपचायक और एटीपी (ATP) के रूप में ऊर्जा की अनिवार्यता होती है।
• नाइट्रोजन स्थिरीकरण सूक्ष्मजीवों मुख्यतः राइजोबियम से होता है। अतः दिये गए कथनों में केवल कथन 2 सही है। अतः विकल्प (b) सही उत्तर है।

13. उच्च पादपों में प्रकाश संश्लेषण

1. स्वपोषी के संदर्भ में निम्नलिखित कथनों पर विचार कीजिये :
1. स्वपोषी संश्लेषण केवल पौधों में ही पाया जाता है।
2. वे सभी जीव जो अपने भोजन के लिये पौधों पर निर्भर होते हैं, स्वपोषी कहलाते हैं।
उपर्युक्त में से कौन-सा/से कथन सत्य है/हैं?
(a) केवल 1
(b) केवल 2
(c) 1 और 2 दोनों
(d) न तो 1 और न ही 2
उत्तर : (a)
व्याख्या : हरे पौधे अपने लिये आवश्यक भोजन का निर्माण ‘प्रकाश संश्लेषण’ द्वारा करते हैं, अतः वे स्वपोषी कहलाते हैं। अन्य सभी जीव | जो अपने भोजन के लिये पौधों पर निर्भर होते हैं, विषमपोषी कहलाते हैं।
• पौधों द्वारा प्रकाश संश्लेषण में उपयोग की गई सौर ऊर्जा पृथ्वी पर जीवन का आधार है। अतः कुल मिलाकर पृथ्वी पर रहने वाले सारे जीव ऊर्जा के लिये सूर्य के प्रकाश निर्भर होते हैं। अतः उपर्युक्त कथनों में केवल कथन – 1 सही है।
2. निम्नलिखित कथनों पर विचार कीजिये :
1. जोसेफ प्रीस्टले ने 1774 में ऑक्सीजन की खोज की थी।
2. टी. डब्ल्यू. एंजिलमैन ने प्रिज़्म की सहायता से प्रकाश संश्लेषण का पहला सक्रिय स्पेक्ट्रम दर्शाया।
उपर्युक्त में से कौन-सा/से कथन सत्य है / हैं?
(a) केवल 1
(b) केवल 2
(c) 1 और 2 दोनों
(d) न तो 1 और न ही 2
उत्तर : (c)
व्याख्या : जोसेफ प्रीस्टले ने 1770 में बहुत से प्रयोग किये जिनसे पता लगा कि हरे पौधों की वृद्धि में हवा की एक अनिवार्य भूमिका है। प्रीस्टले 1774 में ऑक्सीजन की खोज की थी।
• टी.डब्ल्यू. एंजिलमैन ने प्रिज्म की सहायता से प्रकाश को स्पेक्ट्रमी घटकों में अलग किया और फिर एक हरे शैवाल क्लैडोफोरा को जिसे ऑक्सी बैक्टीरिया के निलंबन में रखा गया था, को प्रदीप्त किया।
• बैक्टीरिया का प्रयोग ऑक्सीजन निकलने के केंद्र का पता लगाने के लिये किया था। उन्होंने पाया कि बैक्टीरिया प्रमुखतः लाल एवं नीले प्रकाश क्षेत्रों में एकत्र हो गए थे। इस तरह से उन्होंने प्रकाश संश्लेषण का पहला सक्रिय स्पेक्ट्रम वर्णित किया।
अतः विकल्प (c) सही उत्तर है।
3. पत्तियों का हरा रंग किस वर्णक (Pigment) के कारण दिखाई देता है?
(a) क्लोरोफिल
(b) जैन्थोफिल
(c) कैरोटीनॉयड्स
(d) उपर्युक्त सभी
उत्तर : (d)
व्याख्या: क्रोमैटोग्राफी से पता लगता है कि पत्तियों में स्थित वर्णक के कारण जो हरा रंग दिखाई देता है, वह किसी एक वर्णक के कारण नहीं बल्कि चार वर्णकों- क्लोरोफिल A, क्लोरोफिल B, जैन्थोफिल तथा कैरोटीनॉयड्स के कारण होता है।
अतः विकल्प (d) सही उत्तर है।
4. निम्नलिखित कथनों पर विचार कीजिये :
1. सूक्ष्मजीव वैज्ञानिक ‘कोर्नेलियस वैन नील’ का प्रयोग बैंगनी एवं हरे बैक्टीरिया पर आधारित था ।
2. इन्होंने अपने प्रयोग से निष्कर्ष निकाला कि हरे पौधों द्वारा निकाली गई ऑक्सीजन H2O से आती है, न कि कार्बन डाइऑक्साइड से।
उपर्युक्त में से कौन-सा/से कथन सत्य है/हैं?
(a) केवल 1
(b) केवल 2
(c) 1 और 2 दोनों
(d) न तो 1 और न ही 2
उत्तर : (c)
5. चक्रीय फॉस्फोरिलीकरण के संदर्भ में निम्नलिखित कथनों पर विचार कीजिये :
1. इसमें ऑक्सीजन मुक्त होती है।
2. यह प्रक्रिया मुख्यतः प्रकाश संश्लेषी जीवाणुओं में होती है।
उपर्युक्त में से कौन-सा/से कथन सत्य है/हैं?
(a) केवल 1
(b) केवल 2
(c) 1 और 2 दोनों
(d) न तो 1 और न ही 2
उत्तर : (b)
व्याख्या : चक्रीय फॉस्फोरिलीकरण प्रक्रिया में ऑक्सीजन मुक्त नहीं होती है। इसमें इलेक्ट्रॉन पर्णहरित अणु से आते हैं तथा वापस पर्णहरित अणु पर चले जाते हैं।
• यह प्रक्रिया मुख्यतः प्रकाश संश्लेषी जीवाणुओं में होती है। अतः उपर्युक्त कथनों में से केवल कथन 2 सही है ।
अतः विकल्प (b) सही उत्तर है।
6. प्रकाश अभिक्रिया के संदर्भ में निम्नलिखित कथनों पर विचार कीजिये :
1. प्रकाश अभिक्रिया में प्रकाश अवशोषण, जल विघटन, ऑक्सीजन निष्कर्षण तथा उच्च – ऊर्जा रसायन माध्यमिकों, जैसे एटीपी तथा एनएडीपीएच का निर्माण शामिल है।
2. इस प्रक्रिया में अनेक प्रोटीन कॉम्पलेक्स सम्मिलित होते हैं।
उपर्युक्त में से कौन-सा/से कथन सत्य है/हैं?
(a) केवल 1
(b) केवल 2
(c) 1 और 2 दोनों
(d) न तो 1 और न ही 2
उत्तर : (c)
7. चक्रीय फोटो- फॉस्फोरीलेशन कब होता है?
(a) जब प्रकाश की तरंगदैर्ध्य 680nm से अधिक हो ।
(b) जब प्रकाश की तरंगदैर्ध्य 690nm से अधिक हो ।
(c) जब प्रकाश की तरंगदैर्ध्य 650 nm से कम हो।
(d) जब प्रकाश की तरंगदैर्ध्य 680 nm कम हो।
उत्तर : (a)
व्याख्या: जीवों में ऑक्सीकरणीय पदार्थों से ऊर्जा निकालने तथा उसे बंध – ऊर्जा के रूप में संचय करने की क्षमता होती है। विशेष पदार्थ, जैसे- एटीपी, इस ऊर्जा को अपने रासायनिक बंध में संजोए रखती है। इसे कोशिकाओं द्वारा ( माइटोकॉण्ड्रिया तथा क्लोरोप्लास्ट में ) एटीपी संश्लेषण की प्रक्रिया का ‘फॉस्फोरीलेशन’ कहते हैं।
• चक्रीय फोटो- फॉस्फोरीलेशन तभी होता है जब उत्तेजना के लिये प्रकाश की तरंगदैर्ध्य 680 nm से अधिक हो।
अतः विकल्प (a) सही उत्तर है ।
8. केल्विन चक्र को कितने भागों में बाँटा जा सकता है?
(a) 1
(b) 2
(c) 3
(d) 5
उत्तर : (c)

14. पादप में श्वसन

1. ग्लाइकोलिसिस के संबंध में निम्नलिखित कथनों पर विचार कीजिये :
1. ग्लाइकोलिसिस को EMP मार्ग (Embden Meyerhof Parnas Pathway) भी कहते हैं।
2. इस प्रक्रिया में ऑक्सीजन का प्रयोग होता है।
उपर्युक्त में से कौन-सा/से कथन सत्य है/हैं?
(a) केवल 1
(b) केवल 2
(c) 1 और 2 दोनों
(d) न तो 1 और न ही 2
उत्तर : (a)
व्याख्या: ग्लाइकोलिसिस को EMP मार्ग (Embden Meyerh of Parnas Pathway) भी कहते हैं। यह कोशिका द्रव्य में सम्पन्न होता है। इसमें ऑक्सीजन का प्रयोग नहीं होता, अत: ऑक्सी (Aerobic) तथा अनॉक्सी (Anaerobic) श्वसन दोनों में यह क्रिया होती है। इस क्रिया के अंत में ग्लूकोज के एक अणु से पायरुविक अम्ल के दो अणु बनते हैं। अतः उपर्युक्त कथनों में केवल कथन 1 सही है।
अतः विकल्प (a) सही उत्तर है ।
2. ग्लाइकोलिसिस सजीवों में कहाँ संपन्न होता है?
(a) केंद्रक
(b) त्वचा
(c) ऊतक
(d) कोशिका द्रव्य
उत्तर : (d)
व्याख्या : ग्लाइकोलिसिस शब्द की उत्पत्ति ग्रीक शब्द ग्लाइकोस ( अर्थात् शर्करा एवं लाइसिस) अर्थात् टूटना से हुआ है। ग्लाइकोलिसिस की प्रक्रिया गुस्ताव इंबेडेन, ओटो मेयर हॉफ तथा जे. पर्नास द्वारा दिया गया इसलिये इसे सामान्यतः ‘ईएमपी पाथ’ कहते हैं। ग्लाइकोलिसिस कोशिका द्रव्य में संपन्न होता है और यह सभी सजीवों में मिलता है।
अतः विकल्प (d) सही उत्तर है।
3. इलेक्ट्रॉन परिवहन तंत्र (Electron Transport System) के संदर्भ में निम्नलिखित कथनों पर विचार कीजिये :
1. यह उपापचयी पथ है, जिसके द्वारा इलेक्ट्रॉन एक वाहक से अन्य वाहक की ओर गुज़रता है।
2. इलेक्ट्रॉन परिवहन तंत्र माइटोकॉण्ड्रिया की भीतरी झिल्ली पर संपन्न होता है।
उपर्युक्त में से कौन-सा/से कथन सत्य है/हैं?
(a) केवल 1
(b) केवल 2
(c) 1 और 2 दोनों
(d) न तो 1 और न ही 2
उत्तर : (c)
व्याख्या : उपापचयी पथ, जिसके द्वारा इलेक्ट्रॉन एक वाहक से अन्य वाहक की ओर गुज़रता है, उसे इलेक्ट्रॉन परिवहन तंत्र (ETS) कहते हैं।
• यह माइटोकॉण्ड्रिया के भीतरी झिल्ली पर संपन्न होता है। अतः उपर्युक्त दोनों कथन सत्य हैं।
अतः विकल्प (c) सही उत्तर है ।
4. ऑक्सी श्वसन के बारे में निम्नलिखित कथनों पर विचार कीजिये :
1. माइटोकॉण्ड्रिया में होने वाले ऑक्सी श्वसन के दौरान ग्लाइकोलिसिस का अंतिम उत्पाद पायरुवेट कोशिका द्रव्य से माइटोकॉण्ड्रिया में परिवहन किया जाता है।
2. ऑक्सी- श्वसन में पूर्ण ऑक्सीकरण नहीं होता है।
उपर्युक्त में से कौन-सा/से कथन सत्य है/हैं?
(a) केवल 1
(b) केवल 2
(c) 1 और 2 दोनों
(d) न तो 1 और न ही 2
उत्तर : (a)
व्याख्या : ऑक्सी- श्वसन वह प्रक्रिया है, जिसके द्वारा रासायनिक पदार्थों का ऑक्सीजन की उपस्थिति में पूर्ण ऑक्सीकरण होता है तथा जिसके पश्चात् कार्बन डाइऑक्साइड, जल तथा ऊर्जा निकलती है।
• माइटोकॉण्ड्रिया में होने वाले ऑक्सी श्वसन के दौरान ग्लाइकोलिसिस का अंतिम उत्पाद पायरुवेट कोशिका द्रव्य से माइटोकॉण्ड्रिया में परिवहन किया जाता है।
अतः उपर्युक्त कथनों में केवल कथन 1 सही है।
अतः विकल्प (a) सही उत्तर है ।
5. कोशिकीय श्वसन में ग्लूकोज से पायरुविक अम्ल का बनना कहलाता है—
(a) C3 चक्र
(b) क्रेब्स चक्र
(c) हाइड्रोलिसिस
(d) ग्लाइकोलिसिस
उत्तर : (d)
व्याख्या: ग्लाइकोलिसिस शब्द की उत्पत्ति ग्रीक शब्द ग्लाइकोस अर्थात् | शर्करा एवं लाइसिस अर्थात् टूटना से हुआ है।
• ग्लाइकोलिसिस की प्रक्रिया कोशिका द्रव्य में संपन्न होती है और यह सभी सजीवों में मिलती है। इस प्रक्रिया में ग्लूकोज आंशिक ऑक्सीकरण द्वारा पायरुविक अम्ल के दो अणुओं में बदल जाता है।
अतः विकल्प (d) सही उत्तर है।
6. किण्वन के संदर्भ में निम्नलिखित कथनों पर विचार कीजिये :
1. किण्वन में यीस्ट द्वारा ग्लूकोज का अनॉक्सी परिस्थितियों में अपूर्ण ऑक्सीकरण होता है।
2. एंजाइम, पायरुविक अम्ल डिकार्बोक्सिलेज एवं एल्कोहल डिहाइड्रोजिनेस इस अभिक्रिया को उत्प्रेरित करते हैं।
उपर्युक्त में से कौन-सा/से कथन सत्य है/हैं?
(a) केवल 1
(b) केवल 2
(c) 1 और 2 दोनों
(d) न तो 1 और न ही 2
उत्तर : (c)
व्याख्या: किण्वन में यीस्ट द्वारा ग्लूकोज का अनॉक्सी परिस्थितियों में अपूर्ण ऑक्सीकरण होता है। जिसमें अभिक्रियाओं के विभिन्न चरणों द्वारा पायरुविक अम्ल, कार्बन डाइऑक्साइड एवं एथेनॉल में परिवर्तित हो जाता है।
• एंजाइम, पायरुविक अम्ल डिकार्बोक्सेिलेज (Decarboxylase) एवं एल्कोहल डिहाइड्रोजिनेस इस अभिक्रिया को उत्प्रेरित करते हैं। दूसरे जीव जैसे कुछ बैक्टीरिया पायरुविक अम्ल से लैक्टिक अम्ल का निर्माण करते हैं।
अतः विकल्प (c) सही उत्तर है।

15. पादप वृद्धि एवं परिवर्धन

1. निम्नलिखित कथनों पर विचार कीजिये :
1. पौधों में जीवित विभेदित कोशिकाएँ कुछ खास परिस्थितियों में विभाजन की क्षमता पुनः प्राप्त कर सकती हैं, इस क्षमता को निर्विभेदन कहा जाता है।
2. निर्विभेदित कोशिकाओं के द्वारा उत्पादित कोशिका बाद में पुन: विभाजन की क्षमता खो देती है, इसे पुनर्विभेदन कहा जाता है।
उपर्युक्त में से सही कथन की पहचान कीजिये :
(a) केवल 1
(b) केवल 2
(c) 1 और 2 दोनों
(d) न तो 1, न ही 2
उत्तर : (c)
व्याख्या : मूल शिखाग्र विभज्योतक (Root apical Meristem) एवं प्ररोह शिखाग्र विभज्योतक (Shoot apical Meristem) कैंबियम विभेदित होते हैं। तथा विशिष्ट क्रियाकलाप को संपन्न करने के लिये परिपक्व होते हैं, यह परिपक्वता की ओर अग्रसर होने वाली कार्यवाही ‘विभेदन’ कहलाती है।
• कथन 1 सही है क्योंकि पौधों में अन्य रोचक तथ्य दिखाई देते हैं जैसे- जीवित कोशिकाएँ कुछ खास परिस्थितियों में विभाजन की क्षमता पुनः प्राप्त कर सकती हैं, इस क्षमता को ‘निर्विभेदन’ (Dedifferentiation) कहते हैं।
• कथन 2 सही है क्योंकि निर्विभेदित कोशिकाओं / ऊतकों के द्वारा उत्पादित कोशिका बाद में फिर से विभाजन की क्षमता खो देती है ताकि विशिष्ट कार्यों को संपादित किया जा सके, इसे ‘पुनर्विभेदन’ (Redifferentiation) कहते हैं।
अतः विकल्प (c) सही उत्तर है ।
2. पौधों जैसे- कपास, धनिया एवं लार्कस्पर में विभिन्न आकार की पत्तियों में पत्तियों का आकार किशोरावस्था एवं परिपक्व अवस्था में भिन्न होता है। विषमपर्णता का यह दृश्य उदाहरण है-
(a) प्लास्टिसिटी (Plasticity)
(b) शिखाग्र प्रधान्यता (Apical dominance)
(c) प्रत्यास्थता (Elasticity)
(d) विभेदन (Differentiation)
उत्तर : (a)
व्याख्या: पौधे पर्यावरण के प्रभाव के कारण या जीवन के विभिन्न चरणों में भिन्न पथों का अनुसरण करते हैं, ताकि विभिन्न तरह की संरचनाओं का गठन कर सकें, इस क्षमता को ‘प्लास्टिसिटी’ (Plasticity) कहते हैं।
• उदाहरण के तौर पर कपास, धनिया एवं लार्कस्पर (Larkspur) में विभिन्न आकार की पत्तियों में पत्तियों का आकार किशोरावस्था एवं परिपक्व अवस्था में भिन्न होता है।
• दूसरी तरफ बटरकप (Buttercup) में पत्तियों का आकार वायवीय भागों में अलग होता है। विषमपर्णता का यह दृश्य प्लास्टिकता (Plasticity) का एक उदाहरण है।
अतः विकल्प (a) सही उत्तर है।
3. परिवर्धन (Development) के संदर्भ में निम्नलिखित कथनों पर विचार कीजिये :
1. परिवर्धन (Development) को सामान्यतः वृद्धि एवं विभेदन के योग के रूप में माना जाता है।
2. पौधों में परिवर्धन केवल बाह्य कारकों के नियंत्रण में है ।
3. प्रकाश, ताप एवं जल आदि आंतरिक कारकों के अंतर्गत आते हैं।
उपर्युक्त कथनों में से कौन-सा/से सही है/हैं?
(a) केवल 1
(b) केवल 2
(c) केवल 1 और 3
(d) केवल 1 और 2
उत्तर : (a)
व्याख्या : एक पौधे के जीवन में वृद्धि, विभेदन और परिवर्धन बहुत निकट संबंध रखने वाली घटनाएँ हैं। व्यापक तौर पर या सामान्यतः परिवर्धन को वृद्धि एवं विभेदन के योग के रूप में माना जाता है। अतः कथन 1 सही है।
• पौधों में परिवर्धन अर्थात् वृद्धि एवं विभेदन दोनों आंतरिक एवं बाह्य कारकों दोनों से नियंत्रित होता है। अतः कथन 2 गलत है।
• आंतरिक कारकों के अंतर्गत अंतर – कोशिकीय आनुवंशिक एवं अंतर- कोशिकीय कारक (जैसे कि पादप वृद्धि नियामक रसायन) शामिल होते हैं, जबकि बाह्य कारकों के अंतर्गत प्रकाश, तापक्रम, जल, ऑक्सीजन तथा पोषक तत्त्व आदि शामिल होते हैं। अतः कथन 3 गलत है।
अतः विकल्प (a) सही उत्तर है ।
4. ‘बैकेन’ (फूलिश सीडलिंग) धान के पौधे की बीमारी है, यह किसके द्वारा होती है?
(a) जीवाणु
(b) कवक
(c) विषाणु
(d) प्रोटोजोआ
उत्तर : (b)
व्याख्या: ‘बैकेन’ (फूलिश सीडलिंग) धान के पौधे की बीमारी है, जो रोगजनक कवक जिबरेला फ्यूजीक्यूरॉई के द्वारा होती है। ई. क्यूरोसोवा (जापानी वैज्ञानिक) ने रोगरहित धान के पौधे में रोग के लक्षण को बताया, तब उन्हें कवक के जीवाणुहीन निस्यंदों (फिल्ट्रेट) के साथ उपचारित किया। बाद में सक्रिय तत्त्व की पहचान जिबरेलिक अम्ल के रूप में हुई।
अतः विकल्प (b) सही उत्तर है ।
5. निम्नलिखित में से कृत्रिम ऑक्सिस हैं-
1. इनडोल ब्यूटरिक अम्ल (IBA)
2. नैफ्थलिन एसीटिक अम्ल (NAA)
3. 2, 4 डाइक्लोरो फिनॉक्सी एसीटिक अम्ल (2, 4 डी)
उपर्युक्त में से कौन-सा/से कथन सही है/हैं?
(a) केवल 1 और 2
(b) केवल 2
(c) 1, 2 और 3
(d) केवल 2 और 3
उत्तर : (d)
व्याख्या : ऑक्सिस शब्द इनडोल – 3 एसीटिक अम्ल (IAA) तथा अन्य प्राकृतिक एवं कृत्रिम यौगिक, जिसमें वृद्धि करने की क्षमता हो, के लिये प्रयोग किया जाता है।
• ये प्रायः तने एवं मूल के बढ़ते हुए शिखर पर बनते हैं तथा वहाँ से क्रियाशीलता वाले भाग में जाते हैं।
• ऑक्सिंस जैसे आई. ए. ए. एवं इनडोल ब्यूटरिक अम्ल पौधे से निकाला गया है। एन.ए.ए. (नैफ्थलिन एसीटिक अम्ल) तथा 2,4 डी (2,4 डाइक्लोरो फिनॉक्सी एसीटिक अम्ल) कृत्रिम ऑक्सिस हैं।
अतः विकल्प (d) सही उत्तर है।
6. निम्नलिखित कथनों पर विचार कीजिये :
1. ऑक्सिस तनों की कटिंग (स्टेम कटिंग) में जड़ फूटने (रूटिंग) में सहायता करते हैं।
2. ऑक्सिस पुष्पन ( फ्लॉवरिंग) को घटा देते हैं।
3. इसका उपयोग मालियों के द्वारा लॉन को तैयार करने में किया जाता है।
उपर्युक्त कथनों में से कौन-सा/से सही है/हैं?
(a) केवल 2
(b) केवल 1 और 3
(c) केवल 2 और 3
(d) 1, 2 और 3
उत्तर : (b)
व्याख्या : ऑक्सिस (Auxins ) का उपयोग विस्तृत है और ये बागवानी एवं खेती में प्रयोग किये गए हैं। ये तनों की कटिंग में जड़ फूटने (Rooting) सहायता करते हैं जो पादप प्रवर्धन में व्यापकता से इस्तेमाल होते हैं। अतः कथन 1 सही है।
• ऑक्सिंस पुष्पन (Flowering) को बढ़ा देते हैं जैसे- अनानास में। अतः कथन 2 गलत है।
• ऑक्सिस का व्यापक रूप से प्रयोग शाकनाशी के रूप में किया जाता है । जैसे- 2,4 डी, व्यापक रूप से द्विबीजपत्ती खरपतवारों का नाश कर देता है; लेकिन एकबीजपत्ती परिपक्व पौधों को प्रभावित नहीं करता है। इसका उपयोग मालियों के द्वारा लॉन को तैयार करने में किया जाता है। इसके साथ ही ऑक्सिंस जाइलम विभेदन को नियंत्रित करने तथा कोशिका के विभाजन में मदद करता है। अतः कथन 3 सही है।
अतः विकल्प (b) सही उत्तर है।
7. फलों को पकाने में उपयोगी है-
(a) एथीलीन
(b) एब्सीसिक एसिड
(c) ऑक्सिंस (Auxins)
(d) इनमें से कोई नहीं
उत्तर : (a)
व्याख्या : एथीलीन एक साधारण गैसीय पादप वृद्धि नियामक है, यह जरावस्था को प्राप्त होते ऊतकों तथा पकते हुए फलों के द्वारा भारी मात्रा में संश्लेषित की जाती है। यह फलों को पकाने में बहुत प्रभावी है। फलों के पकने के दौरान यह श्वसन की गति में वृद्धि करता है, श्वसन वृद्धि में गति की इस बढ़त को ‘क्लाइमैक्टिक श्वसन’ कहते हैं।
अतः विकल्प (a) सही उत्तर है।
8. निम्नलिखित में से ‘ दीप्तिकालिता’ (Photoperiodism) के संबंध में सर्वाधिक उपयुक्त विकल्प कौन-सा है ?
(a) इस घटना के अंतर्गत पौधों में पुष्पन सिर्फ प्रकाश और अंधकार की अवधि पर निर्भर करता है।
(b) सभी पौधों में पुष्पन की निर्भरता केवल प्रकाश पर है।
(c) सभी पौधों में पुष्पन की निर्भरता केवल अंधकार पर है।
(d) कुछ पौधों में पुष्पन की निर्भरता केवल प्रकाश और अंधकार पर नहीं बल्कि उसकी सापेक्षिक अवधि पर भी होती है।
उत्तर : (d)
व्याख्या: कुछ पौधों में पुष्पन सिर्फ प्रकाश और अंधकार पर ही निर्भर नहीं करता बल्कि उसकी सापेक्षिक अवधि पर भी निर्भर करता है, इस घटना को दीप्तिकालिता कहते हैं।
अतः विकल्प (d) सही उत्तर है ।

16. पाचन एवं अवशोषण

1. मुखगुहा (Mouth Cavity) के संबंध में निम्नलिखित कथनों पर विचार कीजिये :
1. मुखगुहा एक छोटी ग्रसनी में खुलती है, जो केवल भोजन का ही पथ है।
2. इसके अंतर्गत जिह्वा स्वतंत्र रूप घूमने योग्य एक पेशीय अंग है, जो फ्रेनुलम (Frenulum) द्वारा मुखगुहा के आधार से जुड़ी होती है।
3. मुखगुहा में पाचन की रासायनिक प्रक्रिया कार्बोहाइड्रेट को जल अपघटित करने वाले एंजाइम टायलिन या लार एमाइलेज की सक्रियता से प्रारंभ होती है।
उपर्युक्त कथनों में से कौन-सा/से सही है/हैं?
(a) केवल 1 और 3
(b) केवल 3
(c) केवल 2 और 3
(d) केवल 2
उत्तर : (c)
व्याख्या: मुखगुहा एक छोटी ग्रसनी में खुलती है जो वायु एवं भोजन दोनों का पथ है, न केवल भोजन का। अतः कथन 1 गलत है।
• मुखगुहा में कई दाँत और एक पेशीय जिह्वा होती है। जिह्वा स्वतंत्र रूप से घूमने योग्य एक पेशीय अंग है, जो फ्रेनुलम (Frenulum) द्वारा मुखगुहा के आधार से जुड़ी होती है। अतः कथन 2 सही है।
• पाचन की प्रक्रिया यांत्रिक एवं रासायनिक विधियों द्वारा संपन्न होती है । मुखगुहा के मुख्यतः दो प्रकार्य हैं. भोजन का चवर्ण और निगलने की क्रिया। पाचन की रासायनिक प्रक्रिया मुखगुहा में कार्बोहाइड्रेट को जल अपघटित करने वाले एंजाइम टायलिन या लार एमाइलेज की सक्रियता से प्रारंभ होती है। अतः कथन 3 सही है।
अतः विकल्प (c) सही उत्तर है।
2. यकृत के संदर्भ में असत्य कथन चुनिये :
(a) यकृत मानव शरीर की सबसे बड़ी ग्रंथि है।
(b) इसका वयस्क मनुष्य में भार 4 से 5 किलोग्राम होता है।
(c) यह उदर में मध्यपट के ठीक नीचे स्थित होता है।
(d) यकृत की कोशिकाओं से पित्त का स्राव होता है।
उत्तर : (b)
व्याख्या: यकृत (Liver) मनुष्य के शरीर की सबसे बड़ी ग्रंथि है, जिसका वयस्क में भार लगभग 1.2 से 1.5 किलोग्राम होता है। अतः विकल्प (b) असत्य है।
• यकृत उदर में मध्यपट के ठीक नीचे स्थित होता है और इसकी दो पालियाँ (Lobes) होती हैं।
• यकृत की कोशिकाओं में पित्त का स्राव होता है, जो यकृत नलिका से होते हुए एक पतली पेशीय थैली पित्ताशय में सांद्रित एवं जमा होता है।
अतः विकल्प (b) सही उत्तर है।
3. निम्नलिखित कथनों पर विचार कीजिये :
1. अग्न्याशय केवल अंत: स्रावी ग्रंथि की तरह कार्य करता है।
2. आमाशय की पेशीय दीवार के संकुचन द्वारा भोजन अम्लीय जठर रस से पूरी तरह मिल जाता है, जिसे काइम (Chyme ) कहते हैं।
3. पित्त वसा के इमल्सीकरण में मदद करता है और उसे बहुत छोटे कणों में तोड़ता है।
4. नवजातों के जठर रस में रेनिन नामक प्रोटीन अपघटनीय एंजाइम है।
उपर्युक्त कथनों में से कौन-सा/से सही है/हैं?
(a) केवल 2, 3 और 4
(b) केवल 1 और 4
(c) केवल 1, 2 और 3
(d) उपर्युक्त सभी।
उत्तर : (a)
व्याख्या : अग्न्याशय ‘C’ आकार की ग्रहणी के बीच स्थित एक लंबी ग्रंथि है जो बहि: स्रावी और अंत: स्रावी दोनों ही ग्रंथियों की तरह कार्य करता है । बहि: स्रावी भाग से क्षारीय अग्न्याशयी स्राव निकलता है, जिसमें एंजाइम होते हैं और अंत: स्रावी भाग से इंसुलिन एवं ग्लूकागॉन नामक हार्मोन का स्राव होता है। अतः कथन 1 गलत है।
• आमाशय 4-5 घंटे तक भोजन का संग्रहण करता है । आमाशय की पेशीय दीवार के संकुचन द्वारा भोजन अम्लीय जठर रस पूरी तरह मिल जाता है जिसे काइम (Chyme) कहते हैं। अत: कथन 2 सही है।
• ग्रहणी (Duodenum) में प्रवेश करने वाले पित्त में पित्त वर्णक, पित्त लवण, कोलेस्टेरॉल एवं फास्फोलिपिड होते हैं, लेकिन कोई एंजाइम नहीं होता। पित्त वसा के इमल्सीकरण में मदद करता है और उसे बहुत छोटे महीन कणों में तोड़ता है तथा पित्त लाइपेज एंजाइम को भी सक्रिय करता है। अतः कथन 3 सही है।
• नवजातों के जठर रस में रेनिन नामक प्रोटीन अपघटनीय एंजाइम होता है, जो दूध के प्रोटीन को पचाने में सहायक होता है। जठर ग्रंथियाँ थोड़ी मात्रा में लाइपेज भी स्रावित करती हैं। अतः कथन 4 सही है।
अतः विकल्प (a) सही उत्तर है।
4. जल एवं एल्कोहॉल का अवशोषण निम्नलिखित में से कहाँ होता है?
(a) छोटी आँत
(b) बड़ी आँत
(c) आमाशय
(d) मुख
उत्तर : (c)
5. ‘मरास्मस’ के संदर्भ में, निम्नलिखित कथनों पर विचार कीजिये :
1. यह केवल प्रोटीन की अल्पता से उत्पन्न होता है।
2. यह विकार सामान्यतः 1 वर्ष से कम आयु के शिशुओं में पाया जाता है।
3. इसका मुख्य कारण प्रायः कम अंतराल में पुनः गर्भधारण अथवा शिशु का जन्म है।
4. इसमें वृद्धि की दर एवं शारीरिक भार बहुत अधिक घट जाता है।
उपर्युक्त में से कौन-से कथन सत्य हैं?
(a) केवल 2, 3 और 4
(b) केवल 2 और 3
(c) केवल 1, 2 और 3
(d) 1, 2, 3 और 4
उत्तर : (a)
व्याख्या: ‘मरास्मस’ केवल प्रोटीन से नहीं बल्कि प्रोटीन और कैलोरी दोनों की एक साथ अल्पता से उत्पन्न होता है। अतः कथन 1 असत्य है ।
• यह विकार सामान्यतः 1 वर्ष कम आयु के शिशुओं में पाया जाता है। इसका मुख्य कारण शिशु को माँ के दूध के स्थान पर अल्प प्रोटीन और कम कैलोरी मान वाले आहार को देना है। अतः कथन 2 सत्य है।
• इसका मुख्य कारण प्रायः कम अंतराल में पुनः गर्भधारण अथवा शिशु का जन्म होना है, जबकि बड़ा बच्चा बहुत कम आयु का ही होता है। अतः कथन 3 सत्य है ।
• मरास्मस में प्रोटीन अल्पता के कारण वृद्धि मंद तथा ऊतकों की प्रोटीन का विस्थापन अत्यंत कमजोर शरीर एवं हाथ-पैर अत्यंत पतले हो जाते हैं; साथ ही त्वचा शुष्क, पतली एवं झुर्रीदार हो जाती है।
• वृद्धि की दर एवं शारीरिक भार अत्यधिक घट जाता है। मस्तिष्क की वृद्धि एवं विकास भी अति मंद हो जाता है। मानसिक क्षमता भी असंतुलित हो जाती है।
अतः विकल्प (a) सही उत्तर है।
6. ‘क्वाशिओरकर’ के संदर्भ में निम्नलिखित में से असत्य कथन की पहचान कीजिये।
(a) यह प्रोटीन की अल्पता से उत्पन्न होने वाला विकार है।
(b) यह 1 वर्ष से कम आयु के बच्चों जिनका पोषण माँ के दूध द्वारा होता है, में होता है।
(c) इसमें भी मरास्मस की तरह हाथ-पैर पतले हो जाते हैं।
(d) यह उच्च कैलोरी परंतु अल्प प्रोटीन वाला आहार देने से होता है।
उत्तर : (b)
व्याख्या: ‘क्वाशिओरकर’ प्रोटीन अल्पता से उत्पन्न विकार है। अतः विकल्प (a) सत्य है।
• यह 1 वर्ष से कम आयु के बच्चों में जिनका पोषण माँ के दूध द्वारा होता है, उनमें नहीं होता है। अतः विकल्प (b) असत्य है।
• मरास्मस की तरह ही क्वाशिओरकर में भी माँसपेशियाँ लटक जाती हैं, हाथ-पैर पतले हो जाते हैं तथा वृद्धि एवं मस्तिष्क का विकास रुक जाता है। परंतु मरास्मस के विपरीत त्वचा के नीचे कुछ वसा शेष रहती है, परंतु शरीर के विभिन्न भागों में सूजन दृष्टिगोचर होती है।
• यह उच्च कैलोरी परंतु अल्प प्रोटीन वाला आहार देने से भी होता है। अतः विकल्प (b) सही उत्तर है।

17. श्वसन और गैसों का विनिमय

1. मानव श्वसन तंत्र के अंतर्गत ग्रसनी के संबंध में निम्नलिखित में से कौन-सा कथन असत्य है ?
(a) ग्रसनी केवल वायु का मार्ग है।
(b) ग्रसनी वायु एवं आहार दोनों के लिये उभयनिष्ट मार्ग है।
(c) मनुष्यों में यह नम ऊतक के साथ एक खोखली संरचना होती है।
(d) इनमें से कोई नहीं
उत्तर : (a)
व्याख्या: मानव शरीर में एक जोड़ी बाह्य नासिका द्वार होते हैं, जो होंठों के ऊपर बाहर की तरफ खुलते हैं। ये नासिका मार्ग द्वारा नासिका कक्ष तक पहुँचते हैं। नासिका कक्ष ग्रसनी में खुलते हैं।
• ग्रसनी आहार एवं वायु दोनों के लिये उभयनिष्ठ मार्ग है।
• ग्रसनी कंठ द्वारा श्वासनली में खुलती है।
• यह नम ऊतक के साथ एक खोखली संरचना होती है।
अतः विकल्प (a) सही उत्तर है।
2. निम्नलिखित में से किनमें श्वसन क्लोम (गिल्स) द्वारा होता है ?
1. जलीय आर्थ्रोपोडा तथा मोलस्का में
2. सरीसृप में
3. मछलियों में
4. स्तनधारी में
उपर्युक्त में से कौन-से सही हैं?
(a) केवल 1 और 2
(b) केवल 2, 3 और 4
(c) केवल 1 और 3
(d) 1, 2, 3 व 4 सभी
उत्तर : (c)
व्याख्या : जलीय आर्थ्रोपोडा तथा मोलस्का में श्वसन विशेष संहनीय संरचना क्लोम (गिल्स) द्वारा होता है, जबकि स्थलचर प्राणियों में श्वसन विशेष संवहनीय थैली फुफ्फुस / फेफड़ों द्वारा होता है।
• कशेरूकी में मछलियाँ क्लोम (गिल्स) द्वारा श्वसन करती हैं जबकि एम्फीबिया (उभयचर), सरीसृप, पक्षी और स्तनधारी फेफड़ों द्वारा श्वसन करते हैं।
अतः विकल्प (c) सही उत्तर है।
3. निम्नलिखित में से फेफड़ों (lungs) की रचना होती है-
(a) श्वसनी (Bronchi), श्वसनिकाओं (Bronchioles) और कूपिकाओं (Alveoli) के शाखित जाल से
(b) डायाफ्राम (Diaphragm ), श्वसनी (Bronchi) एवं कूपिकाओं (Alveoli) के शाखित जाल से
(c) श्वासनली (Trachea ), श्वसनिकाओं (Bronchioles) और श्वसनी (Bronchi) के शाखित जाल से
(d) कूपिकाओं (Alveoli), श्वासनलियों (Treacheas) और श्वसनिकाओं (Bronchioles) के शाखित जाल से
उत्तर : (a)
व्याख्या: मानव श्वसन तंत्र के अंतर्गत फेफड़े की रचना श्वसनी (Bronchi), श्वसनिकाओं (Bronchioles) एवं कूपिकाओं ( Alveoli) के शाखित जाल द्वारा होती है।
• हमारे शरीर में दो फेफड़े हैं जो एक द्विस्तरीय फुफ्फुसावरण (Pleura) से ढके रहते हैं, जिनके बीच फुफ्फुसावरणी द्रव भरा होता है। यह फेफड़े की सतह पर घर्षण कम करता है।
• बाहरी फुफ्फुसावरणी झिल्ली वक्षीय परत के निकट संपर्क में रहती है, जबकि आंतरिक फुफ्फुसावरणी झिल्ली फेफड़े की सतह के संपर्क में होती है।
अतः विकल्प (a) सही उत्तर है।
4. निम्नलिखित कथनों पर विचार कीजिये :
1. अंत: श्वसन की शुरुआत डायाफ्राम के संकुचन से होती है ।
2. औसतन एक स्वस्थ मनुष्य प्रति मिनट 12-16 बार श्वसन करता है।
3. श्वसन गतिविधियों में सम्मिलित वायु के आयतन का आकलन स्पाइरोमीटर की सहायता से किया जा सकता है।
उपर्युक्त कथनों में से कौन-सा/से सही है/हैं?
(a) केवल 2
(b) केवल 1 और 3
(c) 1, 2 और 3
(d) केवल 1 और 2
उत्तर : (c)
व्याख्या : श्वसन में दो चरण सम्मिलित हैं, पहला अंतःश्वसन जिसके दौरान वायुमंडलीय वायु को अंदर खींचा जाता है और दूसरा निःश्वसन जिसके द्वारा फुफ्फुसी वायु को बाहर मुक्त किया जाता है।
• अंतःश्वसन डायाफ्राम के संकुचन से प्रारंभ होता है जो डॉर्सो वेन्ट्रल अक्ष (Dorso ventral axis) में वक्ष-गुहा कक्ष का आयतन बढ़ा देता है। अतः कथन 1 सही है ।
• औसतन एक स्वस्थ मनुष्य प्रति मिनट 12-16 बार श्वसन करता है। तथा श्वसन गतिविधियों में सम्मिलित वायु के आयतन का आकलन स्पाइरोमीटर की सहायता से किया जा सकता है जो फुफ्फुसी क्रियाकलापों का नैदानिक मूल्यांकन करने में सहायक होता है।
अतः कथन 2 और 3 सही हैं।
5. निम्नलिखित कथनों पर विचार कीजिये :
1. सामान्य निःश्वसन उपरांत वायु की वह मात्रा जो फेफड़ों में शेष रह जाती है, क्रियाशील अवशिष्ट क्षमता कहलाती है।
2. वायु का वह आयतन जो बलपूर्वक निःश्वसन के बाद भी फेफड़ों में शेष रह जाता है, निःश्वसन सुरक्षित आयतन कहलाता है।
उपर्युक्त कथनों में से कौन-सा / से सत्य नहीं है/हैं?
(a) केवल 1
(b) केवल 2
(c) 1 और 2 दोनों
(d) न तो 1, न ही 2
उत्तर : (b)
व्याख्या: क्रियाशील अवशिष्ट क्षमता (Functional Residual CapactiyFRC) सामान्य निःश्वसन उपरांत वायु की वह मात्रा (आयतन) है, जो फेफड़ों में शेष रह जाती है। इसमें निःश्वसन सुरक्षित आयतन (ERV) और अवशिष्ट आयतन ( RV) सम्मिलित होते हैं (ERV + RV)। अत: कथन 1 सत्य है। T
• वायु का वह आयतन जो बलपूर्वक निःश्वसन के बाद भी फेफड़ों में शेष रह जाता है, अवशिष्ट आयतन ( Residual Volume – RV) कहलाता है। अतः कथन 2 असत्य है।
• वायु आयतन की वह अतिरिक्त मात्रा जो एक व्यक्ति बलपूर्वक निःश्वासित कर सकता है, निःश्वसन सुरक्षित आयतन (Expiratory Reserve Volume—ERV) कहलाता है।
अतः विकल्प (b) सही उत्तर है।
6. निम्नलिखित में से किसके श्वसन में विशिष्टता पाई जाती है?
(a) मेंढक
(b) पक्षी
(c) सरीसृप
(d) स्तनधारी
उत्तर : (a)
व्याख्या: कशेरुकियों में मछलियाँ क्लोम (गिल्स) द्वारा श्वसन करती हैं जबकि एम्फीबिया (उभयचर), सरीसृप, पक्षी और स्तनधारी फेफड़ों द्वारा श्वसन करते हैं।
• लेकिन उभयचर जैसे- मेंढक अपनी आर्द्र (नम) त्वचा द्वारा भी श्वसन कर सकते हैं। इसलिये मेंढक के श्वसन में विशिष्टता पाई जाती है।
अतः विकल्प (a) सही उत्तर है।
7. निम्नलिखित में से कौन – सा / से अंग वक्ष – गुहा में स्थित होता / होते है/हैं?
1. हृदय
2. फेफड़ा
3. अग्न्याशय
नीचे दिये गए कूट का प्रयोग कर सही उत्तर चुनिये :
(a) केवल 2
(b) केवल 1
(c) 1 और 3
(d) 1 और 2
उत्तर : (d)
व्याख्या : फेफड़े वक्ष – गुहा में स्थित होते हैं जो एक वायु रोधी कक्ष है। वक्ष – गुहा कक्ष पृष्ठ भाग में कशेरूक दंड, अधर भाग में उरोस्थि (Sternum), पार्श्व में पसलियों और नीचे से गुंबदाकार डायाफ्राम (Diaphragm) द्वारा बनता है ।
• हृदय की उत्पत्ति मध्यजन स्तर ( मीसोडर्म) से होती है तथा यह दोनों फेफड़ों के मध्य वक्ष गुहा में स्थित होता है। यह थोड़ा सा बाईं तरफ झुका रहता है।
• अग्न्याशय ‘C’ आकार के ग्रहणी ( Duodenum) के बीच स्थित एक लंबी ग्रंथि है, जो बहि: स्रावी और अंत: स्रावी दोनों ही ग्रंथियों की तरह कार्य करती है।
अतः विकल्प (d) सही उत्तर है।

18. शरीर द्रव तथा परिसंचरण

1. प्लाज़्मा संदर्भ में निम्नलिखित कथनों पर विचार कीजिये :
1. प्लाज़्मा एक हल्के पीले रंग का गाढ़ा तरल पदार्थ है।
2. यह रक्त के आयतन का 10% होता है।
3. प्लाज़्मा में 90-92 प्रतिशत जल तथा 6-8 प्रतिशत प्रोटीन पदार्थ होते हैं।
उपर्युक्त कथनों में से कौन-सा/से सही है/हैं?
(a) केवल 2 और 3
(b) 1, 2 और 3
(c) केवल 1 और 2
(d) केवल 1 और 3
उत्तर : (d)
व्याख्या : रक्त एक विशेष प्रकार का ऊतक है, जिसमें द्रव्य आधात्री ( मैट्रिक्स), प्लाज़्मा एवं अन्य संगठित संरचनाएँ पाई जाती हैं।
• प्लाज़्मा एक हल्के पीले रंग का गाढ़ा तरल पदार्थ है, जो रक्त के आयतन का लगभग 55% होता है। अतः कथन 1 सही है एवं कथन 2 गलत है।
• प्लाज़्मा में 90-92 प्रतिशत जल तथा 6-8% प्रोटीन पदार्थ होते हैं।
• फाइब्रिनोजन, ग्लोबुलिन तथा एल्बुमिन प्लाज़्मा में उपस्थित मुख्य प्रोटीन हैं। अतः विकल्प (d) सही उत्तर है ।
2. निम्नलिखित कथनों पर विचार कीजिये :
1. इरिथ्रोसाइट्स, ल्यूकोसाइट्स एवं प्लेटलेट्स तीनों संगठित पदार्थ मिलकर रक्त का लगभग 80% भाग बनाते हैं।
2. इरिथ्रोसाइट्स या लाल रक्त कणिकाएँ अन्य सभी कोशिकाओं से संख्या में अधिक होती हैं।
3. लिंफोसाइट्स में केवल B प्रकार की लिंफोसाइट्स शरीर की प्रतिरक्षा के लिये उत्तरदायी हैं।
उपर्युक्त कथनों में से कौन – सा / से सही नहीं है/हैं?
(a) केवल 3
(b) केवल 1
(c) केवल 1 और 3
(d) केवल 2 और 3
उत्तर : (c)
व्याख्या : लाल रक्त कणिकाएँ (इरिथ्रोसाइट्स), श्वेत रक्त कणिकाएँ (ल्यूकोसाइट्स) तथा प्लेटलेट्स को संयुक्त रूप से संगठित पदार्थ कहते हैं और यह रक्त का लगभग 45% भाग बनाते हैं। अतः कथन 1 गलत है।
• इरिथ्रोसाइट्स या लाल रक्त कणिकाएँ अन्य सभी कोशिकाओं से संख्या में अधिक होती हैं। वयस्क अवस्था में ये अस्थि मज्जा में बनती हैं। अधिकतर स्तनधारियों में लाल रक्त कणिकाओं में केंद्रक नहीं मिलते हैं। अतः कथन 2 सही है।
• लिंफोसाइट्स मुख्यत: B तथा T प्रकार की होती हैं। दोनों प्रकार की लिंफोसाइट्स शरीर की प्रतिरक्षा के लिये उत्तरदायी हैं न कि केवल B प्रकार की । अतः कथन 3 गलत है।
अतः विकल्प (c) सही उत्तर है।
3. परिसंचरण तंत्र के संबंध में निम्नलिखित में से कौन-सा कथन असत्य है?
(a) आर्थ्रोपोडा तथा मोलस्का दोनों में खुला परिसंचरण तंत्र पाया जाता है।
(b) ऐनेलिडा में बंद परिसंचरण तंत्र तथा कशेरूकी में खुला परिसंचरण तंत्र पाया जाता है।
(c) बंद परिसंचरण तंत्र खुले परिसंचरण तंत्र की अपेक्षा अधिक लाभदायक होता है।
(d) सभी कथन सत्य हैं।
उत्तर : (b)
4. निम्नलिखित में से किसके / किनके हृदय में चार कक्ष होते हैं?
1. मछलियाँ
2. मगरमच्छ
3. उभयचर
4. पक्षी
5. मनुष्य
नीचे दिये गए कूट के आधार पर सही विकल्प चुनिये :
(a) केवल 1, 2 और 4
(b) केवल 2, 4 और 5
(c) केवल 2, 3, 4 और 5
(d) 1, 2, 3, 4 और 5
उत्तर : (b)
व्याख्या: सभी कशेरूकी में कक्षों से बना हुआ पेशी हृदय होता है। मछलियों में दो कक्षीय हृदय होता है, जिसमें एक आलिंद (Atria) तथा एक निलय (Ventricle) होता है।
• उभयचरों तथा सरीसृपों (अपवाद – मगरमच्छ) का हृदय तीन कक्षों से बना होता है, जिसमें दो आलिंद तथा एक निलय होता है। मगरमच्छ का हृदय चार कक्षों का बना होता है। जिसमें दो आलिंद तथा दो निलय होते हैं।
• पक्षियों तथा स्तनधारियों में हृदय चार कक्षों का बना होता है जिसमें दो आलिंद तथा दो निलय होते हैं।
अतः विकल्प (b) सही उत्तर है।
5. मानव हृदय के संबंध में निम्नलिखित में से किसे गतिप्रेरक (पेशमेकर) कहते हैं ?
(a) आलिंद – निलय नोड (Atrio ventricular Node)
(b) शिरा – आलिंद नोड ( Sino-Atrial Node – SAN)
(c) आलिंद (Atria)
(d) निलय (Ventricles)
उत्तर : (b)
व्याख्या : शिरा – आलिंद नोड (SAN) सबसे अधिक क्रिया विभव पैदा कर सकता है। यह एक मिनट में 70-75 क्रियाविभव पैदा करता है तथा हृदय का लयात्मक संकुचन ( Rhythmic Contractile) प्रारंभ कर उसे बनाए रखता है, इसलिये इसे गतिप्रेरक (पेशमेकर) कहते हैं। इससे हमारी सामान्य हृदय स्पंदन दर 70-75 प्रति मिनट होती है ( औसतन 72 स्पंदन प्रति मिनट ) |
• हमारे हृदय में चार कक्ष होते हैं जिसमें दो कक्ष अपेक्षाकृत बड़े होते हैं जिन्हें निलय (Ventricle) कहते हैं तथा दो कक्ष अपेक्षाकृत छोटे तथा ऊपर की ओर पाए जाते हैं जिन्हें आलिंद (Atria) कहा जाता है।
अतः विकल्प (b) सही उत्तर है।
6. इलेक्ट्रोकार्डियोग्राम (ECG) के संदर्भ में निम्नलिखित कथनों पर विचार कीजिये?
1. ईसीजी हृदय के हृदयी चक्र की विद्युत क्रियाकलापों का आरेखीय प्रस्तुतीकरण है।
2. ईसीजी के प्रत्येक चर्मोत्कर्ष को P से T तक दर्शाया जाता है।
3. T तरंग निलय का उत्तेजना से सामान्य अवस्था में वापस आने की स्थिति को प्रदर्शित करता है।
उपर्युक्त कथनों में से कौन-से सही हैं?
(a) केवल 1 और 3
(b) केवल 2 और 3
(c) केवल 1 और 2
(d) 1, 2 और 3
उत्तर : (d)
व्याख्या: इलेक्ट्रोकार्डियोग्राफ का उपयोग विद्युत हृद लेख (इलेक्ट्रोकार्डियो ग्राम – ईसीजी) प्राप्त करने के लिये किया जाता है।
• ईसीजी हृदय के हृदयी चक्र की विद्युत क्रियाकलापों का आरेखीय प्रस्तुतीकरण है। अतः कथन 1 सही है।
• ईसीजी के प्रत्येक चर्मोत्कर्ष को P से T तक दर्शाया जाता है, जो हृदय की विशेष विद्युत क्रियाओं को प्रदर्शित करता है। P तरंग को आलिंद के उद्दीपन / विधुवण (Electrical Excitation or Depolarisation) के रूप में प्रस्तुत किया जाता है।
• Q, R तथा S सम्मिश्र निलय के अध्रुवण (Depolarisation of Ventricles) को प्रस्तुत करता है जो निलय के संकुचन को प्रारंभ करता है।
• T तरंग, निलय का उत्तेजना से सामान्य अवस्था में वापस आने की स्थिति को प्रदर्शित करता है। T तरंग अंतः प्रकुंचन अवस्था की समाप्ति का द्योतक है। अतः कथन 2 और कथन 3 सही हैं। अतः विकल्प (d) सही उत्तर है।

19. उत्सर्जी उत्पाद एवं उनका निष्कासन

1. निम्नलिखित कथनों पर विचार कीजिये :
1. अमोनोटेलिक जीवों में उत्सर्जन वृक्कों द्वारा निस्यंदन के पश्चात् संपन्न होता है।
2. यूरियोटेलिक जीवों के उत्सर्जन में वृक्कों की कोई अहम भूमिका नहीं होती है।
3. यूरिकोटेलिक जीवों में उत्सर्जन जल की कम मात्रा के साथ गोलिकाओं या पेस्ट के रूप में होता है।
उपर्युक्त कथनों में से कौन-सा/से सही है/हैं?
(a) केवल 3
(b) केवल 2 और 3
(c) 1, 2 और 3
(d) केवल 1
उत्तर : (a)
व्याख्या : अमोनिया के उत्सर्जन की प्रक्रिया को अमोनियोत्सर्ग प्रक्रिया कहते हैं। अनेक अस्थिल मछलियाँ, उभयचर और जलीय कीट अमोनिया उत्सर्जी प्रकृति के हैं। अमोनिया सरलता से घुलनशील है, इसलिये आसानी से अमोनियम आयनों के रूप में शरीर की सतह या मछलियों के क्लोम (गिल्स) की सतह से विसरण द्वारा उत्सर्जित हो जाता है। इस उत्सर्जन में वृक्क की कोई अहम भूमिका नहीं होती है। इन प्राणियों को अमोनियाउत्सर्जी (अमोनोटेलिक) कहते हैं। अत: कथन 1 गलत है।
• स्तनधारी, कई स्थली उभयचर और समुद्री मछलियाँ मुख्यतः यूरिया का उत्सर्जन करते हैं और यूरियाठत्सर्जी (यूरियोटेलिक) कहलाते हैं। इन प्राणियों में उपापचयी क्रियाओं द्वारा निर्मित अमोनिया को यकृत द्वारा यूरिया में परिवर्तित कर रक्त से मुक्त कर दिया जाता है, जिसे वृक्कों द्वारा निस्यंदन के पश्चात् उत्सर्जित कर दिया जाता है। अतः कथन 2 गलत है।
• सरीसृपों, पक्षियों, स्थलीय घोंघों तथा कीटों में नाइट्रोजनी अपशिष्ट यूरिक अम्ल का उत्सर्जन, जल की कम मात्रा के साथ गोलिकाओं या पेस्ट के रूप में होता है और ये यूरिक अम्ल उत्सर्जी (यूरिकोटेलिक) कहलाते हैं। अतः कथन 3 सही है।
अतः विकल्प (a) सही उत्तर है।
2. मानव उत्सर्जन तंत्र के अंतर्गत वृक्क के संबंध में निम्नलिखित कथनों पर विचार कीजिये :
1. वृक्क (Kidney) सेम के बीज की आकृति के समान गहरे भूरे लाल रंग के होते हैं।
2. वयस्क मनुष्य में प्रत्येक वृक्क का भार लगभग 1-2 किग्रा. होता है।
उपर्युक्त कथनों में से कौन सा/से सही है/हैं?
(a) केवल 1
(b) केवल 2
(c) 1 और 2 दोनों
(d) न तो 1 और न ही 2
उत्तर : (a)
व्याख्या: मनुष्यों में उत्सर्जी तंत्र एक जोड़ी वृक्क एक जोड़ी मूत्र नलिका, एक मूत्राशय और एक मूत्रमार्ग का बना होता है।
• वृक्क सेम के बीज की आकृति के गहरे भूरे लाल रंग के होते हैं।
• वयस्क मनुष्य के प्रत्येक वृक्क की लंबाई 10-12 सेमी., चौड़ाई 5-7 सेमी मोटाई 2-3 सेमी. होती है। तथा भार लगभग 120 170 ग्राम होता है।
अतः विकल्प (a) सही उत्तर है।
3. वृक्कों (Kidney) द्वारा प्रति मिनट औसतन 1100-1200 मिली. रक्त का निस्यंदन किया जाता है जो कि-
(a) हृदय द्वारा एक मिनट में निकाले गए रक्त के बराबर होता है।
(b) हृदय द्वारा एक मिनट में निकाले गए रक्त का आधा होता है।
(c) हृदय द्वारा एक मिनट में निकाले गए रक्त का 1/5 वें भाग के बराबर होता है।
(d) हृदय द्वारा एक मिनट में निकाले गए रक्त के 1/4वें भाग के बराबर होता है।
उत्तर : (c)
व्याख्या : मूत्र निर्माण के प्रथम चरण में केशिकागुच्छ (Glomerulus) द्वारा रक्त का निस्यंदन होता है, जिसे गुच्छ या गुच्छीय निस्यंदन (Glomerular Filtration) कहते हैं।
• वृक्कों द्वारा प्रति मिनट औसतन 1100-1200 मिली. रक्त का निस्यंदन किया जाता है जो कि हृदय द्वारा एक मिनट में निकाले गए रक्त के 1 / 5वें भाग के बराबर होता है।
अतः विकल्प (c) सही उत्तर है।
4. हेनले – लूप के संदर्भ में निम्नलिखित में से कौन-सा कथन असत्य है?
(a) आरोही लिंब में न्यूनतम पुनरावशोषण होता है।
(b) हेनले लूप की अवरोही लिंब जल के लिये अपारगम्य होती है।
(c) वृक्काणु (Nephron ) में हेयर पिन के आकार का हेनले लूप पाया जाता है, जिसमें अवरोही एवं आरोही लिंब होते हैं।
(d) उपर्युक्त में से कोई नहीं
उत्तर : (b)
व्याख्या : नेफ्रॉन में हेयर पिन के आकार का हेनले लूप ( Henle’s Loop) पाया जाता है जिसमें अवरोही व आरोही लिंब होते हैं। हेनले लूप के आरोही लिंब में न्यूनतम पुनरावशोषण होता है।
• हेनले लूप की अवरोही लिंब जल के लिये पारगम्य होती है, किंतु विद्युत अपघट्य के लिये अधिकांशतः अपारगम्य होती है। अतः विकल्प (b) असत्य है।
• हेनले लूप की आरोही लिंब जल के लिये अपारगम्य होती है लेकिन विद्युत अपघट्य का परिवहन सक्रिय या निष्क्रिय रूप करती है।
• जैसे-जैसे सांद्र निस्यंद ( Filtrate) ऊपर की ओर जाता है, वैसे-वैसे विद्युत अपघट्य के मध्यांश तरल में जाने से निस्यंद (Filtrate) तनु (Dilute ) होता जाता है।
अतः विकल्प (b) सही उत्तर है ।
5. निम्नलिखित में से निस्यंद ( Filtrate) को सांद्रित करने में कौन महत्त्वपूर्ण भूमिका निभाता है/ निभाते हैं?
1. हेनले लूप (Henle’s loop)
2. वासा रेक्टा (Vasa recta)
3. संग्रह नलिका (Collecting Duct )
4. दूरस्थल संवलित नलिका (Distal Convoluted Tubule)
नीचे दिये गए कूट का प्रयोग कर सही उत्तर चुनिये :
(a) केवल 2 और 3
(b) 1, 2, 3 और 4
(c) केवल 1, 2 और 3
(d) केवल 1 और 3
उत्तर : (c)
व्याख्या: स्तनधारी सांद्रित मूत्र का उत्पादन करते हैं। इस कार्य में हेनले लूप और वासा रेक्टा महत्त्वपूर्ण भूमिका निभाते हैं। हेनले लूप की दोनों लिंबों में निस्यंद का विपरीत दिशाओं में प्रवाह होता है, जिससे प्रतिधारा उत्पन्न होती है।
• वासा रेक्टा की दोनों लिंबों में रक्त का बहाव भी प्रतिधारा प्रतिरूप (पैटर्न) में होता है।
• संग्रह नलिका (Collecting Duct ) एक लंबी नलिका है जो वृक्क के वल्कुट (Cortex) से मध्यांश (Medulla) के आंतरिक भाग तक फैली रहती है। मूत्र को आवश्यकतानुसार सांद्र करने के लिये जल का बड़ा हिस्सा इस भाग में अवशोषित किया जाता है। अतः यह मूत्र के सांद्रण में महत्त्वपूर्ण भूमिका निभाता है।
• दूरस्थ संवलित नलिका (DCT) : विशिष्ट परिस्थितियों में Na+ और जल का कुछ पुनरावशोषण इस भाग में होता है। दूरस्थ संवलित नलिका रक्त में सोडियम पोटैशियम का संतुलन तथा pH बनाए रखने के लिये बाइकार्बोनेट्स का पुनरावशोषण एवं H+, K+ और अमोनिया का चयनात्मक स्रवण करती है।
अतः विकल्प (c) सही उत्तर है ।
6. मूत्र में ग्लूकोज़ तथा कीटोनकाय (Keton bodies) की उपस्थिति निम्नलिखित में से किस रोग का लक्षण है?
(a) किडनी स्टोन
(b) पीलिया
(c) ग्लूकोमा
(d) मधुमेह (डायबिटीज मेलीटस
उत्तर : (d)
व्याख्या : मूत्र में ग्लूकोज़ की उपस्थिति (ग्लाइकोसूरिया) तथा कीटोनकाय की उपस्थिति (कीटोनयूरिया), मधुमेह (डायबिटीज मेलीटस) के लक्षण हैं।
• मूत्र का विश्लेषण वृक्कों के कई उपापचयी विकारों का पता लगाने और रोग के निदान में मदद करता है।
अतः विकल्प (d) सही उत्तर है।
7. निम्नलिखित कथनों पर विचार कीजिये :
1. एक वयस्क मनुष्य प्रतिदिन औसतन 1-1.5 किग्रा. यूरिया का उत्सर्जन करता है।
2. मूत्र का pH मान लगभग 6 होता, जो इसके अम्लीय होने का प्रतीक है।
उपर्युक्त कथनों में से कौन सा/से सही है/हैं?
(a) केवल 1
(b) केवल 2
(c) 1 और 2 दोनों
(d) न तो 1 और न ही 2
उत्तर : (b)
8. निम्नलिखित में से कौन से अंग उत्सर्जन में सहायक होते हैं?
1. फेफड़े
2. यकृत
3. त्वचा
4. वृक्क
नीचे दिये गए कूट का प्रयोग कर सही उत्तर चुनिये :
(a) केवल 2 और 4
(b) केवल 2, 3 और 4
(c) केवल 1, 3 और 4
(d) 1, 2, 3 और 4
उत्तर : (d)

20. गमन एवं संचलन

1. चलन के संदर्भ में निम्नलिखित कथनों पर विचार कीजिये :
1. चलन के लिये केवल पेशीय तथा कंकाल तंत्र की आवश्यकता होती है।
2. सभी चलन गति होते हैं, लेकिन सभी गति चलन नहीं हैं।
3. जंतुओं के चलन के तरीके प्रायः परिस्थिति की मांग और आवास के अनुरूप बदलते हैं।
उपर्युक्त कथनों में से कौन-सा/से सत्य है/हैं?
(a) केवल 1
(b) केवल 2
(c) केवल 1 और 3
(d) केवल 2 और 3
उत्तर : (d)
व्याख्या: कथन 1 असत्य है क्योंकि चलन के लिये केवल पेशीय तथा कंकाल तंत्र की नहीं बल्कि पेशीय, कंकाल और तंत्रिका तंत्र की पूर्ण समन्वित क्रिया की आवश्यकता होती है।
• गति और चलन का पृथक रूप से अध्ययन नहीं किया जा सकता। उदाहरण के लिये हाइड्रा अपने स्पर्शक शिकार पकड़ने एवं चलन दोनों के लिये प्रयोग कर सकता है तथा हम अपने पैरों को शरीर की मुद्रा बदलने तथा चलन दोनों के लिये प्रयोग में लाते हैं। इन प्रेक्षणों के आधार पर दोनों के संबंध को इस उक्ति में समाहित किया जा सकता है कि सभी चलन गति होते हैं, लेकिन सभी गति चलन नहीं हैं। अतः कथन 2 सत्य है।
• जंतुओं के चलन के तरीके प्रायः परिस्थिति की मांग और आवास के अनुरूप बदलते हैं। फिर भी चलन की क्रिया प्रायः भोजन, आश्रय, साथी, अनुकूल प्रजनन स्थल, अनुकूल प्राकृतिक स्थिति की तलाश या शत्रुओं / भक्षियों से पलायन के लिये की जाती है। अतः कथन 3 सत्य है।
अतः विकल्प (d) सही उत्तर है।
2. निम्नलिखित में से कौन-सी अरेखित पेशियाँ हैं ?
(a) कंकाल पेशियाँ
(b) अंतरंग पेशियाँ
(c) हृद पेशियाँ
(d) उपर्युक्त सभी
उत्तर : (b)
व्याख्या: स्थापन के आधार पर पेशियों के तीन प्रकार हैं- 1. कंकाल पेशियाँ (Skeletal Muscles); 2. अंतरंग पेशियाँ (Visceral Muscles); 3. हृद पेशियाँ (Cardiac Muscles)
• कंकाल पेशियाँ शारीरिक कंकाल अवयवों के निकट संपर्क में होती हैं। सूक्ष्मदर्शी द्वारा देखने पर इनमें धारियाँ दिखती हैं, अत: इन्हें रेखित पेशी कहते हैं।
• अंतरंग पेशियाँ शरीर के खोखले अंतरंग अंगों, जैसे- आहार नाल, जनन मार्ग आदि की भीतरी भित्ति में स्थित होती हैं। ये अरेखित और चिकनी दिखती हैं।
• हृद पेशियाँ हृदय की पेशियाँ हैं। कई हृद पेशी कोशिकाएँ हृद पेशी के गठन के लिये शाश्वत रचना में एकत्रित होती हैं। रंग रूप के आधार पर, हृद पेशियाँ रेखित होती हैं।
अतः विकल्प (b) सही उत्तर है।
3. पेशी के संदर्भ में निम्नलिखित कथनों पर विचार कीजिये :
1. एक वयस्क मनुष्य के शरीर के भार का लगभग 40-50 प्रतिशत हिस्सा पेशियों का होता है।
2. पेशी रेशा (Muscle Fibre), पेशी की संरचनात्मक इकाई है।
उपर्युक्त कथनों में से कौन-सा/से सही है/हैं?
(a) केवल 1
(b) केवल 2
(c) 1 और 2 दोनों
(d) न तो 1 और न ही 2
उत्तर : (c)
व्याख्या : पेशी एक विशेष प्रकार का ऊतक है जिसकी उत्पत्ति मंध्यत्वचा (Mesodermal) में होती है। एक वयस्क मनुष्य के शरीर के भार का 40-50 प्रतिशत हिस्सा पेशीय ऊतकों का होता है। इनके कई विशेष गुण होते हैं, जैसे- उत्तेजनशीलता, संकुचनशीलता, प्रसार्य एवं प्रत्यास्थता । अतः कथन 1 सही है।
• पेशी रेशा (Muscle Fibre), पेशी की संरचनात्मक इकाई है। प्रत्येक पेशी रेशा प्लाज़्मा झिल्ली से आस्तरित (Lined) होता है जिसे सार्कोलेमा कहते हैं। अतः कथन 2 सही है।
अतः विकल्प (c) सही उत्तर है।
4. निम्नलिखित में से थकान का मुख्य कारण क्या है?
(a) लैक्टिक अम्ल के जमाव के कारण
(b) एब्सीसिक अम्ल के जमाव के कारण
(c) मायोग्लोबिन की मात्रा बढ़ने के कारण
(d) हाइड्रोक्लोरिक अम्ल की अधिकता के कारण
उत्तर : (a)
व्याख्या : पेशियों के बार-बार उत्तेजित होने पर उनमें ग्लाइकोजन के अवायवीय विखंडन से लैक्टिक अम्ल का जमाव होने लगता है जिसके कारण थकान होती है।
• पेशी में ऑक्सीजन भंडारित करने वाला लाल रंग का प्रोटीन मायोग्लोबिन होता है। कुछ पेशियों में मायोग्लोबिन की मात्रा ज़्यादा होती है जिससे वे लाल रंग के दिखते हैं। ऐसी पेशियों को ‘लाल पेशियाँ’ कहते हैं।
अतः विकल्प (a) सही उत्तर है।
5. पेशियों के लाल रंग के लिये निम्नलिखित में से किसकी अधिकता उत्तरदायी है?
(a) हीमोग्लोबिन
(b) मायोग्लोबिन
(c) माइटोकॉण्ड्रिया
(d) (a) और (b) दोनों
उत्तर : (b)
व्याख्या : हीमोग्लोबिन रक्त के लाल रंग के लिये उत्तरदायी है।
• पेशी में ऑक्सीजन भंडारित करने वाला लाल रंग प्रोटीन ‘मायोग्लोबिन’ होता है। कुछ पेशियों में मायोग्लोबिन की मात्रा ज्यादा होती है जिससे वे लाल रंग की दिखती हैं। ऐसी पेशियों को ‘लाल पेशियाँ’ कहते हैं। ऐसी पेशियों में माइटोकॉण्ड्रिया अधिक होता है जो ATP के निर्माण हेतु उनमें भंडारित ऑक्सीजन की बड़ी मात्रा का उपयोग कर सकता है। इसलिये इन पेशियों को ‘वायुजीवी पेशियाँ’ भी कह सकते हैं।
• वहीं दूसरी तरफ, कुछ पेशियों में मायोग्लोबिन की बहुत कम मात्रा पाई जाती है जिससे वे हल्के रंग की अथवा श्वेत प्रतीत होती हैं। ये ‘श्वेत पेशियाँ’ हैं। इनमें माइटोकॉण्ड्रिया तो अल्पसंख्यक होता है, लेकिन पेशीद्रव्य जालिका (Sarcoplasmic Reticulum) अधिक मात्रा में होती है। ये अवायवीय विधि द्वारा ऊर्जा प्राप्त करती हैं।
अतः विकल्प (b) सही उत्तर है।
6. अस्थि एवं उपास्थि हैं-
(a) संयोजी ऊतक
(b) उपकला ऊतक
(c) पेशी ऊतक
(d) तंत्रिका ऊतक
उत्तर : (a)
7. मानव कंकाल तंत्र के संबंध में निम्नलिखित कथनों पर विचार कीजिये :
1. अक्षीय कंकाल में 80 अस्थियाँ होती हैं जो शरीर के मुख्य अक्ष पर वितरित होती हैं।
2. पसलियों की कुल 12 जोड़ियाँ होती हैं ।
3. फीमर सबसे लंबी अस्थि है।
उपर्युक्त कथनों में से कौन-सा/से सही है/हैं?
(a) केवल 1 और 3
(b) 1, 2 और 3
(c) केवल 3
(d) केवल 1
उत्तर : (b)
8. संधियाँ या जोड़ों के संदर्भ में निम्नलिखित में से कौन-सा कथन सत्य नहीं है?
(a) जोड़ अस्थियों अथवा एक अस्थि एवं एक उपास्थि के बीच का संधि स्थल है।
(b) रेशीय जोड़ (Fibrous Joints) सभी प्रकार की गति में सहायक होते हैं।
(c) साइनोवियल जोड़ चलन सहित कई तरह की गति में सहायता करते हैं।
(d) सैडल जोड़ साइनोवियल जोड़ का उदाहरण है।
उत्तर : (b)
9. पेशीय व कंकाल तंत्र के विकार के संदर्भ में निम्नलिखित युग्मों पर विचार कीजिये :
1. गाउट (Gout) : जोड़ों की शोथ (सूजन)
2. अस्थि सुषिरता (Osteoporosis) : एस्ट्रोजन स्तर में कमी
3. पेशीय दुष्पोषण (Muscular dystrophy) : कंकाल पेशी का अनुक्रमित अपह्रासन
उपर्युक्त युग्मों में से कौन-से सुमेलित हैं?
(a) केवल 1 और 3
(b) केवल 2 और 3
(c) 1, 2 और 3
(d) केवल 1 और 2
उत्तर : (c)
व्याख्या : पेशीय व कंकाल तंत्र के विकार :
• पेशीय दुष्पोषण (Muscular Dystrophy) : विकारों के कारण कंकाल पेशी का अनुक्रमित अपह्रासन |
• अपतानिका (Tetany) : शरीर में कैल्शियम आयनों की कमी से पेशी में तीव्र ऐंठन |
• अस्थि सुषिरता (osteoporosis ) : यह उम्र संबंधित विकार जिसमें अस्थि के पदार्थों में कमी से अस्थि भंग की प्रबल संभावना होती है। एस्ट्रोजन स्तर में कमी इसका सामान्य कारक है।
• गाउट (Gout) : जोड़ों में यूरिक एसिड के जमा होने के कारण जोड़ों में शोथ (सूजन)। तीनों युग्म सही सुमेलित हैं।
अतः विकल्प (c) सही उत्तर है।

21. तंत्रिकीय नियंत्रण एवं समन्वय

1. मानव तंत्रिका तंत्र के संदर्भ में निम्नलिखित कथनों पर विचार कीजिये :
1. मस्तिष्क तथा मेरुरज्जु केंद्रीय तंत्रिका तंत्र में सम्मिलित हैं।
2. अनुकंपी तंत्रिका तंत्र स्वायत्त तंत्रिका तंत्र का एक भाग है।
3. अंतरंग तंत्रिका तंत्र (Visceral Nervous System), परिधीय तंत्रिका तंत्र का एक भाग है।
उपर्युक्त कथनों में से कौन-सा/से सही है/हैं?
(a) केवल 1
(b) केवल 1 और 3
(c) केवल 2
(d) 1, 2 और 3
उत्तर : (d)
2. निम्नलिखित में से तंत्रिका तंत्र की संरचनात्मक एवं क्रियात्मक इकाई के संदर्भ में सर्वाधिक उपयुक्त विकल्प कौन-सा है ?
(a) माइटोकॉण्ड्रिया
(b) नेफ्रॉन
(c) न्यूरॉन
(d) कोशिका
उत्तर : (c)
3. निम्नलिखित कथनों पर विचार कीजिये :
1. तंत्रिका आवेगों का एक न्यूरॉन से दूसरे न्यूरॉन तक संचरण सिनेप्सिस द्वारा होता है।
2. रासायनिक-सिनेप्सिस से आवेग का संचरण, विद्युतीय-सिनेप्सिस से संचरण की तुलना में अधिक तीव्र होता है।
उपर्युक्त कथनों में से कौन-सा/से सही है/हैं?
(a) केवल 1
(b) केवल 2
(c) 1 और 2 दोनों
(d) न तो 1 और न ही 2
उत्तर : (a)
व्याख्या : तंत्रिका आवेगों का एक न्यूरॉन से दूसरे न्यूरॉन तक संचरण सिनेप्सिस द्वारा होता है । एक सिनेप्सिस का निर्माण पूर्व सिनैप्टिक न्यूरॉन तथा पश्च सिनैप्टिक न्यूरॉन की झिल्ली द्वारा होता है। अतः कथन 1 सही है।
• सिनेप्सिस दो प्रकार के होते हैं- विद्युतीय सिनेप्सिस एवं रासायनिक सिनेप्सिस । विद्युतीय सिनेप्सिस से आवेग का संचरण, रासायनिक सिनेप्सिस से संचरण की तुलना में अधिक तीव्र होता है। हमारे तंत्र में विद्युतीय सिनेप्सिस बहुत कम होते हैं। अतः कथन 2 गलत है।
अतः विकल्प (a) सही उत्तर है।
4. मानव मस्तिष्क के संदर्भ में निम्नलिखित कथनों पर विचार कीजिये :
1. अग्र मस्तिष्क सेरीब्रम, थैलेमस और हाइपोथैलेमस का बना होता है।
2. हाइपोथैलेमस शरीर के तापमान एवं खाने-पीने का नियंत्रण करता है।
3. पोंस पश्च मस्तिष्क का हिस्सा है जो रेशेनुमा पथ का बना होता है।
उपर्युक्त कथनों में से कौन-सा/से सही है/हैं?
(a) केवल 1
(b) केवल 1 और 3
(c) केवल 2 और 3
(d) 1, 2 और 3
उत्तर : (d)
व्याख्या : मस्तिष्क हमारे शरीर का केंद्रीय सूचना प्रसारण अंग है और यह ‘आदेश व नियंत्रण तंत्र’ की तरह कार्य करता है।
• अग्र मस्तिष्क (Forebrain) सेरीब्रम, थैलेमस और हाइपोथैलेमस का बना होता है। सेरीब्रम (प्रमस्तिष्क) मानव मस्तिष्क का एक बड़ा भाग बनाता है।
• सेरीब्रम थैलेमस नामक संरचना के चारों ओर लिपटा होता है, जो कि संवेदी और प्रेरक संकेतों का मुख्य संपर्क स्थल है। थैलेमस के आधार पर मस्तिष्क का दूसरा भाग हाइपोथैलेमस स्थित होता है। हाइपोथैलेमस में कई केंद्र होते हैं जो शरीर के तापमान, खाने-पीने का नियंत्रण करते हैं। अतः कथन 1 और कथन 2 सही हैं।
• पश्च मस्तिष्क ( Hindbrain) पोंस, अनुमस्तिष्क और मध्यांश (मेड्यूला ओबलोंगेटा) का बना होता है। पोंस रेशेनुमा पथ का बना होता है जो कि मस्तिष्क के विभिन्न भागों को आपस में जोड़ते हैं। अतः कथन 3 सही है।
अतः विकल्प (d) सही उत्तर है।
5. नेत्र के संबंध में निम्नलिखित में से कौन-सा कथन असत्य है ?
(a) नेत्र की दीवारें तीन परतों की बनी होती हैं।
(b) इसकी सबसे बाहरी परत घने संयोजी ऊतकों की बनी होती है, जिसे कोरॉइड (रक्त पटल) कहते हैं।
(c) लेंस के सामने आइरिस से घिरा हुआ एक छिद्र होता है, जिसे ‘प्यूपिल’ कहते हैं।
(d) इसकी आंतरिक परत रेटिना (दृष्टि पटल) कहलाती है और यह कोशिकाओं की तीन तंत्रिकीय परतों से बनी होती है।
उत्तर : (b)
व्याख्या : वयस्क मनुष्य के नेत्र की संरचना लगभग गोलाकार होती है।
• नेत्र की दीवारें तीन परतों की बनी होती हैं।
• बाहरी परत घने संयोजी ऊतकों की बनी होती है जिसे स्क्लेरा (श्वेत पटल) कहते हैं। इसका अग्र भाग कॉर्निया कहलाता है। अतः विकल्प (b) असत्य है।
• मध्य परत कोरॉइड (रक्त पटल) में अनेक रक्त वाहनियाँ होती हैं और यह हल्के नीले रंग की दिखती है।
• लेंस के सामने आइरिस से घिरा हुआ एक छिद्र होता है, जिसे ‘प्यूपिल’ कहते हैं। ‘प्यूपिल’ के घेरे का नियंत्रण आइरिस के पेशी तंतु करते हैं।
• नेत्र की आंतरिक परत रेटिना (दृष्टि पटल) कहलाती है और यह कोशिकाओं की तीन तंत्रिकीय परतों से बनी होती है।
अतः विकल्प (b) सही उत्तर है।
6. मानव नेत्र के संबंध में ‘एक्वस चैंबर’ क्या है?
(a) लेंस और रेटिना के बीच का रिक्त स्थान
(b) नेत्र गोलक के भीतर पारदर्शी क्रिस्टलीय लेंस
(c) कॉर्निया और लेंस के बीच का स्थान
(d) इनमें से कोई नहीं
उत्तर : (c)
व्याख्या: कॉर्निया और लेंस के बीच के स्थान को ‘एक्वस चैंबर’ (जलीय कोष्ठ) कहते हैं। जिसमें पतला जलीय द्रव नेत्रोद (Aqueous Humor ) पाया जाता है।
• लेंस और रेटिना के बीच रिक्त स्थान को द्रव कोष्ठ (Vitreous Chamber) कहते हैं और यह पारदर्शी द्रव काचाभ द्रव (Vitreous Humor) कहलाता है।
अतः विकल्प (c) सही उत्तर है।
7. कर्ण के संदर्भ में निम्नलिखित कथनों पर विचार कीजिये :
1. बाह्य कर्ण पिन्ना (Pinna) या ऑरीकुला और बाह्य श्रवण गुहा का बना होता है तथा पिन्ना वायु में उपस्थित तरंगों को एकत्र करता है जो ध्वनि उत्पन्न करती है।
2. मध्य कर्ण दो अस्थिकाओं से बना होता है – इंकिस एवं स्टेपीज |
3. यूस्टेकियन नलिका मध्यपूर्ण गुहा को फेरिंक्स से जोड़ती है।
उपर्युक्त कथनों में से कौन-सा/से सही है/हैं?
(a) केवल 2
(b) केवल 3
(c) केवल 1 और 2
(d) केवल 1 और 3
उत्तर : (d)
व्याख्या : शरीर की क्रिया विज्ञान की दृष्टि से कर्ण को तीन में बाँटा गया है- बाह्य कर्ण, मध्य कर्ण और अंतः कर्ण । मुख्य भागों
• बाह्य कर्ण पिन्ना (Pinna) या ऑरीकुला तथा बाह्य श्रवण गुहा का बना होता है। पिन्ना वायु में उपस्थित तरंगों को एकत्र करता है जो ध्वनि उत्पन्न करती है । पिन्ना तथा मीटस में कुछ महीन बाल और मोम स्रावित करने वाली ग्रंथियाँ होती हैं। अतः कथन 1 सत्य है।
• मध्य कर्ण तीन अस्थिकाओं से बना होता है जिन्हें मैलियस, इंकिस और स्टेपीज कहते हैं। ये एक-दूसरे से श्रृंखला के रूप में जुड़ी रहती हैं। मैलियस कर्ण पटल झिल्ली से और स्टेपीज कोक्लिया (Cochlea) की अंडाकार खिड़की से जुड़ी होती है। अतः कथन 2 असत्य है।
• कर्ण अस्थिकाएँ ध्वनि तरंगों को अंतः कर्ण तक पहुँचाने की क्षमता बढ़ाती हैं। यूस्टेकियन नलिका मध्यकर्ण गुहा को फेरिंक्स (Pharynx) से जोड़ती है। यूस्टेकियन नलिका कर्ण पटल के दोनों ओर दाब को समान रखती हैं। अतः कथन 3 सत्य है।
अतः विकल्प (d) सही उत्तर है।
8. कर्ण के संबंध में ‘कोक्लिया (Cochlea ) ‘ क्या है?
(a) द्रव से भरा अंतः कर्ण
(b) लेबरिंथ (labyrinth) का घुमावदार भाग
(c) यूट्रीकल में उभारनुमा संरचना
(d) इनमें से कोई नहीं
उत्तर : (b)
व्याख्या : द्रव से भरा अंतः कर्ण लेबरिंथ (labyrinth) कहलाता है, जो कि अस्थिल और झिल्लीनुमा लेबरिंथ से बना होता है । अत: विकल्प (a) गलत है।
• लेबरिंथ के घुमावदार भाग को ‘कोक्लिया’ कहते हैं। कोक्लिया (Cochlea) को दो झिल्लियों द्वारा तीन कक्षों में विभक्त किया जाता है, जिन्हें बेसिलर झिल्ली और राइजनर्स झिल्ली कहते हैं। अतः विकल्प (b) सही है।
• यूट्रीकल में उभारनुमा संरचना मैक्यूला कहलाती है। क्रिस्टा व मैक्यूला (Macula) शरीर के संतुलन व सही स्थिति के लिये उत्तरदायी होते हैं। विकल्प (c) गलत है।
अतः विकल्प (b) सही उत्तर है।
9. निम्नलिखित में से कर्ण कौन-सी संवेदी क्रिया करते हैं?
(a) सुनना
(b) शरीर का संतुलन
(c) आवेगों का संचरण
(d) (a) और (b) दोनों
उत्तर : (d)
व्याख्या: कर्ण दो संवेदी क्रियाएँ करते हैं सुनना एवं शरीर का संतुलन बनाना ।
• शरीर क्रिया विज्ञान की दृष्टि से कर्ण को तीन मुख्य भागों में विभक्त किया जा सकता है— (i) बाह्य कर्ण (ii) मध्य कर्ण (iii) अंतः कर्ण
• तंत्रिका आवेगों का एक न्यूरॉन से दूसरे न्यूरॉन तक संचरण सिनेप्सिस द्वारा होता है। अतः विकल्प (c) गलत है।
अतः विकल्प (d) सही उत्तर है।

22. रासायनिक समन्वय तथा एकीकरण

1. अंत:सावी ग्रंथियों के संदर्भ में निम्नलिखित कथनों पर विचार कीजिये :
1. अंतःस्रावी ग्रंथियाँ नलिकाविहीन ग्रंथियाँ होती हैं।
2. पीयूष ग्रंथि एक अंतःस्रावी ग्रंथि है।
उपर्युक्त कथनों में से कौन-सा/से सही है/हैं?
(a) केवल 1
(b) केवल 1
(c) 1 और 2 दोनों
(d) न तो 1 और न ही 2
उत्तर : (c)
व्याख्या : अंतःस्रावी ग्रंथियों में नलिकाएँ नहीं होती हैं इसलिये ये नलिकाविहीन ग्रंथियाँ कहलाती हैं। अतः कथन 1 सही है।
• अंतःस्रावी ग्रंथियाँ और शरीर के विभिन्न भागों में स्थित हार्मोन स्रावित करने वाले ऊतक / कोशिकाएँ मिलकर अंत: स्रावी तंत्र का निर्माण करती हैं। पीयूष ग्रंथि, पिनियल ग्रंथि, थायरॉइड, एड्रिनल, अग्न्याशय, पैराथायरॉइड, थाइमस और जनन ग्रंथियाँ हमारे शरीर के सुनियोजित अंतःस्रावी अंग हैं। अतः कथन 2 सही है।
अतः विकल्प (c) सही उत्तर है।
2. निम्नलिखित कथनों पर विचार कीजिये :
1. न्यूरोहाइपोफाइसिस या पश्च पीयूष ग्रंथि हाइपोथैलेमस द्वारा उत्पादित किये जाने वाले हार्मोन ऑक्सीटोसिन और वैसोप्रेसिन का संग्रह और स्राव करती है।
2. पिनियल ग्रंथि अग्र मस्तिष्क के पृष्ठीय (ऊपरी भाग में स्थित होती है।
3. पिनियल ग्रंथि मेलेटोनिन नामक हार्मोन स्रावित करती है।
उपर्युक्त कथनों में से कौन-सा/से सही है/हैं?
(a) केवल 1
(b) केवल 2 और 3
(c) केवल 3
(d) 1, 2 और 3
उत्तर : (d)
व्याख्या : पीयूष ग्रंथि एक सेला टर्सिका नामक अस्थिल गुहा में स्थित होती है और एक वृत द्वारा हाइपोथैलेमस से जुड़ी होती है। न्यूरोहाइपोफाइसिस या पश्च पीयूष ग्रंथि, हाइपोथैलेमस द्वारा उत्पादित किये जाने वाले हार्मोन ऑक्सीटोसिन और वैसोप्रेसिन (Oxytocin and Vasopressin) का संग्रह और स्राव करती है। ये हार्मोन वास्तव में हाइपोथैलेमस द्वारा संश्लेषित होते हैं और तंत्रिका से होते हुए पश्च पीयूष ग्रंथि में पहुँचा दिये जाते हैं। अतः कथन 1 सही है।
• पिनियल ग्रंथि अग्र मस्तिष्क के पृष्ठीय (ऊपरी भाग में स्थित होती है । पिनियल ग्रंथि मेलेटोनिन हार्मोन स्रावित करती है। मेलेटोनिन (Melatonin) हमारे शरीर के दैनिक लय (24 घंटे) के नियमन का एक महत्त्वपूर्ण कार्य करता है। उदाहरण के लिये यह सोने-जागने के चक्र एवं शरीर के तापक्रम को नियंत्रित करता है। अतः कथन 2 और 3 सही हैं।
अतः विकल्प (d) सही उत्तर है।
3. थायरॉइड ग्रंथि के संदर्भ में निम्नलिखित कथनों पर विचार कीजिये :
1. थायरॉइड हार्मोन के सामान्य दर से संश्लेषण के लिये आयोडीन आवश्यक है।
2. थायरॉइड हार्मोन लाल रक्त कणिकाओं (RBC) के निर्माण की प्रक्रिया में सहायता करते हैं।
3. एक्सोथैलमिक ग्वायटर थायरॉइड अतिक्रियता का एक रूप है।
उपर्युक्त कथनों में से कौन-सा/से सही है/हैं?
(a) केवल 1
(b) केवल 1 और 3
(c) केवल 2
(d) 1, 2 और 3
उत्तर : (d)
व्याख्या : थायरॉइड ग्रंथि श्वास नली के दोनों ओर स्थित दो पालियों से बनी होती है। दोनों पालियाँ ऊतक के पतली पल्लीनुमा इस्थमस से जुड़ी होती हैं।
• थायरॉइड हार्मोन के सामान्य दर से संश्लेषण के लिये आयोडीन आवश्यक है। हमारे भोजन में आयोडीन की कमी से अवथायरॉइडता एवं थायरॉइड ग्रंथि की वृद्धि हो जाती है, जिसे साधारणतया गलगंड कहते हैं। अतः कथन 1 सही है।
• थायरॉइड हार्मोन आधारीय उपापचयी दर के नियमन में मुख्य भूमिका निभाते हैं। ये हार्मोन लाल रक्त कणिकाओं (RBC) के निर्माण की प्रक्रिया में भी सहायता करते हैं। अतः कथन 2 सही है।
• थायरॉइड ग्रंथि से एक प्रोटीन हार्मोन, थाइरोकैल्शिटोनिन (TCT) का भी स्राव होता है जो रक्त में कैल्शियम स्तर को नियंत्रित करता है। नेत्रोत्सेधी गलगंड (एक्सोथैलमिक ग्वायटर) थायरॉइड अतिक्रियता का एक रूप है। थायरॉइड ग्रंथि में वृद्धि, नेत्र गोलकों का बाहर की ओर उभर आना, आधारी उपापचय दर में वृद्धि एवं भार में ह्रास इसके अभिलक्षण हैं। अतः कथन 3 सही है।
अतः विकल्प (d) सही उत्तर है।
4. थाइमस ग्रंथि के संबंध में निम्नलिखित में से कौन-सा कथन असत्य है?
(a) यह ग्रंथि थायरॉक्सिन नामक हार्मोन का स्राव करती है।
(b) यह प्रतिरक्षा तंत्र के विकास में महत्त्वपूर्ण भूमिका निभाती है।
(c) वृद्धों में प्रतिरक्षा प्रतिक्रिया का कमज़ोर होना, थाइमोसिन हार्मोन की कमी के कारण है।
(d) उपर्युक्त सभी कथन सत्य हैं।
उत्तर : (a)
व्याख्या : थाइमस ग्रंथि महाधमनी के उदर पक्ष पर उरोस्थि (Sternum) के पीछे फेफड़ों के बीच स्थित एक पॉलीयुक्त संरचना है।
• थाइमस ग्रंथि प्रतिरक्षा तंत्र के विकास में महत्त्वपूर्ण भूमिका निभाती है।
• यह ग्रंथि थाइमोसिन नामक पेप्टाइड हार्मोन का स्राव करती है जबकि थायरॉक्सिन हार्मोन का स्राव थायरॉइड ग्रंथि के द्वारा होता है। अतः विकल्प (a) असत्य है।
• थाइमोसिन टी-लिंफोसाइट्स के विभेदीकरण में मुख्य भूमिका निभाते हैं जो कोशिका माध्य प्रतिरक्षा के लिये महत्त्वपूर्ण है।
• बढ़ती उम्र के साथ थाइमस का अपघटन होने लगता है, फलस्वरूप थाइमोसिन का उत्पादन घट जाता है। इसी के परिणामस्वरूप वृद्धों में प्रतिरक्षा प्रतिक्रिया कमज़ोर पड़ जाती है।
अतः विकल्प (a) सही उत्तर है।
5. निम्नलिखित में से किस कारण से गर्भावस्था के समय गर्भ में विकसित हो रहे बालक की वृद्धि विकृत हो जाती है?
(a) ल्यूटीनाइजिंग हार्मोन की कमी के कारण
(b) मेलेटोनिन हार्मोन की अधिकता के कारण
(c) थाइरोकैल्शिटोनिन के कारण
(d) अवथायरॉइडता के कारण
उत्तर : (d)
व्याख्या : थायरॉइड हार्मोन के सामान्य दर से संश्लेषण के लिये आयोडीन आवश्यक है। हमारे भोजन में आयोडीन की कमी से अवथायरॉइडता एवं थायरॉइड ग्रंथि की वृद्धि हो जाती है ।
• गर्भावस्था के समय अवथायरॉइडता के कारण गर्भ में विकसित हो रहे बालक की वृद्धि विकृत हो जाती है। इससे बच्चे की अवरोधित वृद्धि (क्रिटेनिज्म) या वामनता तथा मंदबुद्धि, त्वचा असमान्यता, मूक बधिरता आदि हो जाती है। वयस्क स्त्रियों में अवथायरॉइडता मासिक चक्र को अनियमित कर देता है।
अतः विकल्प (d) सही उत्तर है।
6. लैंगर हैंस द्वीप में पाई जाने वाली B कोशिकाएँ निम्नलिखित में से किस हार्मोन का स्राव करती हैं?
(a) ग्लूकागॉन
(b) इंसुलिन
(c) एंड्रोजन
(d) वृद्धि हार्मोन
उत्तर : (b)
व्याख्या : अंतःस्रावी अग्न्याशय ‘लैंगरहैंस द्वीपों’ से निर्मित होता है। प्रत्येक लैंगरहैंस द्वीप में मुख्य रूप से दो प्रकार की कोशिकाएँ होती हैं, जिन्हें α और β कोशिकाएँ कहते हैं ।
• α कोशिकाएँ ग्लूकागॉन का तथा β कोशिकाएँ इंसुलिन हार्मोन का स्राव करती हैं।
अतः विकल्प (b) सही उत्तर है।
7. निम्नलिखित कथनों पर विचार कीजिये :
1. अग्न्याशय केवल एक अंतःस्रावी ग्रंथि है।
2. अंतःस्रावी अग्न्याशय ‘लैंगरहैंस द्वीपों’ से निर्मित होता है।
3. गैस्ट्रिन हार्मोन, जठर ग्रंथियों पर कार्य कर हाइड्रोक्लोरिक अम्ल और पेप्सिनोजेन के स्राव को प्रेरित करता है।
उपर्युक्त कथनों में से कौन सा/से सही है/हैं?
(a) केवल 1
(b) केवल 2
(c) केवल 3
(d) केवल 2 और 3
उत्तर : (d)
व्याख्याः अग्न्याशय एक संयुक्त ग्रंथि है जो अंतःस्रावी और बहि:स्रावी दोनों के रूप में कार्य करती है। अतः कथन 1 गलत है।
• अत: स्रावी अग्न्याशय ‘लैंगर हैंस द्वीपों’ से निर्मित होता है। साधारण मनुष्य के अग्न्याशय में लगभग 10 से 20 लाख लैंगरहैंस द्वीप होते हैं, जो अग्न्याशयी ऊतकों का 1 से 2 प्रतिशत होता है। प्रत्येक लैंगर हैंस द्वीप में मुख्यतः दो प्रकार की कोशिकाएँ होती हैं जिन्हें a और B कोशिकाएँ कहते हैं। अत: कथन 2 सही है।
• जठर आंत्रीय पथ के विभिन्न भागों में उपस्थित अंतःस्रावी कोशिकाएँ चार मुख्य पेप्टाइड हार्मोन का स्राव करती हैं; गैस्ट्रिन, सेक्रेटिन, कोलिसिस्टोकाइनिन और जठर अवरोधी पेप्टाइड (जी आई पी )।
• गैस्ट्रिन, जठर ग्रंथियों पर कार्य कर हाइड्रोक्लोरिक अम्ल (HCL) और पेप्सिनोजेन के स्राव को प्रेरित करता है। अतः कथन 3 सही है ।
अतः विकल्प (d) सही उत्तर है।

Leave a Reply

Your email address will not be published. Required fields are marked *